Sei sulla pagina 1di 152

Bulls Eye

XAT 2013

www.hitbullseye.com

SECTION A
Verbal and Logical Ability
DIRECTIONS for questions 1 to 3: Choose the most appropriate option after reading the following
statements.
1.

A. Whether due to haste or design, the new laws are marked by vagueness, leaving officials all down
the organization's bureaucratic chain great latitude in enforcing them.
B. The opacity of the language leaves the law open to manipulation on political grounds.
1. Statement B can be induced from statement A.
2. Statement A can be induced from statement B.
3. Statement B can be deduced from statement A.
4. Statement A can be deduced from statement B.
5. Statements A and B are independent.

2.

Choose the most appropriate option after reading the following statements.
A. If there is any endeavour whose fruits should be freely available, that endeavour is surely publicly
financed science.
B. There is a widespread feeling that the journal publishers who have mediated the scientific exchange
for the past century or more are becoming an impediment to free distribution of knowledge.
C. Internet revolution is happening, making knowledge transfer cheaper. Technology permits it;
researchers and politicians want it, more public money can be spent on it.
1. Statement B definitely illustrates statement A.
2. Statement C is a facilitating condition for statement A
3. Statement C states a condition under which statement A would be invalid
4. Statement B can be deduced from statement C but independent of Statement A.
5. Statement A, B and C are necessarily independent.

3.

Choose the most appropriate option after reading the following statements.
A. Business schools are ideally positioned to point out when an action that provides a benefit for an
individual comes at a cost to society, but in reality they rarely bother.
B. It is part of the malaise that has befallen the political debate on capitalism, which has been taken
over by special interests and people who have no faith in a real market-based system.
C. When governments favours the private sector it is all too often by being "pro-business" rather than
"pro-market", meaning that favourable conditions are provided to particular institutions rather than to
institutions broadly.
1. Statements A and B are necessarily dependent.
2. Statements B and C are necessarily dependent.
3. Statements B and C may be dependent.
4. Statements A, B and C cannot be independent.
5. All the three statements are necessarily independent.

DIRECTIONS for questions 4 to 6: Fill in the blanks with the most appropriate option that follows.
4.

_____________ wolf, meeting with ___________lamb astray from ___________fold, resolved not to
lay violent hands on him, but to find some plea to justify to ____________ lamb ___________ wolfs
right to eat him.
1. a, a, the, the, the
4. the, the, the, the, the

2. the, a, the, a, the


5. the, a, the, a, a

3. a, a, a, the, the

5.

Bulls Eye

XAT 2013

www.hitbullseye.com

____________ bat who fell upon ____________ground and was caught by ____________weasel
pleaded to be spared his life ____________ weasel refused, saying that he was by nature
____________ enemy of all birds. _____________ bat assured him that he was not ___________ bird,
but _____________ mouse, and thus was set free.
1. a, the, a, the, the, the, the, a
3. the, a, a, the, the, a, the, the
5. the, a, a, a, the, the, a, a

6.

2. a, the, a, the, a, the, a, a


4. a, the, a, the, the, the, a, a

He got ____________ next morning, to be sure, and had his meals ___________ usual, though he ate
_____________ and had more, I am afraid, than his usual supply of rum, for he helped himself
_____________ the bar, scowling and blowing ___________ his nose, and no one dared ___________
cross him.
1. Down, like, a little, out of, out, to
3. Downstairs, as, little, out of, out of, through
5. Down, like, a little, of, of, through

2. Down, as, little, of, out, to


4. Downstairs, like, a little, out, of, to

DIRECTIONS for questions 7 to 9: Read the following sentences and choose the option that best arranges
them in a logical order.
7.

A. I was scarcely in position ere my enemies began to arrive, seven or eight of them, running hard, their
feet beating out of time along the road and the man with the lantern some paces in front.
B. My curiosity, in a sense, was stronger than my fear, for I could not remain where I was, but crept
back to the bank again, whence, sheltering my head behind a bush of broom, I might command the road
before our door.
C. Three men ran together, hand in hand; and I made out, even through the mist, that the middle man of
this trio was the blind beggar.
D. The next moment his voice showed me that I was right.
1. A, B, C, D

8.

3. A, B, D, C

4. A, C, D, B

5. A, D, B, C

A. Finally he took a wrong turn and ran a few steps past me, towards the hamlet, crying, "Johnny, Black
Dog, Dirk," and other names, "you won't leave old Pew, mates - not old Pew!"
B. This quarrel was the saving of us, for while it was still raging, another sound came from the top of
the hill on the side of the hamletthe tramp of horses galloping.
C. And that was plainly the last signal of danger, for the buccaneers turned at once and ran, separating
in every direction, one seaward along the cove, one slant across the hill, and so on, so that in half a
minute not a sign of them remained but Pew.
D. Him they had deserted, whether in sheer panic or out of revenge for his ill words and blows I know
not; but there he remained behind, tapping up and down the road in a frenzy, and groping and calling
for his comrades.
E. Almost at the same time a pistol-shot, flash and report, came from the hedge side.
1. E, A, C, D, B

9.

2. B, A, C, D

2. A, B, C, E, D

3. B, E, C, D, A

4. D, C, B, E, A

5. B, E, D, C, A

A. As chroniclers of an incremental process, they discover that additional research makes it harder, not
easier, to answer questions like: When was oxygen discovered? Who first conceived of energy
conservation?
B. Simultaneously, these same historians confront growing difficulties in distinguishing the "scientific"
component of past observation and belief from what their predecessors had readily labeled "error" and
"superstition."
C. Increasingly, a few of them suspect that these are simply the wrong sorts of questions to ask. Perhaps
science does not develop by the accumulation of individual discoveries and inventions.

Bulls Eye

XAT 2013

www.hitbullseye.com

D. In recent years, however, a few historians of science have been finding it more and more difficult to
fulfill the functions that the concept of development-by-accumulation assigns to them.
1. B, A, C, D

2. D, C, A, B

3. D, B, C, A

4. D, C, B, A

5. D, A, C, B

DIRECTIONS for questions 10 to 12: Read the following sentence and choose the best alternative which
should replace the italicized part of the sentence.
10.

To be a great manager requires, strong inter-personal skills, the ability to think fast, and demands a
can-do attitude.
1. requires strong inter-personal skills, the ability to think fast, and demands
2. requires strong inter-personal skills, the ability to think fast, and
3. requires strong inter-personal skills, demands the ability to think fast, and
4. requiring strong inter-personal skill, an ability to think fast, and demands
5. demands strong inter-personal skills, an ability to think fast, but with

11.

The tremendous insight of Einstein was that the passage of time does not appear to be the same while
standing still as it does to a person traveling at a speed which is a significant fraction of the speed of
light.
1. while standing still as it does to a person traveling
2. to a person standing still as to a person traveling
3. to a person who is standing still as a person who is traveling
4. while standing still as to traveling
5. to a person standing still as to a person who travels

12.

Economic theory fails to explain the extent to which savings from personal income has shifted to shortterm bonds, money-market funds, and other near-term investments by the instability in the futures
market.
1. to which savings from personal income has shifted
2. of savings from personal income that has been shifted
3. of savings from personal income shifting
4. to which savings from personal income have shifted
5. to which savings from personal income have been shifted

DIRCTIONS for questions 13 to 16: Analyse the following passage and provide appropriate answers for the
questions that follow.
An example of scientist who could measure without instruments is Enrico Fermi (1901-1954), a physicist who
won the Nobel Prize in physics in 1938. He had a well-developed knack for intuitive, even casual-sounding
measurements. One renowned example of his measurement skills was demonstrated at the first detonation of the
atom bomb the Trinity Test site, on July 16, 1945, where he was one of the atomic scientists observing the blast
from base camp. While final adjustments were being made to instruments used to measure the yield of the blast,
Fermi was making confetti out of a page of notebook paper. As the wind from the initial blast wave began to
blow through the camp, he slowly dribbled the confetti into the air, observing how far back it was scattered by
the blast (taking the farthest scattered pieces as being the peak of the pressure wave). Fermi concluded that the
yield must be greater than 10 kilotons. This would have been news, since other initial observers of the blast did
not know that lower limit. After much analysis of the instrument readings, the final yield estimate was
determined to be 18.6 kilotons. Like Eratosthenes, Fermi was aware of a rule relating one simple observation
the scattering of confetti in the wind to a quantity he wanted to measure.

Bulls Eye

XAT 2013

www.hitbullseye.com

The value of quick estimates was something Fermi was familiar with throughout his career. He was famous for
teaching his students skills at approximation of fanciful-sounding quantities that, at first glance, they might
presume they knew nothing about. The best-known example of such a "Fermi question" was Fermi asking his
students to estimate the number of piano tuners in Chicago, when no one knows the answer. His students
science and engineering majors would begin by saying that they could not possibly know anything about
such a quantity. Of course, some solutions would be to simply do a count of every piano tuner perhaps by
looking up advertisements, checking with a licensing agency of some sort, and so on. But Fermi was trying to
teach his students how to solve problems where the ability to confirm the results would not be so easy. He
wanted them to figure out that they knew something about the quantity in question.
13.

Suppose you apply the same logic as Fermi applied to confetti, which of the following statements
would be the most appropriate?
1. You can calculate the minimum pressure inside the cooker by calculating the maximum distance
travelled by any of its parts after it explodes.
2. You can calculate the average potency of a fire cracker by calculating the distance covered by one of
its bigger fragments.
3. You can easily find out the average potency of an earthquake by measuring the length of a crack it
makes on the surface of the earth.
4. You can calculate the exact volume of water stored in a tank by measuring the distance covered by
the stream of water coming out of the tap fixed on the lower corner of the tank.
5. All of the above conclusions can be drawn.

14.

Quick estimate, as per Fermi, is most useful in:


1. In finding an approximate that is more useful than existing values.
2. In finding out the exact minimum value of an estimate
3. In finding out the exact maximum value of an estimate
4. In finding out the range of values of an estimate
5. In finding out the average value of an estimate

15.

Given below are some statements that attempt to capture the central idea of the passage:
A. It is useful to estimate: even when the exact answer is known.
B. It is possible to estimate any physical quantity.
C. It is possible to estimate the number of units of a newly launched car that can be sold in a city.
D. Fermi was a genius.
Which of the following statement(s) best captures the central idea?
1. A, B and D

16.

2. B, C and D

3. B and C

4. B only

5. A, B and C

Read the statements given below:


A. Atomic bomb detonation was a result of Fermi's Nobel Prize contribution
B. Fermi's students respected him as a scientist
C. Yield of atomic bomb can only be measured in Kilotons
Which of the following statement(s) can be inferred from the passage?
1. A, B
4. B only

2. B, C
3. A, C
5. None of the three statements is correct

Bulls Eye

XAT 2013

www.hitbullseye.com

DIRECTIONS for questions 17 to 21: Analyse the following passage and provide appropriate answers for the
questions that follow.
"Whatever actions are done by an individual in different embodiments, [s]he reaps the fruit of those actions in
those very bodies or embodiments (in future existences) ".
A belief in karma entails, among other things, a focus on long run consequences, i.e., a long term orientation.
Such an orientation implies that people who believe in karma may be more honest with themselves in general
and in setting expectations in particulara hypothesis we examine here. This research is based on three simple
premises. First, because lower expectations often lead to greater satisfaction, individuals in general, and
especially those who are sensitive to the gap between performance and expectations, have the incentive to and
actually do "strategically" lower their expectations. Second, individuals with a long term orientation are likely
to be less inclined to lower expectations in the hope of temporarily feeling better. Third, long term orientation
and the tendency to lower expectations are at least partially driven by cultural factors. In India, belief in karma,
with its emphasis on a longer term orientation, will therefore to some extent counter-act the tendency to lower
expectations. The empirical results support our logic; those who believe more strongly in karma are less
influenced by disconfirmation sensitivity and therefore have higher expectations.
Consumers make choices based on expectations of how alternative options will perform (i.e., expected utility).
Expectations about the quality of a product also play a central role in subsequent satisfaction. These
expectations may be based on a number of factors including the quality of a typical brand in a category,
advertised quality, and disconfirmation sensitivity. Recent evidence suggests that consumers, who are more
disconfirmation sensitive (i.e., consumers who are more satisfied when products perform better than expected or
more dissatisfied when products perform worse than expected) have lower expectations. However, there is little
research concerning the role of culture-specific variables in expectation formation, particularly how they relate
to the impact of disconfirmation sensitivity on consumer expectations.
17.

"Future existences" in the first paragraph can refer to:


A. Human life, 5 years afterwards
B. Next birth in human form
C. Next birth in any embodiment
Which of the following statement(s) is correct?
1. A, B

18.

3. A, C

4. B only

5. None of the three

Consider the following assertion and conclusion:


Assertion: The meaning of karma in the above passage (refer to first two lines of the paragraph in
italics).
Conclusion: Belief that long term consequences are important.
Now read the following statements carefully:
A. The conclusion will always follow the assertion.
B. The conclusion may follow the assertion.
C. The conclusion may follow the assertion only if an individual lives long enough.
D. The conclusion cannot follow the assertion.
Which of the following statement(s) is correct?
1. A only

19.

2. B, C

2. A and B

3. B only

4. C only

5. D only

Which of following statements, if true, would contradict the first of the three premises mentioned in the
first paragraph?
1. Higher satisfaction leads to lower expectation.
2. Lower expectation leads to long term consequences
3. Satisfaction depends on achievement and not on expectation
4. Karma affects our immediate feelings
5. Lower expectation would lead to lower efforts

20.

www.hitbullseye.com

Read the following statements carefully:


A. Temporary feelings and law of karma are independent.
B. As per theory of karma, temporary feelings would not lower the expectation.
C. Temporary feelings and law of karma are contradictory.
Which of the following combination of statements is consistent with the second premise?
1. A only

21.

Bulls Eye

XAT 2013

2. A and B

3. A and C

4. C only

5. A, B and C

A manager went out to have dinner in a restaurant and found the food to be good. When asked to
provide feedback on the quality of food, the manager rated the quality as "excellent".
Which of the following can be concluded from this?
1. The manager does not believe in karma.
2. The manager definitely has discontinuation sensitivity.
3. It is not possible to comment on the disconfirmation sensitivity of the manager.
4. The manager does not have disconfirmation sensitivity.
5. None of the above.

DIRECTIONS for questions 22 to 26: Analyse the following passage and provide appropriate answers for the
questions that follow.
Soros, we must note, has never been a champion of free market capitalism. He has followed for nearly all his
public life the political ideas of the late Sir Karl Popper who laid out a rather jumbled case for what he dubbed
"the open society" in his The Open Society and Its Enemies (1953). Such a society is what we ordinarily call the
pragmatic system in which politicians get involved in people's lives but without any heavy theoretical
machinery to guide them, simply as the ad hoc parental authorities who are believed to be needed to keep us all
on the straight and narrow. Popper was at one time a Marxist socialist but became disillusioned with that idea
because he came to believe that systematic ideas do not work in any area of human concern.
The Popperian open society Soros promotes is characterized by a very general policy of having no firm
principles, not even those needed for it to have some constancy and integrity. This makes the open society a
rather wobbly idea, since even what Popper himself regarded as central to all human thinking, critical
rationalism, may be undermined by the openness of the open society since its main target is negative: avoid
dogmatic thinking, and avoid anything that even comes close to a set of unbreachable principles. No, the open
society is open to anything at all, at least for experimental purposes. No holds are barred, which, if you think
about it, undermines even that very idea and becomes unworkable.
Accordingly, in a society Soros regards suited to human community living, the state can manipulate many
aspects of human life, including, of course, the economic behavior of individuals and firms. It can control the
money supply, impose wage and price controls, dabble in demand or supply-side economics, and do nearly
everything a central planning board might -provided it does not settle into any one policy firmly, unbendingly.
That is the gist of Soros's Popperian politics.
Soros' distrusts capitalism in particular, because of the alleged inadequacy of neoclassical economics, the
technical economic underpinnings of capitalist thinking offered up in many university economics departments.
He, like many others outside and even inside the economics discipline, finds the arid reductionism of this social
science false to the facts, and rightly so. But the defense of capitalist free markets does not rest on this position.
Neo-classical thinking depends in large part on the 18th- and 19th-century belief that human society operates
according to laws, not unlike those that govern the physical universe. Most of social science embraced that
faith, so economics isn't unusual in its loyalty to classical mechanics. Nor do all economists take the
deterministic lawfulness of economic science literally - some understand that the laws begin to operate only
once people embark upon economic pursuits. Outside their commercial ventures, people can follow different

Bulls Eye
www.hitbullseye.com

XAT 2013

principles and priorities, even if it is undeniable that most of their endeavors have economic features. Yet, it
would be foolish to construe religion or romance or even scientific inquiry as solely explicable by reference to
the laws of economics.
In his criticism of neo-classical economic science, then, George Soros has a point: the discipline is too
dependent on Newtonian physics as the model of science. As a result, the predictions of economists who look at
markets as if they were machines need to be taken with a grain of salt. Some - for example the school of
Austrian economists - have made exactly that point against the neo-classical.
Soros draws a mistaken inference: if one defense of the market is flawed, the market lacks defense. This is
wrong. If it is true that from A we can infer B, it does not prove that B can only be inferred from A; C or Z, too,
might be a reason for B.
22.

As per the paragraph, author believes that


1. Free market capitalism can be explained using neo-classical economics.
2. Neo-classical economics does not address the idea of free-market system.
3. Free market capitalism and open society are not different from each other.
4. Free market capitalism and laissez-faire are not different from each other.
5. Technical underpinning of neo-classical economics can address the idea of laissez-faire.

23.

As per the paragraph, which of the following statements is true?


1. Economic benefits of open society and laissez-faire are same.
2. Soros' open society means no interference from the government.
3. Free market capitalism means no interference from the government.
4. Laws of economics are not capable of explaining the human nature completely.
5. Laws of economics capture the human nature completely as most of the human endeavors are
economic in nature.

24.

According to the author,


1. George Soros believes in regulated economies.
2. George Soros does not believe in government intervention in state policies.
3. George Soros believes in state intervention provided it does not remain static.
4. George Soros believes that laissez-faire economies perform better than free-market economies.
5. George Soros believes that free-market economies perform better than controlled economies.

25.

According to the author which of the following statement could be true about critical rationalism.
1. Ideas of critical rationalism underpin the foundation of neo-classical economics.
2. Ideas of critical rationalism underpin the foundation of laissez-faire.
3. Ideas of critical rationalism underpin the foundation of open society.
4. Ideas of critical rationalism underpin the foundation of Newtonian physics.
5. None of the above.

26.

The word deterministic (used in fourth line of fifth paragraph), in the above passage refers to:
1. An effect can only be caused by a single event.
2. An effect may be produced by many causes.
3. An effect cannot be produced by a cause.
4. Cause(s) of an effect can always be known.
5. Economics does not follow cause and effect relationship.

XAT 2013

Bulls Eye
www.hitbullseye.com

DIRECTIONS for questions 27 to 30: Analyse the following passage and provide appropriate answers for the
questions that follow.
Popper claimed, scientific beliefs are universal in character, and have to be so if they are to serve us in
explanation and prediction. For the universality of a scientific belief implies that, no matter how many instances
we have found positive, there will always be an indefinite number of unexamined instances which may or may
not also be positive. We have no good reason for supposing that any of these unexamined instances will be
positive, or will be negative, so we must refrain from drawing any conclusions. On the other hand, a single
negative instance is sufficient to prove that the belief is false, for such an instance is logically incompatible with
the universal truth of the belief. Provided, therefore, that the instance is accepted as negative we must conclude
that the scientific belief is false. In short, we can sometimes deduce that a universal scientific belief is false but
we can never induce that a universal scientific belief is true.
It is sometimes argued that this 'asymmetry' between verification and falsification is not nearly as pronounced
as Popper declared it to be. Thus, there is no inconsistency in holding that a universal scientific belief is false
despite any number of positive instances; and there is no inconsistency either in holding that a universal
scientific belief is true despite the evidence of a negative instance. For the belief that an instance is negative is
itself a scientific belief and may be falsified by experimental evidence which we accept and which is
inconsistent with it. When, for example, we draw a right-angled triangle on the surface of a sphere using parts
of three great circles for its sides, and discover that for this triangle Pythagoras' Theorem does not hold, we may
decide that this apparently negative instance is not really negative because it is not a genuine instance at all.
Triangles drawn on the surfaces of spheres are not the sort of triangles which fall within the scope of
Pythagoras' Theorem. Falsification, that is to say, is no more capable of yielding conclusive rejections of
scientific belief than verification is of yielding conclusive acceptances of scientific beliefs. The asymmetry
between falsification and verification, therefore, has less logical significance than Popper supposed.
We should, though, resist this reasoning. Falsifications may not be conclusive, for the acceptances on which
rejections are based are always provisional acceptances. But, nevertheless, it remains the case that, in
falsification, if we accept falsifying claims then, to remain consistent, we must reject falsified claims. On the
other hand, although verifications are also not conclusive, our acceptance or rejection of verifying instances has
no implications concerning the acceptance or rejection of verified claims. Falsifying claims sometimes give us a
good reason for rejecting a scientific belief, namely when the claims are accepted. But verifying claims, even
when accepted, give us no good and appropriate reason for accepting any scientific belief, because any such
reason would have to be inductive to be appropriate and there are no good inductive reasons.
27.

According to Popper, the statement "Scientific beliefs are universal in character" implies that
1. Positive instances of scientific belief imply that it is universal in character.
2. There are equal numbers of negative and positive instances of a universal scientific belief.
3. If there are negative and positive instances of a scientific belief then it cannot be universal.
4. We can only deduce that a scientific belief is false but cannot induce that it is true.
5. We can only induce that a scientific belief is false but cannot induce that it is true.

28.

The statement, "this 'asymmetry' between verification and falsification is not nearly as pronounced as
Popper declared it to be", implies that
1. Falsification is better than verification in universal acceptance of scientific beliefs.
2. Verification is better than falsification in universal acceptance of scientific beliefs.
3. Both falsification and verification together can result in universal acceptance of scientific beliefs.
4. Capability of falsification in accepting of scientific beliefs is not better than that of verification in
rejection of scientific beliefs.
5. Capability of falsification in rejection of scientific beliefs is not always better than that of verification
in acceptance of scientific beliefs.

Bulls Eye
www.hitbullseye.com

29.

With which of the following statements, would the author agree most?
1. Verification is better than falsification in establishing the claims.
2. Falsification and verification are equally good in establishing the claims.
3. Verification and falsification are equally bad in establishing the claims.
4. Falsification is better than verification in disproving the claims.
5. Verification is better than falsification in disproving the claims.

30.

Which of the following would be the most appropriate conclusion?


1. Falsification gives us an appropriate reason for rejecting a scientific belief.
2. Falsification gives us all the reasons for accepting a claim.
3. Verification gives us a reason for rejecting a claim.
4. Verification gives us an appropriate reason for accepting a scientific belief.
5. Verification gives us an appropriate reason for rejecting a scientific belief.

XAT 2013

10

Bulls Eye

XAT 2013

www.hitbullseye.com

SECTION B
Decision Making
DIRECTIONS for questions 31 to 32: Read the following caselet and choose the best alternative for the
questions that follow.
Island of Growth was witnessing a rapid increase in GDP. Its citizens had become wealthier in recent times, and
there had been a considerable improvement in the standards of living. However, this rapid growth had increased
corruption and nepotism in the Island. In the recent times, a fear had gripped the population that corruption
would destroy the inclusive nature of the society and hinder economic progress. However, most citizens had
kept quiet because:
1. they had benefitted from the corruption indirectly, if not directly.
2. they did not have the time and energy to protest.
3. they did not have courage to rise against the established power centers.
There was a need to remove corruption but no one was willing to stick his neck out. Many politicians,
bureaucrats and private organizations were corrupt. Media and intellectuals kept quiet, as they benefitted
indirectly from corruption. The common man was scared of state's retribution and the youngsters feared
insecure future.
Against this background, an old, unmarried and illiterate gentleman of high moral and ethical authority,
Shambhu, decided to take on the issue of corruption. He sat on a hunger strike in the heart of the capital city of
the Island. Shambhu demanded that the Government should constitute new laws to punish the corrupt across all
walks of life. Media and the citizens of the island gave massive support to Shambhu. Buckling under the
pressure, the Government promised to accept Shambhu's demands. He ended the hunger strike immediately
following the Government's announcement. Shambhu became the darling of the media. He used this
opportunity as a platform to spread the message that only citizens with an unblemished character should be
allowed to hold a public office.
A few months later, it was found that the Government had not fulfilled any of its promises made to Shambhu.
Infuriated, he was thinking of launching another island-wide protest. However, this time, he sensed that not
many people and media persons were willing to support him.
31.

Read the following statements:


A. People's latent anger against corruption
C. Hungry media looking to raise issues.

B. Shambhu's moral courage


D. Rising income level

In your opinion, which combination of the above statements is the most unlikely reason for Shambhu's
initial success?
1. A,B,D
32.

2. C,D

3. A,C

4. A,C,D

4. A,B,C,D

Which of the following could be the most likely reason for decline in public support for Shambhu?
1. The common man had become sick and tired of Government's inaction against rising corruption.
2. Shambhu was old and he lacked the energy to garner the same support that he enjoyed from the
media and the public in the initial stages.
3. The general public may have realized that Shambhu was focusing too much on "indirect
involvement" in past incidences of corruption. Common men found it difficult to live up to the high
standards set by Shambu.
4. Shambhu's colleagues were misleading him.
5. Shambhu came from a village, while most of his supporters were city dwellers.

Bulls Eye
www.hitbullseye.com

XAT 2013

11

DIRECTIONS for questions 33 to 35: Read the following caselet and choose the best alternative for the
questions that follow.
Head of a nation in the Nordic region was struggling with the slowing economy on one hand and restless
citizens on the other. In addition, his opponents were doing everything possible to discredit his government. As
a famous saying goes, "There is no smoke without a fire", it cannot be said that the incumbent government was
doing all the right things. There were reports of acts of omission and commission coming out every other day.
Distribution of public resources for private businesses and for private consumption had created a lot of
problems for the government. It was being alleged that the government has given the right to exploit these
public resources at throw-away prices to some private companies. Some of the citizens were questioning the
government policies in the Supreme Court of the country as well as in the media, hi the midst of all this, the
head of the nation called his cabinet colleagues for a meeting on the recent happenings in the country.
He asked his minister of water resources about the bidding process for allocation of rights to setup mini-hydel
power plants. To this, the minister replied that his ministry had followed the laid out policies of the government.
Water resources were allocated to those private companies that bid the highest and were technically competent.
The minister continued that later on some new companies had shown interest and they were allowed to enter the
sector as per the guidelines of ;i|e(jbyefnmeht. This, the minister added, would facilitate proper utilization of
water resources and provide better services to the citizens. The new companies were allocated the rights at the
price set by the highest bidders in the previous round of bidding. After hearing this, the head of the nation
replied that one would expect the later allocations to be done after a fresh round of bidding. The minister of
water resources replied that his ministry had taken permissions from the concerned ministries before allocating
the resources to the new companies.
33.

Media reports suggested that the minister of water resources had deliberately allocated the water
resources at old prices to the new companies, and in return some received kickbacks. However, the
minister denied these charges. His counter argument was that he followed the stated policies of the
Government and it is very difficult to price a scarce resource. He also said that the loss that the media is
talking about is notional and in reality the Government and the citizens have gained by the entry of new
players. Which of the following is the most appropriate inference?
1. If benefit to the citizens is higher than notional losses then it is not unethical.
2. If benefit to the citizens is lower than notional losses then it is unethical.
3. If benefit to the citizens is higher than actual losses then it is not unethical.
4. If benefit to the citizens is lower than actual losses then it is unethical.
5. All of the above A, B, C and D are inappropriate.

34.

Subsequently, the minister questioned the role of the media in the whole affair. He said that the media
cannot act like a reporter, prosecutor and judge at the same time. Mr. Swamy, an independent observer,
was asked about appropriateness of the minister's opinion. What should be Mr. Swamy's reply?
1. Media has been rightly accused by the minister.
2. Minister's statement may be factually incorrect.
3. Media has rightly accused the minister.
4. Media has wrongly accused the minister.
5. None of above

35.

Looking at the public unrest and discontent, the Government's anti-corruption branch was entrusted
with the task of investigating the matter. Within a week's time the branch charge- sheeted top corporate

12

Bulls Eye

XAT 2013

www.hitbullseye.com

managers and the minister for wrong doings. Mr. Swamy was again asked to identify theTguilty. Who
should Mr. Swamy pick?
1. Only corporate managers.
3. Only the Head of the nation.
5. None of the above.
36.

2. Only the minister.


4. All of the above.

Over the last five years, Bank of Bharat has seen the number of its retail customer accounts drop by
over 40 percent. Over the same period, the share price of Bank of Bharat has increased by more than 80
percent. This amazed a few investors, who believe that a bank's share price should drop if its number of
customers drops.
Which of the following, if true over the last five years, best accounts for the observed movement in the
price of Bank of Bharat's equity shares?
1. Two years ago Securities and Exchange Board started an investigation on the bank for accounting
irregularities, but last year the company was cleared of all charges.
2. The bank recently implemented a highly publicized program for free home loans.
3. The bank has been switching its customer base from retail customers to commercial customers,
which now accounts for over 75 percent of the bank's revenues.
4. There have been many new banks, which have entered retail banking business over the last five
years.
5. The bank is known to be the best paymaster in the industry.

37.

Gastric bypass surgery has been shown to be effective at helping extremely obese people lose weight.
Some patients have lost as much as 300 pounds after undergoing the surgery, thereby substantially
prolonging their lives. Despite the success of the treatment, most doctors have not embraced the
surgery.
Which of the following statements, if true, best accounts for the lukewarm reaction of the medical
community to gastric bypass surgery?
1. Gastric bypass surgery carries a high risk of serious complications, including death.
2. Obesity is one of the leading contributors to heart disease and hypertension, two leading causes of
death.
3. Incidences of obesity among the Indian urban middle class population have been increasing
consistently for the last three decades.
4. Many patients report that losing weight through diets is ineffective, since they usually gain the
weight back within six months.;
5. Most health insurance plans will cover the cost of gastric bypass surgery for morbidly obese 1
patients at high risk of heart disease.

DIRECTIONS for questions 38 to 41: Read the following caselet and choose the best alternative for the
questions that follow.
The BIG and Colourful Company
You are running "BIG and Colourful (BnC)" company that sells books to customers through three retail
formats:
1. You can buy books from bookstores,
2. You can buy books from supermarket,
3. You can order books over the Internet (Online).
Your manager has an interesting way of classifying expenses: some of the expenses are classified in terms of
size: Big, Small and Medium; and others are classified in terms of the colors, Red, Yellow, Green and Violet.

Bulls Eye

XAT 2013

www.hitbullseye.com

13

The company has a history of categorizing overall costs into initial cost* and additional costs. Additional costs
are equal to the sum of Big, Small and Medium expenses. There are two types of margins, contribution (sales
minus initial costs) and profit (contribution minus additional costs). Given below is the data about sales and
costs of BnC:
Sales

60000

Initial Costs

39000

Contribution (Sales - Initial Costs)

21000

Additional Costs
Big

9300

Small

3000

Medium

3500
15800

Profit (Contribution -Additional Cost)

5200

Each of the Big, Small and Medium cost is categorized by the manager into Red, Yellow, Green and Violet
costs. Breakdown of the additional costs under these headings is shown in the table below:
Expenses

Total

Red

Yellow

Green

Violet

Big
Small
Medium

9300
3000
3500

5100
400

1200
400
1500

1400
2000
1400

1600
600
200

Total

15800

5500

3100

4800

2400

Red, Yellow, Green and Violet costs are allocated to different retail formats. These costs are apportioned in the
ratio of number of units consumed by each retail format. The number of units consumed by each retail format is
given in the table below:
Retail Format
Online

38.

Red
200

Yellow
50

Green
50

Violet
50

Supermarket

65

20

21

21

Book Store

10

30

Total

275

100

80

80

Read the following statements:


Statement I: Online store accounted for 50% of the sales at BnC and the ratio of supermarket sales and
book store sales is 1:2.
Statement II: Initial Cost is allocated in the ratio of sales.
If you want to calculate the profit/loss from the different retail formats, then

1. Statement I alone is sufficient to calculate the profit/loss.


2. Statement II alone is sufficient to calculate the profit/loss.
3. Both statements I and II are required to calculate the profit/loss.
4. Either of the two statements is sufficient to calculate the profit/loss.
5. Neither Statement I nor Statement II is sufficient to calculate the profit/loss.
DIRECTIONS for questions 38 to 41: Based on your answer to the above question, answer for the questions
that follow.

14

39.

www.hitbullseye.com

What is the profit/loss from "online" sales?


1. 0
4. + 450

40.

2. -310
3. + 20
5. Cannot be determined from given information

Which retail format is least profit making for BnC?


1. Online
4. All formats are loss making.

41.

Bulls Eye

XAT 2013

2. Supermarket
5. All formats are profit making.

3. Book Store

Which retail format gives the highest profit for BnC?


1. Book Store
4. All are equally profitable.

2. Supermarket
3. Online
5. Cannot be determined from given information.

DIRECTIONS for questions 42 to 43: Read the following caselet and choose the best alternative for the
questions that follow.
A teacher wanted to administer a multiple choice (each question having six choices) based quiz of high
difficulty level to a class of sixty students. The quiz had sixty questions. The probability of selecting the correct
answer for a good student and a brilliant student was 0.2 and 0.25 respectively. The poor students had no
learning advantage. The teacher did not want students to cheat but does not have time and resources to monitor.
All students were seated serially in 10 rows and 6 columns.
42.

Is it possible for the teacher to detect cheating without monitoring? Choose the statement that best
describes your opinion:
1. It is not at all possible; teacher will have to introduce technology if there is no human support.
2. It is always possible; but teacher has to calculate the exact answer.
3. It is possible when many students sitting next to each other have the same incorrect answers for
multiple questions. However, there can be a small error in judgment.
4. It is possible when many students sitting next to each other have the same correct answers for
multiple questions. However, there can be a small error in judgment.
5. It is possible only for poor students but not for good and brilliant students. However, there can be a
small error in judgment.

43.

Three good students were seated next to each other. What is the probability of them having the same
incorrect choice for four consecutive questions?
1. 256/390625
4. 1/3125

44.

2. 256/3125
5. Cannot be calculated

3. 4/3125

Students from four sections of a class accompanied by respective class teachers planned to go for a field
trip. There were nineteen people in all. However, on the scheduled day one of the four teachers and a
few students could not join the rest. Given below are some statements about the group of people who
ultimately left for the trip.
I: Section A had the largest contingent.
II: Section B had fewer students than Section A.
III: Section C's contingent was smaller than Section B.
IV: Section D had the smallest contingent.
V: The product of the number of student from each section is a multiple of 10.
VI: The number of students from Section C is more than 2.

Bulls Eye

XAT 2013

www.hitbullseye.com

15

VII. The product of the number of students from each section is a multiple of 24.
VIII. The largest contingent has more than 4 students. IX: Each section contributed different number of
students
The statements that taken together can give us the exact number of students from each section:
1. I, II, III, IV, VI
3. I, II, III, IV, V, VI
45.

2. I, VI, VIII, IX
4. I, II, III, IV, VII

5. I, IV, VI, VII, IX

There is a lot of interest in the first five ranks for Class XI students. One student guessed the rank order
as Ankita, Bhagyashree, Chanchal, Devroopa and Esha. Later upon announcement of the results, it was
found that not only did he get each student out off her true position, none of the students in his ranking
correctly followed her immediate predecessor. Another student guessed Devroopa, Ankita, Esha,
Chanchal and Bhagyashree. Even his guess was wrong. It was found that he had got two positions
correct, and two students in his ranking correctly followed their immediate predecessors. Which of the
following is true about the correct rank order?
1. Ankita got the third position
3. Chanchal got the second position
5. Esha got the fourth position

2. Bhagyashree got the fourth position


4. Devroopa stood first

DIRECTIONS for questions 46 to 48: Read the following caselet and choose the best alternative for the
questions that follow.
Marathe is a Vice President in a construction equipment company in the city of Mumbai. One day, his
subordinate Bhonsle requested that Kale, a project manager, be transferred to the Chennai office from the
Mumbai office. In Chennai, Kale would work alone as a researcher. Bhonsle gave the following reasons for his
request: "Kale is known to frequently fight with his colleagues. Kale is conscientious and dedicated only when
working alone. He is friendly with seniors, but refuses to work with colleagues, in a team. He cannot accept
criticism and feels hostile and rejected. He is over-bearing and is generally a bad influence on the team"
Marathe called upon Gore, another project manager, and sought further information on Kale. Gore recalled that
a former colleague, Lakhote (who was also Kale's former boss) had made a few remarks on his appraisal report
about Kale. In his opinion, Kale was not fit for further promotion as he was emotionally unstable to work in
groups though he had seven years of work experience. Lakhote had described Kale as too authoritative to work
under anyone. Lakhote had further told Gore that Kale had an ailing wife, and an old mother, who does not
want to stay with his wife.
46.

Consider the following solutions to the problem mentioned above:


A. Marathe should transfer Kale to Chennai office
B. Marathe should try and verify the facts from other sources as well
C. Kale should be sacked
D. Kale should be demoted
E. Marathe should suggest Kale to visit a family counselor
Which of the following would be the most appropriate sequence of decisions in terms of immediacy:
starting from immediate to a longer term solution?
1. B, A, E

47.

2. A, D, E

3.B, C, D

4. B, E, A

5. B, E, D

Marathe sought an appointment with Lakhote to find out ways to help Kale. Lakhote is of the opinion
that the company's responsibility is restricted to the workplace and it should not try to address the

16

Bulls Eye

XAT 2013

www.hitbullseye.com

personal problems of employees. If Marathe has to agree to Lakhote's opinion, which of the solutions
presented in the previous question would be weakened:
1. 1
48.

2. 2

3. 3

4. 4

5. 5

Which of the following statements, if true, would weaken the decision to sack Kale the most?
1. A Government of India study established that employees with 5-10 years of work experience tend to
have conflicting responsibilities at home and office. However, these conflicts wither away after 10
years of experience.
2. Another article published in the magazine, Xaviers Quarterly, highlighted that employees' problems
at home affect their performance at work
3. In the latest issue of a reputed journal, Xaviers Business Review, it was published that most top
managers find it difficult to work in a group.
4. It was published in Xaviers Management Review (another reputed journal) that individuals who
cannot work in teams find it difficult to adjust to a new location.
5. Bhonsle was of the opinion that emotionally unstable persons find it difficult to get back to normal
working life.

DIRECTIONS for questions 49 to 51: Read the following caselet and choose the best alternative for the
questions that follow.
It was the end of performance review cycle for the year 2012 when you asked your subordinates about any
problems they were facing. Natrajan told you that an important member of his team, Vardarajan, who had also
won the best performance award for the year 2011, was not taking interest in work. Despite Natrajan's
counseling, no change was noticed in Vardarajan, rather his attitude deteriorated. You had also received such
information from other employees. You had not interfered hoping that Natrajan, an experienced hand, would be
able to solve the problem. But now that Natrajan himself brought this to your notice, you decided to call
Vardarajan and counsel him.
49.

Which of the following could be the most unlikely reason for Vardarajan's declining involvement in the
workplace?
1. Vardarajan does not find the work challenging enough as he has already achieved the best
performance award.
2. Others in the organization have been trying to pull him down, since he was declared best performer.
3. Vanlarajan was not promoted after his superlative performance, while another colleague,
Sundararajan, was promoted although he was not as good a performer as Vardarajan.
4. After putting in lots of effort for the superlative performance, Vardarajan felt burnt out.
5. Vardarajan was appreciated by his basses for his achievement last year

50.

Vardarajan did not find his work challenging enough, given below are some steps that could be taken to
motivate him:
A. Give Vardarajan a more challenging assignment
B. Transfer Vardarajan from Projects department to Training department
C. Ask him to take a vacation for two months
D. Send him for further training on decision making under stress
1. A, B, C

51.

2. A, C, D

3. A, D

4. B, C D

5. A, B, C, D

You overheard a conversation between Vardarajan and his colleague over an official dinner. He
expressed his unhappiness about the fact that good performers were not given their due credit while

Bulls Eye
www.hitbullseye.com

XAT 2013

17

poor performers were promoted faster. If Vardarajan is right, which of the following steps would help
in creating a better organization?
1. Promote Vardarajan with immediate effect.
2. Ensure that performance is objectively and transparently assessed.
3. Give another assignment to Vardarajan.
4. Give higher salary to Vardarajan.
5. Fire Vardarajan.
DIRECTIONS for questions 52 to 55: Read the following caselet and choose the best alternative for the
questions that follow.
Prof. Vijya, the chairperson of the Faculty Academic Committee (FAC), was trying to understand the
implications of decisions taken by the Student Placement Committee (SPC) on placement issues.
It was alleged that Biswajit, a final year student, inflated his grades in his bio-data that was sent to the
recruiters. The President of SPC requested the FAC to debar Biswajit from the campus recruitment process.
When the matter was brought up for discussion in FAC, one of the professors remarked that Biswajit too should
be allowed to defend himself. When Biswajit arrived for the meeting the situation became even more
challenging.
Biswajit raised the issue that many other students who had misrepresented grades to get coveted jobs had gone
scot-free. He alleged that these students were close to the President of SPC and therefore, no action was taken
against them. He stated that somebody has deliberately manipulated his grades in the bio-data.
This allegation confused the members and it was decided to adjourn the meeting. Vijya was to decide on the
next course of action.
52.

If you were Vijya what in your opinion would be the most appropriate action?
1. Debar Biswajit and the President of SPC from the placement process as they have failed to uphold
the rules of the SPC.
2. Suspend Biswajit and the President of SPC.
3. Constitute a fact finding committee to investigate the matter and ask them to submit a report to you
within a week's time.
4. Apprise the corporate recruiters of the situation and assure them that corrective actions will be taken.
5. Both options C and D.

53.

It was found that a large section of the students have been indulging in such practices. Unfortunately,
the HR manager of a much coveted campus recruiter, who is an alumnus of the college came to know
about this. Considering yourself in the position of that HR manager, what I would be your reaction?
1. Express your displeasure and stop any further recruitment from the college.
2. Talk to your contact in the college and try to find out the truth.
3. Do not change anything and continue the process as if nothing has happened.
4. Ask the college to send a fresh set of bio-data as you wanted verified grades of the students.
5. Ask the SPC to resend the details of the short-listed students including their verified grades.

54.

As a potential entrant you are having an informal Facebook chat with one of the college seniors. You
wanted to know about the pay packages of the graduating students. The senior replied that one will be
able to understand this only after joining the college. He did not reveal any information but suggested

18

XAT 2013

Bulls Eye
www.hitbullseye.com

that it is not very difficult to get a high salary job as you have already started networking. He also
cautioned that you should not believe any rumors and you should directly contact the student body for
any further information. What will be your most appropriate choice for seeking further information
about placements?
1. This conversation will increase your interest and you will network more with the students of the
college for increasing your chances of getting a high salary job.
2. You will contact the college authorities to get more insights about the placement process.
3. You will start networking with the HR managers to understand their requirements.
4. You will try to contact the President of SPC
5. You will contact your other Facebook friends to find out about the placement activities at the
55.

Vijya found that there were many such cases of grade inflation. She was giving final touches to tbe
report when her attention was diverted by a phone call on her personal cell phone. It was from an
unlisted number. The caller conveyed to her that it will be in her interest as well as in the interest of the
college if the report is not presented to the Director. The caller also told her these findings will change
nothing only result in bad publicity for the college. The caller identified himself as a well-wisher of the
college before hanging up. Consider yourself in Vijya's position and choose an appropriate decision
from the following choices.
1. Disregard the phone call and do not share its details with others.
2. Understand the implications of the phone call and apply Fora leave.
3. Call up the Director, tell him about the phone call, and excuse yourself from the responsibility.
4. Talk to the Director and seek his opinion.
5. Constitute a different committee to investigate the "phone call" and carry on with normal activities.

Bulls Eye

XAT 2013

www.hitbullseye.com

19

SECTION C
Quantitative Ability and Data Interpretation
56.

Albela, Bob and Chulbul have to read a document of seventy eight pages and make a presentation next
day. They realize that the article is difficult to understand and they would require team work to finish
the assignment. Albela can read a page in 2 minutes, Bob in 3 minutes, and Chulbul in 4 minutes. If
they divide the article into 3 parts so that all three of them spend the equal amount of time on the
article, the number of pages that Bob should read is
1. 24

57.

3. 26

4. 27

5. 28

The taxis plying in Wasseypur have the following fare structure: Rs 20 for the first two kilometers, Rs 5
for every km in excess of 2 km and up to 10 km, and Rs 8 for every km in excess of 10 km. Bullock
carts on the other hand charge Rs 2 per km. Sardar Khan takes a taxi from the Wasseypur railway
station to his home. On the way, at a distance of 14 km from the railway station, he meets Faizal Khan,
and gets down from the taxi to talk to him. Later he takes a bullock cart to reach his home. He spends a
total of Rs 102 to reach his home from the railway station. How far is his home from the railway
station?
1. 17

58.

2. 25

2. 18

3. 19

4. 20

5. 21

Consider the expression


(a2 + a + 1) (b2 + b + 1) (c2 + c + 1) (d2 + d + 1) (e2 + e + 1)
abcde
where a, b, c, d and e are positive numbers. The minimum value of the expression is
1. 3

59.

2. 1

5. 243

2. 120 minutes
5. None of the above

3. 140 minutes

At the centre of a city's municipal park there is a large circular pool. A fish is released in the water at
the edge of the pool. The fish swims north for 300 feet before it hits the edge of the pool. It then turns
east and swims for 400 feet before hitting the edge again. What is the area of the pool?
1. 62500
4. 500000

61.

4. 100

Prof. Mandal walks to the market and comes back in an auto. It takes him 90 minutes to makes the
round trip. If he takes an auto both ways it takes him 30 minutes. On a Sunday, he decided to walk both
ways. How long would it take him?
1. 100 minutes
4. 150 minutes

60.

3. 10

2. 125000
3. 250000
5. Cannot be answered from the given data

Mr. Mehra is planning for higher education expenses of his two sons aged 15 and 12. He plans to divide
Rs 15 lakhs in two equal parts and invest in two different plans such that his sons may have access to
Rs 21 lakhs each when they reach the age of 21. He is looking for plans that will give him a simple
interest per annum. The rates of interest of the plans for his younger son and his elder son should be
1. 5% and 7.5% respectively
3. 10% and 15% respectively
5. 20% and 30% respectively

2. 8% and 12% respectively


4. 4. 15 % and 22.5% respectively

20

62.

www.hitbullseye.com

The mean of six positive integers is 15. The median is 18, and the only mode of the integers is less than
18. The maximum possible value of the largest of the six integers is
1. 26

63.

Bulls Eye

XAT 2013

2. 28

3. 30

5. 34

Ramesh bought a total of 6 fruits (apples and oranges) from the market. He found that he required one
orange less to extract the same quantity of juice as extracted from apples. If Ramesh had used the same
number of apples and oranges to make the blend, then which of the following correctly represents the
percentage of apple juice in the blend?
1. 25%
4. 66.6%

64.

4. 32

2. 33.3%
5. None of the above

3. 60%

Sara has just joined Facebook. She has 5 friends. Each of her five friends has twenty five friends. It is
found that at least two of Sara's friends are connected with each other. On her birthday, Sara decides to
invite her friends and the friends of her friends. How many people did she invite for her birthday party?
1. 105
4. 100 and 125

2. 123
5. 105 and 123

3. < 125

DIRECTIONS for questions 65 to 66: Answer the questions the data given below.
65.

Arun has to go to the country of Ten to work on a series of tasks for which he must get a permit from
the Government of Ten. Once the permit is issued, Arun can enter the country within ten days of the
date of issuance of the permit. Once Arun enters Ten, he can stay for a maximum of ten days. Each of
the tasks has a priority, and takes a certain number of days to complete. Arun cannot work on more than
one task at a time. The following table gives the details of the priority and the number of days required
for each task.
Task
T1
T2
T3
T4
T5

Priority
1
2
5
3
4

Number of Days Required


3
5
3
4
2

Arun's first priority is to complete as many tasks as possible, and then try to complete the higher
priority tasks. His last priority is to go back as soon as possible.
The tasks that Arun should try to complete are:
1. T1 and T2
66.

2. T1, T2 and T5

3. T1, T4, and T5

4. T1, T2 and T4

5. T1, T3 and T4

However, Arun's manager has told him to do some background research on the tasks before leasing for
Ten. At the same time, there is no guarantee that the Government of Ten will give the permit to Arun.
Background research involves substantial costs, and therefore Arun has decided TfiKfewHlnot start his
background research without getting the permit.
The following table gives the details of the priority, the number of days required for each task and the
number of days required for background researph on each task.

Bulls Eye

XAT 2013

www.hitbullseye.com

Task

Priority

21

Number of Days Required

No. of Days Required for


Background Research
T1
1
3
3
T2
2
5
5
T3
5
3
2
T4
3
4
2
T5
4
2
3
Arun's first priority is to complete as many tasks as possible, and then try to complete the higher
priority tasks. His last priority is to go back as soon as possible within ten days.
The tasks that Arun should try to complete are:
1. T1, T2, and T3
67.

74.

4. 23

5. 22

2. 0.5 and <0.75

3. 0.75 and <1.0

4. 1.0 and <1.25

5. 1.25

2. 8

3. 10

4. 12

5. 14

2. 214

3. 220

4. 240

5. 248

2. 50 and <60

3. 60 and <70

4. 70 and <80

5. 80 and <90

A number is interesting if on adding the sum of the digits of the number and the product of the digits of
the number, the result is equal to the number. What fraction of numbers between 10 and 100 (both 10
and 100 included) is interesting?
1. 0.1
4. 0.22

73.

3. 24

p, q and r are three non-negative integers such that p + q + r = 10. The maximum value of pq + qr + pr+
pqr is
1. 40 and <50

72.

2. 25

How many whole numbers between 100 and 800 contain the digit 2?
1. 200

71.

5. T1, T4 and T5

In a square PQRS, A
and B are two points on PS and SR such that PA = 2AS, and RB = 2BS. If
PQ = 6, the area of the triangle ABQ is (is it a repeat)
1. 6

70.

4. T1, T3 and T4

Six playing cards are lying face down on a table, two of them are kings. Two cards are drawn at
random. Let a denote the probability that at least one of the cards drawn is a king, and b denote the
probability of not drawing a king. The ratio a/b is
1. 0.25 and <0.5

69.

3. T1, T2 and T4

The radius of a circle with centre O is 50 cm. A and C are two points on the circle, and B is a point
inside the circle. The length of AB is 6 cm, and the length of BC is 2 cm. The angle ABC is a right
angle. Find the square of the distance OB.
1. 26

68.

2. T1, T2 and T5

2. 0.11
5. None of the above

3. 0.16

70% of the students who joined XLRI last year play football, 75% play cricket, 80% play basketball
and 85% play carrom. The minimum percentage of students who play all four games is:
1. 5%
2. 10%
4. 20%
5. None of the above
p and q are positive numbers such that pq = qp, and q = 9p. The value of p is

3. 15%

22

www.hitbullseye.com

1.
75.

2. 7

2. 12.5 metres

3. 6

4. 5

5. 4

3. 14 metres

4. 15 metres

5. 16 metres

2. 6
5. None of the above

3. 7

2. 27.32 km

3. 28.30 km

4. 30 km

5. 36 km

Consider a function f(x) = x4+ x3 + x2+ x + 1, where x is a positive integer greater than 1. What will be
the remainder if f(x5) is divided by f(x)?
1. 1

80.

5. 3 9

In the country of Four, there are four cities, A, B, C and D. B is to the East of A, C is to the South of B,
D is to the West of C, and A is to the North of D. The Government of Four is planning to connect these
four cities by road such that it is possible for a person to go from a city to any of the other three cities.
At the same time, the Government wants to ensure that the total road length is minimum. The distances
between A to B, B to C, C to D and D to A are all equal to 10 km. What should be the total length of
the road?
1. 25.64 km

79.

4. 8 9

Consider the expression: (xxx)b = x3, where b is the base, and x is any digit of base b. Find the value of
b:
1. 5
4. 8

78.

3. 9 9

The central park of the city is 40 metres long and 30 metres wide. The mayor wants to construct two
roads of equal width in the park such that the roads intersect each other at right angles and the diagonals
of the park are also the diagonals of the small rectangle formed at the intersection of the two roads.
Further, the mayor wants that the area of the two roads to be equal to the remaining area of the park.
What should be the width of the roads?
1. 10 metres

77.

2. 6 9

Ram, Shyam and Hari went out for a 100 km journey. Ram and Hari started the journey in Ram's car at
the rate of 25 kmph, while Shyam walked at 5 kmph. After sometime, Hari got off and started walking
at the rate of 5kmph and Ram went back to pick up Shvam. All three reached the destination
simultaneously. The number of hours required for the trip was:
1. 8

76.

Bulls Eye

XAT 2013

2. 4

3. 5

4. a monomial in x 5. a polynomial in x

Please read the following sentences carefully:


I. 103 and 7 are the only prime factors of 1000027
II. 6 6! 7 7!
III. If I travel one half of my journey at an average speed of x km/h, it will be impossible for me to
attain an average speed of 2x km/h for the entire journey.
1. All the statements are correct
2. Only Statement II is correct
3. Only Statement III is correct
4. Both statements I and II are correct
5. Both statements I and HI are correct

81.

The figure below shows the graph of a function f(x). How many solutions does the equation f(f(x)) =
15 have?
1. 5
4. 8

2. 6
3. 7
5. Cannot be determined from the given graph.

Bulls Eye

XAT 2013

www.hitbullseye.com

23

DIRECTIONS for questions 82 to 84: Answer the questions based on the following information.
Given below are a few data points on the Indian economy from 2005 to 2010:

Indicator

Unit

2005

2006

GDP, current prices

Rs Billions

356.62.2 41159.73 47675.86 54470.27 60712.76 73555.34

GDP per capita, current


prices

Rs

32128.1

36553.93 41747.69 47038.23 51714.45 61784

Gross national savings


Percent of GDP

32.88

34.28

36.65

32.17

35.08

32.14

Inflation, average consumer Index


prices Index

115.67

122.92

130.75

141.67

157.08

175.92

Volume of imports of goods % change


and services

17.99

9.438

16.3

10.84

8.321

16.49

Volume of exports of goods % change


and services

18.88

13.83

17.13

10.63

0.813

21.86

Unemployment rate

9.2

8.9

7.8

7.2

6.8

7.32

Current account balance


Percent of GDP

-1.272

-1.024

-0.701

-2.475

-2.066

-3.268

* Per capita GDP is arrived by dividing GDP by population.

2007

2008

2009

2010

24

82.

www.hitbullseye.com

What is the ratio of the current account balance in 2010 to the current account balance in 2005?
1. 0.35

83.

2. 4.56

3. 5.01

4. 2.57

5. 5.30

Read the statements given below:


A. Exports were more than imports in 2006
B. Imports were more than exports in 2009
C. Exports increased at faster rate than imports during the period 2005 to 2010
D. Which of the above statements is necessarily true?
1. A and B

84.

Bulls Eye

XAT 2013

2. A, B and C

3. C only

4. B Only

5. A only

What was the approximate number of unemployed persons in 2006?


1. A. 100 million

2. 102 million

3. 98 pillion

4. 105 million

5. 104 million

DIRECTIONS for questions 85 to 88: Answer the questions based on the following information.
Data on an ongoing football league of a country is given below. 20 teams are playing in the league. The rules of
the league are as follows:
1. Each team plays all the other teams twice, once in its home ground, and once in the opponent's home ground.
These matches are known as the "Home" match and the "Away" match respectively.
2. A win results in 3 points, a draw in 1 point, and a loss in 0 point for the team.
3. The number of goals a team scores is termed as "Goals For" and the number of goals it concedes is termed as
"Goals Against". We get the "Goal Difference" by subtracting "Goals Against" from "Goals For".
The ranking of the teams is decided on the total points. If two teams are tied on their total points, the team
which has a higher Goal Difference gets the higher rank. If the tie cannot be resolved on Goal Difference, Goals
For is checked followed by Goals Against. If the tie persists, the teams are ranked in the ascending order of
their names.
Table 1 provides data on the current top 13 teams based on the overall situation, i.e., by taking into account both
home matches and away matches of each team.
Table 2 provides data on the current top 13 teams based on home matches only.
Chart 1 provides a plot of the goal difference of each of the 13 teams based on the overall situation.

OVERALL
Team M
W
D
CH
3
3
0
SW
3
2
1
WB
3
2
1
MC
3
2
1
MU
3
2
0
WH
3
2
0
EV
3
2
0
AS
3
1
2
WG
3
1
1
NC
3
1
1
FU
3
1
0
ST
3
0
3
SU
2
0
2

GF
8
10
6
8
6
4
4
2
4
3
7
3
2

Team
WB
WH
CH
MC
SW
NC
FU
EV
MU
TOT
NW
AS
ST

M
2
2
2
2
2
2
1
1
1
2
1
1

HOME
W
D
2
0
2
0
2
0
2
0
1
1
1
1
1
0
1
0
1
0
0
2
0
1
0
1
0
1

GF
5
4
6
6
5
3
5
1
3
2
1
0
0

GA
0
0
2
3
2
2
0
0
2
2
1
0
0

Bulls Eye

XAT 2013

www.hitbullseye.com

85.

Considering away matches only, which of the following teams is the second ranking team?
1. AS

86.

3. WG

4. SW

5. WB

2. 3

3. 4

4. 5

5. 6

Let us define a term pos as the difference between "home rank" and "away rank". Which of the
following has the maximum value of pos?
1. AS

88.

2. CH

Considering away matches only, the least number of teams with either 0 or 1 point is:
1. 2

87.

25

2. WB

3. WH

4. MC

5. SW

How many unique values of goal difference are there for away matches?
1. 5

2. 6

3. 6

4. 6 and 13

5. 6 and 14

26

89.

www.hitbullseye.com

Which is the fastest train between HHH and NNN?


1. 1111

90.

2. 2222

3. 4444

4. 777

5. 9999

4. 4444

5. 3333

Which is the fastest train between AAA and NNN ?


1. 8800

91.

Bulls Eye

XAT 2013

2. 8888

3. 6666

If you have to travel from AAA and reach HHH at around 9:00 AM, and then further travel to NNN at
around 6:00 PM, which is the best combination of trains for you?
1. 1111 and 8800

2. 1111 and 9900

3. 8888 and 7777

4. 8888 and 8800

5. 8888 AND 9900

Bulls Eye

XAT 2013

www.hitbullseye.com

Part A General Awareness


1.

Which of the following Indian companies feature in Fortune Five hundred list for 2012?
1. Reliance Industries, Tata Motors, Tata Steel, IOC
2. Tata Motors, SBI, CTS, Wipro, IOC
3. Tata Steel, IOD, Reliance Industries, Ranbaxy
4. IOC, reliance Industries, SBI, GAIL
5. IOC, Reliance Industries, Ranbaxy

2.

Which of these bodies decide the monetary policy in India?


1. Reserve Bank of India
3. Ministry of Commerce
5. Planning Commission

3.

2. Ministry of Finance
4. Ministry of Finance and Commerce

CPI and WPI as acronyms, refer to:


1. Political Parties
3. Price Indices
5. Unemployment indices

4.

2. Purchasing capacity indices


4. Poverty Indices

What is Davos famous for?


1. It is best known tourist place of Switzerland
3. EU financial headquarter
5. Best ski resorts in the world

5.

2. G8 members meet here, every year


4. World Economic Forum

Which of the followings are millennium development goals?


1. Poverty eradication, reducing child mortality, reducing trade barriers, combating AIDS
2. Poverty Reduction, environment sustainability, universal primary education, equal employment
3. Reduce unemployment; promote diversity, combating malaria, improving maternal health
4. Globalization, reduction in poverty, combating AIDS, universal primary education
5. Poverty eradication, improving maternal health, universal primary education, combating AIDS

6.

Which of countries in Europe were most hit by sovereign debt crisis?


1. Greece, Germany, Spain, Iceland
2. Portugal, Spain, Italy, Czech Republic
3. Greece, Spain, France, Switzerland
4. Spain, Greece, Iceland, Portugal
5. Germany, U.K, France, Greece

7.

Which of the following countries would account for maximum petroleum production of the world?
1. Saudi Arabia, Iran, Kazakhstan, US, Nigeria
2. Russia, U.S, Iran, China, Saudi Arab
3. Saudi Arabia, U.S, Venezuela, Iran, Iraq
4. Iran, Russia, Saudi Arabia, Nigeria, Libya E: Iran, Iraq, Saudi Arabia, Libya, Venezuela
5. Iran, Iraq, Saudi, Arbia, Libya, Venezuela

27

28

8.

Bulls Eye

XAT 2013

www.hitbullseye.com

Which of the following was not the reason for recent slide of Indian rupee?
1. Huge trade Deficit
3. Low Capital Inflow
5. High unemployment rate

9.

Which of followings combination belongs to Bretton Woods institute?


1. IMF, World Bank
3. World Bank, WTO, IBRD .
5. None of the above

10.

2. Low Growth and High Inflation


4. Burgeoning current account deficit

2. IMF, WTO, Doha Round


4. IMF, World Bank, UNESCO

Read the statements carefully and answer the questions that follow:
1. Earth Summit is held every year
2. Last earth summit took place in Rio Di Janeiro
3. Another name for earth summit is United Nation Conference on Environment and Development
Which of the above is the right combination?
1. 1 and 2

11.

2. 2 and 3

4. 1,2 and 3

5. 2 only

Which of the following countries meet more than one fourth of their energy requirement from nuclear"
power?
1. France, U.S, Russia, Germany
3. France, India, Ukraine, Germany
5. Japan, Germany, Korea, England

12.

3. 1 and 3

2. France, Ukraine, Sweden, Korea


4. Germany, Russia, Canada, Japan

Pick the right match from the following combinations


1. Shenzhen-China, NSE-India, SGX-Singapore, DAX-Germany
2. NASDAQ-U.S, MCX-India, SGX-Korea, FTSE-France
3. NYSE-U.S, BSE-India, FTSE-England, DAX-Denmark
4. NASDAQ-Canada, BSE-India, FTSE-France, DAX-Germany
5. Shenzhen-Singapore, Shanghai-China, FTSE-France, BSE-India

13.

Harish Hande who won^he Magsaysay Award in 2011 is famous for


1. RTE

14.

2. RTI

3. Lokpal bill

4. MRTPC

Indian government is planning to introduce GAAR. What is the purpose of GAAR?


1. Increase rural income
2. Increase agriculture income
3. Regulatory authority for managing agriculture resources
4. Regulate foreign investment in agriculture
5. Curtail tax avoidance

15.

5. SELCO

Who is the chairman of Indian planning commission?


1. Dr. M.S Ahluwalia
3. Dr. Man Mohan Singh
5. Dr. Subba Rao

2. Dr.C Rangarajan
4. Raminder Singh Gujral

Bulls Eye

XAT 2013

www.hitbullseye.com

16.

29

Given below are some well-known awards and their respective fields. Choose the correct combination:
1. Palme d'Or - Television; Pulitzer - Advertising; Grammy - Music; Booker - Literature
2. Pulitzer - Advertising; Grammy - Music; Cannes Lions - Advertising; Booker - Literature
3. Grammy - Music; Booker - Journalism; Emmy - Television; Palme d'Or - Movies
4. Cannes Lions - Movie, Grammy - Music, Pulitzer - Journalism, Emmy - Television
5. Palme d'Or - Movie; Pulitzer - Journalism; Cannes Lions - Advertising, Booker Literature

17.

A few combinations of names of companies and their products are given below. Choose the right
option:
1. Amazon - Kindle; Roxio - Angry Birds; Google - Ad Sense; Zynga - Nook
2. Amazon - Kindle; Nintendo - Wii; Rovio - Angry Birds; Zynga - CastleVille
3. Zynga - Angry Birds; Nintendo - Wii; Google - Ad Sense; Rovio - CastleVille
4. Amazon - Nook; Sony - Wii; Zynga - Castle Ville; Google - Ad Sense
5. Amazon - Nook; Nintendo - Wii; Rovio - Castle Ville; Roxio - Kindle

18.

Read the following statements carefully:


I. Excise duty is a tax on manufacture or production of goods and services ^L,
II. Not all kinds of excise duty is collected by the Central Government in India
III. Excise duty is an indirect tax
Which of the statements given above are correct?
1. Statements I and II
3. Statement I and III

19.

2. Jute
5. Leather

3. IT Services

Which of the following acids helps in human digestion?


1. Lactic

21.

5. None of the above

From which of the following items India export the most?


1. Textile goods
4. Gems and Jewellery

20.

2. Statements II and III


4. All of the above

2. Tartaric

3. Hydrochloric

4. Sulphuric

5. Acetic

Which of the following is the right match for Olympics medal winners from India?
1. Sushil Kumar-Haryana, Mary Kom-Assam, Saina Nehwal-Andhra Pradhesh, Vijay Kumar- Madhya
Pradesh
2. Sushil Kumar- Rajasthan, Mary Kom- Manipur, Vijay Kumar-Himachal Pradesh, Yogeshwar DuttHaryana
3. Yogeshwar Dutt: Haryana, Mary Kom-Manipur, Vijay Kumar-Himachal Pradesh, Gaurav NarangAndhra Pradesh
4. Yogeshwar dutt-Delhi, Saina Nehwal- Uttaranchal, Sushil Kumar- Haryana, Mary Kom-Manipur
5. Saina Nehwal-Uttaranchal, Gaurav Narang-Punjab, Sushil Kumar-Haryana, Mary Kom-Manipur

22.

Which of the following African countries underwent political turmoil in the last two years?
1. Tunisia, Egypt, Libya, Algeria, Morocco, Sudan
2. Tunisia, Yemen, Algeria, Libya, Morocco, Kenya
3. Egypt, Libya, Sudan, Mauritania, Nigeria, Malta
4. Mauritania, Egypt, Libya, Algeria, Sudan, Maldives
5. Egypt, Libya, Lebanon, Algeria, Sudan, Kenya

30

23.

Bulls Eye

XAT 2013

www.hitbullseye.com

Which of the following education institutions was Mr. Rajat Gupta, the former McKinsey chief
associated with?
1. XLRI Jamshedpur
2. Indian Institute of Management Calcutta
3. Indian Institute of Management Bangalore
4. Indian Institute of Management Lucknow
5. Indian School of Business, Hyderabad

24.

Which country was the highest producer of cars in the world in 2011?
1. U.S.A

25.

3. Korea

4. Japan

5. China

What is the approximate share of manufacturing sector in Indian GDP in 2011 -12?
1. 22%

26.

2. Germany

2. 24%

3. 31%

4. 15%

5. 10%

Which of the following combination of scholars have won the Nobel Prize in Economics?
1. Amartya Sen, Joseph Stiglitz, Oliver Wiiliamson
2. Mehboob Haque, Amartya Sen, Robert McNamara
3. Amartya Sen, Mohammad Yunus, Ronald Coase
4. Mohammad Yunus, Paul Krugman, Oliver Williamson
5. Amartya Sen, Elvin Roth, Mohammad Yunus

27.

For which of the following reasons was Maruti in the news in July, 2012?
1. Producing most fuel efficient cars
2. Transferring its production to Gujarat
3. Industrial violence
4. Disrupting dairy movement of vehicles in New Delhi
5. In Jury, it became the highest producer of cars in the country

28.

What is meaning of one unit carbon permit?


1. One cubic meter of carbon dioxide
2. One quintal of carbon dioxide
3. One metric ton of carbon dioxide
4. One thousand metric tons of carbon dioxide
5. One million ounce of carbon dioxide

29.

Who is author of "Imagining India"?


1. Khushwant Singh
4. Chetan Bhagat

30.

2. Nandan Nilekani
5. APJ Abdul Kalaam Azad

V Kurien was associated with:


1. Operation Flood
3. Operation Blue Star
5. White Operation

2. Operation Milk
4. Operation X

3. Gurcharan Das

Bulls Eye

XAT 2013

www.hitbullseye.com

31

XAT 2013 Key


ANSWER KEY
#

DISCLAIMER: The below given answer key & Explanations are based on Personal opinion of Bull Eye Expert Faculty.

1.
2.
3.
4.
5.
6.
7.
8.
9.
10.
11.
12.
13.
14.
15.
16.
17.
18.
19.
20.
21.
22.
23.
24.
25.
26.
27.
28.
29.
30.

1
2
3
1
4
2
2
3
5
2
2
5
1
4
4
5
2
3
3
4
3
2
4
3
5
4
4
5
4
1

31.
32.
33.
34.
35.
36.
37.
38.
39.
40.
41.
42.
43.
44.
45.
46.
47.
48.
49.
50.
51.
52.
53.
54.
55.
56.
57.
58.
59.
60.

2
3
5
2
2
3
1
3
4
2
1
3
3
3
4
1
5
2
5
3
2
5
4
2
5
1
3
5
4
1

61.
62.
63.
64.
65.
66.
67.
68.
69.
70.
71.
72.
73.
74.
75.
76.
77.
78.
79.
80.
81.
82.
83.
84.
85.
86.
87.
88.
89.
90.
91.

5
4
5
2
2
5
1
5
3
2
3
5
2
4
1
1
5
3
5
3
3
5
3
1
4
4
1
2
4
1
4

PART-A
General Awareness
1.
1
2.
1
3.
3
4.
4
5.
5
6.
4
7.
2
8.
5
9.
1
10.
2
11.
2
12.
1
13.
5
14.
5
15.
3
16.
5
17.
2
18.
4
19.
4
20.
3
21.
3
22.
1
23.
5
24.
5
25.
4
26.
1
27.
3
28.
3
29.
2
30.
1

XAT 2013
EXPLANATIONS
1.
2.
3.
4.
5.
6.

7.

8.

The first statement will lead to second statement as vagueness in the laws will lead to manipulation of laws by Bureaucratic
chain.
Internet revolution is making transfer of knowledge cheaper and public money can be spent on it therefore this is facilitating
condition for question statement 1.
Both the statements talks about con side of capitalism and how government is also involved in all this.
We use a when we talk about a noun for first time or to represent one. The when we refer to the noun already mentioned
or to represent anything specific.
We use a when we talk about a noun for first time and the when we refer to the noun already mentioned.
We use the with ground because it is touching the ground.
Downstairs and down are very close options but because choice with downstairs does not contain other relevant fillers. That
is why we rule it out We always use dared to Little refers to small amount and a little refers to almost no. Another hint is
as usual.
Sentence 2 is the opening sentence .It is introducing the paragraph telling us the thoughts of author ,the situation and his
location
Sentence 1 continues sentence 2 and tells in detail the beginning of arrival of enemies
In sentence 3, the keyword is the man which is referring to the man with lantern in previous sentence..so it follows
Sentence 4 indicates the conclusion of authors thoughts .the keyword is his
The story opens with sounds coming from different directions because of the fighting. After this sentence 5 will come as

32

9.

10.
11.
12.

13.
14.
15.
16.

17.
18.
19.
20.
21.
22.
23.
24.
25.
26.
27.
28.
29.
30.
31.
32.
33.
34.
35.
36.

37.
38.

39.

XAT 2013

Bulls Eye
www.hitbullseye.com

almost the same time refers to time mentioned in sentence 2.That was plainly last signal of the danger refers to incident
mentioned in sentence 3.The concluding sentence will be 1, the hint is finally.
4 is the opening statement. It is independent and introduces the topic of development through accumulation. After this 1will
come as the keyword is they which is referring to the historians introduced in opening statement. After this 3 will come as
keyword is Questions which are given in the earlier statement. 2 will finally conclude as the keyword is historians which
refers to the few mentioned in previous statement
Requires and demands serve similar purpose in a sentence, it gets redundant
Because there is a comparison does not appear to be same means to whom it does not appear? I.e. to a person standing still
as to a person who travels. In other words it is case of parallelism.
To is used after extent :- option b, c will be rejected
, Savings is a plural word therefore have is used and been is required as the agent is there to bring the changes i.e...
by the instability in the futures market.
Is similar to that of bomb blast which Fermi conducted through confetti as one can find the correlation between distance
travelled by the exploded parts and magnitude of the force that caused the explosion.
After reading the passage it is clear that Fermi was able to know the ranges of various values through estimation and this
corresponds to answer option.
Refer last line last Para where it is mentioned that Fermi was famous for teaching his student the art of approximation of
fanciful sounding quantity.
It is not given that if there is any correlation of making the Atomic bomb and Fermi getting the noble prize. It is not given in
the passage that what his students use to consider him. There are other methods/units to develop/ measure the impact of
atomic bomb explosion.
Refer lines 1-2 last words future existences which means re birth or next birth in any form as it is given in those very
bodies.
We may or may not believe in long term orientation knowing the importance of Karma therefore Conclusion MAY follow
assertion
Refer lines 5-6 of the passage.
Refer to the line .second, individuals with long term orientation..
As we are not given what were the expectations of the manager from the quality of the food so it is not possible for us
comment on his disconfirmation sensitivity.
The line "Soros distrust .economic departments" suggests the idea that neoclassical economics does not explain
the concept of free-market system.
In 5th paragraph, the statement "It would..laws of economics" supports the fact that laws of economics are not
capable of explaining the nature of human in complete sense.
The 3rd paragraph supports the belief of the author that state can intervene in various aspects of human life provided it does
not remain static.
None of the statements has been mentioned in regards to the critical rationalism.
Bythe word 'determinstic' in statement "Nor do.literally", the author means that cause(s) of an effect can always be
ascertained as
In first paragraph, it is clearly stated that one can at times deduce a universal scientific belief to be false however one can
never induce the validity of same.
In 2nd paragraph, it is mentioned that falsification is no more ..scientific beliefs.Thus supporting the statement
"this asymmetry it to be".
The significance of falsification is clearly illustrated in last paragraph with the help of statement "Falsifyingscientific
belief".
As in the concluding paragraph the author is emphasizing the fact that falsification of claims provides us with appropriate
reason for rejecting a scientifc belief.
Statement 1 &statement 2 were actually the reasons for fight against corruption. Media swung into action after shambhu
decided to sit on for a hunger strike. Statement 4 is not highlighting the category of people whose incomes are rising.
It could be one of the reasons for the public not showing support.
Whether the scheme was beneficial for the public or not but if it based on unethical principles then no logical justification
can justify the goodness of the scheme.
There cannot be smoke without fire as stated by the opposition party members so it is highly likely that Mr. swamy will
also criticize the ruling party.
As per the conditions/ situation given Mr. swamy is likely to pick only the minister because media and the opposition party
were able to point out the flaws in policies pertaining to resources allocation to the private companies.
The assumption made by the author is that there is strong and positive correlation between the increase in number of
customers catered by the firm and increase in prices of the share of the firm. If customer base is increasing by other means
then the share price would increase as a result.
If after the surgery the patient suffers from any major side effect then the popular and prescribed treatment is likely to lose its
sheen.
In order to find the profit /loss, we require the information about initial costs and the distribution of costs. As from statement
I, we can find the distribution of sales of all the three retail formats. Now from statement II, the information about the initial
costs is given and from the given tables, we already have the distribution of additional costs as well as the ratio of units
consumed. So on combining both the statements, we can easily find the profit/loss of different retail formats.
For Online :

Bulls Eye
www.hitbullseye.com

Sales = 50% of 60000 = 30000 ( as given above)


Initial costs = 50% of 39000 = 19500 (as given above)
Contribution = 30000-19500 = 10500
Additional costs :
R = 5500

200
50
50
50
= 4000, Y = 3100
= 1550, G = 4800
= 3000, V = 2400
=1500
275
100
80
80

Total = 10050
Therefore, Profit = 10500 10050 = 450.
40.

For Online :
Sales = 50% of 60000 = 30000 ( as given above)
Initial costs = 50% of 39000 = 19500 (as given above)
Contribution = 30000-19500 = 10500
Additional costs :
R = 5500

200
50
50
50
= 4000, Y = 3100
= 1550, G = 4800
= 3000, V = 2400
=1500
275
100
80
80

Total = 10050
Therefore, Profit = 10500 10050 = 450
For Supermarket :
Sales = 1/3 of 30000 = 10000 ( as given above)
Initial costs = 1/3 of 19500 = 6500 (as given above)
Contribution = 10000-6500 = 3500
Additional costs :
R = 5500

60
20
21
21
= 1200, Y = 3100
= 620, G = 4800
= 1260, V = 2400
=630
275
100
80
80

Total = 3710
Therefore, Loss = 3710 3500 = 210
For Bookstore :
Sales = 2/3 of 30000 = 20000 ( as given above)
Initial costs = 2/3 of 19500 = 13000 (as given above)
Contribution = 20000-13000 = 7000
Additional costs :
R = 5500

10
30
9
9
= 200, Y = 3100
= 930, G = 4800
= 540, V = 2400
=270
275
100
80
80

Total = 1940
Therefore, Profit = 7000 1940 = 5060

41.

So clearly Supermarket is making least profit, as it in loss.


For Online :
Sales = 50% of 60000 = 30000 ( as given above)
Initial costs = 50% of 39000 = 19500 (as given above)
Contribution = 30000-19500 = 10500
Additional costs :
R = 5500

200
50
50
50
= 4000, Y = 3100
= 1550, G = 4800
= 3000, V = 2400
=1500
275
100
80
80

Total = 10050
Therefore, Profit = 10500 10050 = 450
For Supermarket :
Sales = 1/3 of 30000 = 10000 ( as given above)
Initial costs = 1/3 of 19500 = 6500 (as given above)
Contribution = 10000-6500 = 3500
Additional costs :
R = 5500

60
20
21
21
= 1200, Y = 3100
= 620, G = 4800
= 1260, V = 2400
=630
275
100
80
80

Total = 3710
Therefore, Loss = 3710 3500 = 210
For Bookstore :

XAT 2013

33

34

Bulls Eye

XAT 2013

www.hitbullseye.com

Sales = 2/3 of 30000 = 20000 ( as given above)


Initial costs = 2/3 of 19500 = 13000 (as given above)
Contribution = 20000-13000 = 7000
Additional costs :
R = 5500

42.

43.
44.

45.

46.
47.
48.
49.
50.
51.
52.
53.
54.
55.

56.

10
30
9
9
= 200, Y = 3100
= 930, G = 4800
= 540, V = 2400
=270
275
100
80
80

Total = 1940
Therefore, Profit = 7000 1940 = 5060
Clearly, Bookstore is making the highest profit
Since students sitting next to each other have marked same answers, thus we get 2 possibilities i.e.
1. They have marked correct answers
2. They have marked incorrect answers
Now, in case of correct answers, teacher cannot detect cheating, however, students would have cheated definitely if most of
the closely seated students have same incorrect answers. Thus correct option is 3.
Here I, II, II, IV statements are required to know the strength of the class i.e. Strength order is A>B>C>D, also Strength of C
is greater than 2 from statement VI. From statement V, ABCD = 10x, which implies that one of A, B, C, D is 5. Taking
the value of x from 1-11, we either get the less factors or the number of students come out to be same. When x = 12, we get
the factors to be 2, 3, 4, 5. So these numbers satisfy the given statements and give us the strength of all the sections. Thus we
require only first 6 statements to find the strength of the sections.
From 1st guess, the ranking is A, B, C, D, E but this ranking is wrong which means A doesnt precede B in correct ranking, B
doesnt precede C, C doesnt precede D and D doesnt precede E
From 2nd guess, the ranking is D, A, E, C, B but in this ranking 2 positions are correct and 2 students in ranking followed
their immediate predecessors
Now if we consider option 1 to be correct, it will imply that both A and E are at incorrect positions and out of B, C and D one
more should be incorrect. Secondly, E will either come at position I or IV, because if its on II, it will make 3 positions in 2nd
guess correct, if its on V it will negate the incorrectness of 1st guess.
So we get two possible rankings i.e.
E, D, A, C, B and D, C, A, E, B
In first case, we have two predecessors at their correct positions according to 2nd guess but in 2nd case, only one predecessor
is at their correct position.
Thus option 1 is correct.
By demoting or sacking kale will not solve the problem as person kept in place of kale may have same or different set of
problems. Rest of options either will solve the problem or at least reduce the impact of the problem.
It is clear if companys responsibility is restricted to workplace only then Marathe should not tell Kale to visit family
counselor.
If there is correlation between problems at home and performance at work then there is no point in removing/sacking kale on
behavioral grounds as he is also going through same.
Had he been appreciated by his bosses for his achievement, he would have involved himself more at the workplace.
The simplest will be to give him more challenging assignment and also train him to work under stress. By doing this
Vardarajan will be in better position to handle stress and performing duties at the same time.
It will be better to make the assessment of performances transparent so as to address Vardarajans grievances thereby making
systemic changes in a positive direction.
Verify the key facts related to the case and take the corrective measures.
Ask for the verified and correct data.
There cannot be better option than clearing things from the college authorities and if they direct me to person concerned, it
would be their call/responsibility as college authorities will be better position to deal with the problem in hand.
There is no point in running away from the responsibility because of a call from unknown caller .We are also not sure of
concern shown by the caller so it would be a right decision to continue with the work and get the team to investigate about
the caller/call etc.
Let x, y and z be the no of pages read by Albela, Bob and Chulbul respectively. Also time taken by each of them is equal i.e.
2x = 3y = 4z. Since total pages are 78, which means
x + y + z = 78 . Putting the values of x and z in terms of y, we get

57.

58.
59.
60.

3y
3y
y
78 y = 24.
2
4

For first 2 km, the fare is 2 10 = Rs 20, for next 8 kms, the fare is 8 5 = Rs 40, for next 4kms, the fare is 4 8 = Rs 32.
So total fare = 20 + 40 + 32 = Rs 92
So amount left is Rs 10 and the remaining distance to be covered by Bullock = 10/2 = 5km
Thus total distance = 14 + 5 = 19km
Putting the value of all the variables a, b, c, d, e as 1 and solving we get 35 = 243.
Since both ways Auto takes 30 minutes so one way Auto is 15 minutes. Now Walking and Auto takes 90 minutes, which
implies 90 15 = 75 minutes is for Walking. So, both ways Walking would take 75 2 = 150 minutes.
Let point A be the first edge where the fish is released and B be the point where they divert after travelling for 300 mtrs. and
C be the point where they reach another edge after travelling for 400 mtrs. Now for finding the area of the circular pool we
assume the angle between AB & BC to be 90. Then AC must be 500 mtrs. and hence the diameter of the pool which gives

Bulls Eye

XAT 2013

www.hitbullseye.com

35

the radius to be = 250 mtrs. Now, are will be r2 = (250)2 = 62500.

B
400

300

A
500

61.

Here P = 15/2 = 7.5 lakhs and Simple interest = 21 7.5 = 13.5 lakhs

I 100 13.5 100

20
PT
7.5 9
I 100 13.5 100

30
Similarly, for older son, rate =
PT
7.5 6
Therefore, for younger son, rate =

62.

63.
64.
65.
66.

As the sum of all six digits is 90 and the median is 18. Therefore, the first two numbers will be 1 and the next two will be 17
and 19 ( as median is 18 ). No the sum of remaining two digits is 52, out of which the largest possible value of number can
be 32

Arun can do a maximum of 3 tasks. So as per priority and no of days required, he should first do T1, and then T2 as it has
higher priority and last must be T5, not T4 as no of days required in T4 are more, thus option 2
Arun can do a maximum of 3 tasks. So as per priority and no of days required, he should first do T1, next he cannot take T2
as it has highest number of background research days so he will take next two tasks T4 and T5 as per their priority number,
thus option 5

67.
68.

Probability (a) = 1 king & 1 not king OR Both kings =

Probability (b) = Not drawing a king =


69.

C2

C2

C1 4 C1 2 C 2
6

C2

10 2
.
15 3

6 2
. Now ratio of a:b = 2/3 : 2/5 = 5/3, which is greater than 1.25.
15 5

In the figure PQRS, PQ = 6, PA = 4 so QA = 213(By Pythagoras) and AB = 22. Now for area of ABQ we need the
height which will be
(213)2 = x2 + (22)2. Solving x = 50. Now area = Base Height;
22 50 = 10.
6
Q
P

4
A
2
S
70.

Since in every interval of 100 numbers we have 19 numbers which have a particular digit used so from 100 to 800 we have 6
(except from 200 to 299 which has all 100 numbers having 2 in them) groups of 19 each. So, the total numbers will be 19 6
= 114+100 =214.
B

400

300

A
500

71.
73.

Since we need to maximize the relation so we try to equalize the variables p, q & r and also their sum is . This happens at
3+3+4=10. Now putting values we get 9+12+12+36=69 which lies in the range 60 to 70.
The equation is 10x+y = (x + y)(xy) which results in y4+100-16y20. Now putting y2=t we get t2-16t+1000, which does not

36

74.
75.
76.

77.

78.

79.

80.

Bulls Eye

XAT 2013

www.hitbullseye.com

have any real roots. So no such number exists.


Totaling the percentage of players who do not play each of the games we get 30+25+20+15=90. Now the minimum
percentage of students who must be playing all the four games will be 100-90=10.
Pq = qp and q = 9p, putting the value of q in first equation, we get p9p=(9p)p p9p = 9p.pp p8p = 9p Taking 1/8p root on
both sides, we get p = 91/8
Since the options are whole numbers lets assume that Hari got off the car after 3 hrs. (as 1, 2 and 4 hrs. is not possible). Now
that means Ram & Hari must have travelled for 25 3 = 75 kms., so 25 kms. still remain to be covered which Hari will do at
5 km/h in 25/5=5 hrs. Now in those 5 hrs. Ram can reach back for Shyam and take him to the destination in those 5 hrs.
Let x be the width of the roads. As length and breadth are 40m and 30m respectively
Now as per question, 40x + 30x x2 = 600 70x x2 = 600 x2 70x + 600 = 0
x = 60 or 10. As 60 is not possible, therefore width of the road is 10m
The expression (xxx)b = x3, when expanded for base b we get xb2 + xb + x = x3.
Taking x common we get b2 + b + 1 = x2. Now putting values for b none of the options satisfy to be a perfect square(x2). By
hit and trial value of b comes to be 18.
The road connecting all the cities should be the diagonals from A to C and from B to D. Since the distances between
A,B,C&D are 10 each so the diagonals must be 102 each(By Pythagoras). So, the total length should be 2(102) = 202 =
20 1.414 = 28.3 approx.

x 25 1
x5 1
and f(x5) = x20+x15+x10+x5+1 =
x 1
x5 1
x 25 1
Dividing f(x5) by f(x), we get
, which is a polynomial in x
x 1

f(x) = x4+x3+x2+x+1 =

81.
82.

Statement I is wrong as 19 is also one of the prime factor, Statement II is also wrong while Statement III is true
Current balance in 2010 =

73555.34

3.268
100
= - 2403.78billion Rs.

1.272
Current balance in 2005 = 35662.2
100
= - 453.62

2403.78
Required Ratio =
5.30
453.62
83.

1st statement is false as export were less than imports in 2006.


2nd is also false as Export were more than imports in 2009.
3rd is true as %change in export is more while in import %change is negative.

84.

Population in 2006 =

41159.73 1012
36553.93
= 1.12 1012
= 112 1010
= 112 million.

Unemployed in 2006 =

112

8.9
99.6million
100

Team

Won

Draw

Lost

Points

GF

GA

Goal Diff
=GF GA

Rank

100 million
85-88. The following table can be formed for the AWAY matches:

CH

SW

2
8

WB

MC

MU

WH

-3

10

EV

AS

Bulls Eye

XAT 2013

www.hitbullseye.com

WG

85.
86.
87.

88.
89.
90.
91.

Information cannot be determined

NC

-2

FU

-4

11

ST

SU

37

Information cannot be determined

According to the above table, it can be seen that based upon AWAY matches ranking, team SW will be the second ranking
team.
Atleast these teams have 0 or 1 points WB, MC, WH, NC & FU. These are 5 in number.
The pos table for the answer options can be formed as follows:
Team
Home Rank
Away Rank
pos
AS

12

11

WB

-7

WH

10

-8

MC

-3

SW

Thus, maximum value of pos is for the team AS i.e. 11.


These are different values possible for Goal differences: 2, 5, 0, - 3, - 2 and 4. These are 6 different values.
Fastest train between HHH and NNN among given options is 7777 as it is the only train that is taking less than two hours
while rest are taking more than two hours.
1st two options get rejected as 1111 reach HHH after 10AM. Third option gets rejected as 7777 starts around 8PM instead of
6PM. Between 4th and 5th options, 4th option is better as 8800 starts travelling exactly at 6PM.
Fastest train between AAA and NNN among given trains is 8800 as it is the only train that is taking less than 3 hours while
rest of the trains take more than 3 hours.

Bulls Eye

XAT PAPER 2014


1

www.hitbullseye.com

PART I
Section A: Verbal Ability
DIRECTIONS for questions 1-2: Read the following sentences and choose the option that best -arranges
them in a logical order.
1.

Choose the best option:


A. The mechanism of electroweak symmetry breaking is one of the most important issues in the
present Particle Physics.
B. They are required to give masses for all quarks and leptons and to guarantee the absence of the
gauge anomaly.
C. In the standard electroweak model a fundamental Higgs doublet is introduced to cause the
spontaneous symmetry breaking.
D. Supersymmetry (SUSY), eliminating all quadratic divergences, may provide a better
theoretical basis to describe a fundamental Higgs boson with a relatively small mass to a high
energy cutoff scale, say the Planck scale for example.
E. In the minimal SUSY extension of the standard electroweak model the Higgs sector consists
of two chiral superfields of Higgs doublets (HI and H2 with opposite hypercharges).
1. A,C,D,E,B

2.

2. A,D,C,E,B

3. C,D,B,E,A

4. A,E,B,C,D

5. E,D,B,C,A

Choose the best option:


1. Shakespeare did not personally prepare his plays for publication, and no official collection of
them appeared until after his death.
2. Some were probably based on actors' memories of the plays.
3. Many of these quartos are quite unreliable.
4. A collection of his sonnets, considered by critics to be among the best ever written in English,
appeared in 1609.
5. Marry individual plays were published during his lifetime in unauthorized editions known as
quartos.
1. A,B,C,E,D

2. A,B,D,E,C

3. C,A,D,E,B

4. C,E,A,B,D

5. A,D,E,C,B

DIRECTIONS for questions 3-8: Fill in the blanks with the most appropriate option that follows:
3.

The defense proposes to show that the incident that the prosecution so ______rejects as
____indeed take place.
1. blithely, undesirable
3. vehemently, factual
5. convincingly, inevitable

4.

2. cavalierly, apocryphal
4. persuasively, pointless

Not just the absence of __________, but also the presence of ____________and honesty is required to
bind up the nation's wound.
1. retribution, camaraderie
4. love, austerity

5.

did

2. guilt, religion
5. recrimination, insight

3. crime, religion

In many cases in physics, one has to deal simultaneously with collective and single-particle
excitations of the system. The collective excitations are usually bosonic in nature while the
single-particle excitations are often fermionic. One is therefore led to consider a system which
includes bosons and fermions. Hence,

Bulls Eye

XAT PAPER 2014


2

www.hitbullseye.com

Which of the following options is most likely to follow the paragraph given above?
1. in this book, we discuss application of bosonic particles and their consequences on physics.
2. in this book, we discuss applications of a general algebraic theory of mixed Bose- Fermi
systems to atomic nuclei.
3. in this book, we discuss excitations of collective and single particles separately.
4. in this book, we prove how collective excitations are bosonic and singular excitation are
fermionic.
5. in this book, we prove that collective and singular excitation cannot exist together.
6.

Peter has suggested to me that the _______of highly systematic and _________planning techniques
may have led to a substantial _________in firms' notions of what is likely to happen in the future, and
thus to a _________in the incidence of mistakes, especially on the part of the
modern
_______corporations.
1. progressive development, exact, decline, decrease, tiny
2. progress deterioration, exact, increase, decrease, tiny
3. progressive development, precise, improvement, reduction, large
4. progressive development, precise, increase, decrease, large
5. gradual growth, exact, increase, decline, large

7.

Clinical practitioners ________integrated mindfulness _____treatment of ______host of emotional


and behavioral disorders, ________borderline personality disorder, major depression, chronic pain, or
eating disorders. Number of such practitioners _________increased substantially
1. have, in the, a, such as, has
3. were, for, a, like, has
5. could, in the, the, such as, have

8.

Ontologies are ______equated with taxonomic hierarchies of classes, class definitions, and
_________subsumption relation, __________ontologies need not be limited to forms.
1. often, a, but, such
3. frequently, a, yet, such
5. often, the, but, these

9.

2. have, in the, the, like, have


4. have, for, a, like, has

2. often, the, yet, such


4. often, a, yet, these

The MBA (1) is hardly a prerequisite for success, but it (2) certainly helps (3), and it has been
getting more important (4) in recent years. Most (5) MBA programs equip their graduates to
understand how (6) to deal with many of the important questions that their organizations will
need to tackle (7) over time, and (8) that they will face in their careers.
The above italicized numbered words will be correctly represented by the following parts of speech:
1. 1-Noun, 2-Pronoun, 3-Adverb, 4-Adjective, 5-Adjective, 8-Conjunction
2. 1-Noun, 3-Adjective, 4-Adjective, 6-Adverb, 7-Adverb, 8-Article
3. 1-Noun, 2-Pronoun, 4-Adjective, 7-Adverb, 8-Conjunction
4. 2-Pronoun, 3-Verb, 4-Adverb, 6-Adverb, 8-Article
5. 2-Pronoun, 3-Verb, 4-Adjective, 7-Verb, 8-Conjunction

Bulls Eye
www.hitbullseye.com

XAT PAPER 2014


3

DIRECTIONS for the question 10: Read the following paragraph carefully and answer the question that
follows:
It is one week since Uttarakhand's worst disaster in living memory. Flash floods resulting from extremely
intense rainfall swept away mountainsides, villages and towns, thousands of people, animals, agricultural
fields, irrigation canals, domestic water sources, dams, roads, bridges, and buildings anything that stood
in the way.
A week later, media attention remains riveted on the efforts to rescue tens of thousands of pilgrims and
tourists visiting the shrines in the uppermost reaches of Uttarakhand's sacred rivers. But the deluge spread far
beyond the Char Dhams Yamunotri, Gangotri,^ Kedarnath and Badrinath to cover the entire state. The
catchments of many smaller rivers also witnessed flash floods but the media has yet to report on the
destruction there. Eyewitness accounts being gathered by official agencies and voluntary organizations have
reported devastation from more than 200 villages so far and more affected villages are being reported every
day.
10.

Which of the following would the author agree the most with?
1. Char Dhams were most affected by Uttarakhand disaster.
2. Entire catchment of rivers flowing in Uttarakhand was affected.
3. Media attention was on Char Dhams but the entire catchment area of rivers flowing in Uttarakhand
was affected.
4. Media cannot be trusted as it focuses only on important places and events.
5. Voluntary organizations are better than media in reaching out to the affected people.

11.

Which of the following is the correct form of expression for the underlined part of the sentence
below?
Patna is not only the capital of Bihar, but it is also one of the oldest cities in the world and the largest
city in the state.
1. capital of Bihar, but it is also one of the oldest
2. capital of Bihar, nevertheless also one of the oldest
3. Bihar's capital city, and it is also one of the older
4. Bihar's capital, but it is also one of the older
5. capital of Bihar, but it is also one of the older

DIRECTIONS for questions 12 - 13: Read the definitions below and select the best match between the
numbered sentences and the definitions.
Premise: A proposition from which another statement is inferred or follows a conclusion.
Assumption: Something, which is accepted as true.
Fact: Something, which can be checked.
Reason: A cause, explanation or justification for an action or event.
Conclusion: An end, finish or summarization of process or argument.
Proposition: A statement that expresses judgment or opinion.
Question: A sentence worded or expressed so as to elicit opinion.
Inductive Inference: An end, finish or summarization reached for "the whole", based on "a particular" real
incidence.
Deductive Inference: An end, finish or summarization reached based on the combining and recombining
two or more than two assumptions.

XAT PAPER 2014


4

12.

Bulls Eye
www.hitbullseye.com

When you look at the people who make fundamental, revolutionary breakthroughs in any
field, you keep noticing over and over again a high preponderance of them have some sort of
disability when they were younger, whether it was a physical disability or mental disability,
which leads to lower expectations from others, whom they always wanted to prove wrong (1).
And what does it do (2)? What does that do to you, when you try to prove someone wrong (3)?
You increase your engagement in something because you want to fight against those
expectations (4). So it seems like it actually can be a gift having what we label as a disability, or
disorder, and cause people to overcompensate and engage in things in other ways (5). A research
study shows that higher number of people with dyslexia become social entrepreneurs because they
over-compensate their disability through nonverbal communication, initiative and grit (6). And this
overcompensation leads to greatness (7).
The best match would be:
1. 1-Assumption, 2-Question, 3-Reason, 4-Fact
2. 1-Assumption, 2-Question, 4-Reason, 7-Conclusion
3. 1-Fact, 2-Question, 4-Reason, 6-Reason
4. 1-Fact, 3-QuestioB, 4-Reason, 7-Inductive Inference '
5. 3-Question, 4-Reason, 5-Fact, 6-Proposition

13.

The fatal consequences of having a routine mid-day meal for at least twenty two children in Bihar's
Saran district expose the chronic neglect of school education in a large part of India (1). That
governments cannot find a small piece of land for a school and are unable to store food materials
without the risk of contamination is a telling commentary on their commitment to universal primary
education (2). The Bihar horror clearly points to the absence of strong normative procedures for the
provision of infrastructure, even for a new school (3). The best match would be:
1. 1-Fact, 2-Reason, 3-Deductive Inference
2. 1-Reason, 2-Reason, 3-Inductive Inference
3. 1-Inductive Inference, 2-Fact, 3-Inductive Inference
4. 1-Inductive Inference, 2- Inductive Inference, 3-Inductive Inference
5. 1-Reason, 2-Reason, 3-Reason

DIRECTIONS for questions 14 - 16: Analyze the following passage and provide appropriate answers
for the question that follows.
I heartily accept the motto, "That government is best which governs least"; and I should like to see it acted up
to more rapidly and systematically. Carried out, it finally amounts to this, which also I believe--"That
government is best which governs not at all"; and when men are prepared for it, that will be the kind of
government which they will have. Government is at best but an expedient; but most governments are usually,
and all governments are sometimes, inexpedient. The objections which have been brought against a standing
army, and they are many and weighty, and deserve to prevail, may also at last be brought against a standing
government. The standing army is only an arm of the standing government. The government itself, which is
only the mode which the people have chosen to execute their will, is equally liable to be abused and perverted
before the people can act through it.
After all, the practical reason why, when the power is once in the hands of the people, a majority are
permitted, and for a long period continue, to rule is not because they are most likely to be in the right, nor
because this seems fairest to the minority, but because they are physically the strongest. But a government in
which the majority rules in all cases cannot be based on justice, even as far as men understand it. Can there
not be a government in which the majorities do not virtually decide right and wrong, but conscience?~in
which majorities decide only those questions to which the rule of expediency is applicable? Must the citizen
ever for a moment, or in the least degree, resign his conscience to the legislator? Why has every man a
conscience then? I think that we should be men first, and subjects afterward. It is not desirable to cultivate a

Bulls Eye

XAT PAPER 2014


5

www.hitbullseye.com

respect for the law, so much as for the right. The only obligation which I have a right to assume is to do at any
time what I think right. It is truly enough said that a corporation has no conscience; but a corporation of
conscientious men is a corporation with a conscience. Law never made men a whit more just; and, by means
of their respect for it, even the well-disposed are daily made the agents on injustice.
14.

According to the author of the paragraph, army is_________?


1. required

15.

2. fallible

4. necessary evil

5. not required

In general, when would government of majority be good for minorities?


1. when it is fair.
3. when it abides by the law.
5. such a government cannot exist.

16.

3. must

2. when it is right.
4. when it is conscientious.

Which of the following statements would the author agree the most with?
1. Men are bigger than the governments.
2. Business houses are best for the growth of a society.
3. Governments and armies are not required.
4. Concept of nations is redundant.
5. Democracy is best for citizens.

DIRECTIONS for questions 17 - 19: Analyze the following passage and provide appropriate answers
for the question that follows.
The assumption of rationality puts an economist in a position to "explain" some features of market behavior,
such as the dispersion of prices of psychophysically identical goods such as beer according to the amount
spent on advertising them (no doubt, the fact that most beer is bought by individuals rather than as a raw
material by firms, which could be expected to be more rational than individuals, is part of the explanation).
Clearly, something is wrong somewhere with the usual model of a competitive market with perfect
information, for the virtually content-less advertising cannot be considered as increasing the utility of beer in
an obvious way. But if one can keep the assumption of rational actors, one need not get into the intellectual
swamp of sentiment nor of preferences that depend on price. If one agrees, for example, that consumers use
advertising as an index of the effort a producer will put into protecting its reputation and so as a predictor of
quality control efforts, one can combine it with the standard mechanism and derive testable consequences
from it.
But why, logically speaking, does it not matter that any of us, with a few years' training, could disprove the
assumptions? It is for the same reason that the statistical mechanics of gases is not undermined when
Rutherford teaches a lot of only moderately bright physicists to use X-ray diffraction to disprove the
assumption that molecules are little hard elastic balls. The point is, departures that Rutherford teaches us to
find from the mechanism built into statistical mechanics are small and hardly ever systematic at the level of
gases. Ignorance and error about the quality of beer is also, unlikely to be systematic at the level of the
consumers' beer market, though it would become systematic if buyers imposed quality control procedures on
sellers in contracts of sale (as corporations very often do in their contracts with suppliers). So, when we find
beers that are apparently psychophysically identical selling for prices that depend on their advertising budgets,
we have a dull alternative hypothesis and an interesting one. The dull one is that advertising can make the
ignorance and error systematic at the level of markets, just as lasers with wavelengths resonant with the
internal structures and the sizes of molecules can make molecular motions in gases systematic. The interesting
one is that virtually content-less advertising is nevertheless information to a rational actor.

Bulls Eye

XAT PAPER 2014


6

17.

www.hitbullseye.com

Which of the following statements would be the closest to the arguments in the passage?
1. Individuals are more rational than firms.
2. Firms are rational.
3. Firms are more rational than individuals.
4. Firms are, most of the times, more rational than individuals.
5. Market behavior of psychophysical goods would be the same as that of physical goods.

18.

Why has the author referred to Rutherford in the passage?


1. To prove that molecules are elastic.
2. To highlight that we should not compare apples and oranges.
3. To hint that only very good students understood Physics taught by Rutherford.
4. To equate beer with little hard elastic balls.
5. To state that Mechanics is more amenable to application of Statistics than gasses.

19.

Which of the following, as per author, are psychophysical goods?


1. Concrete
1. 1 and 2

2. Car
2. 2 and 3

3. 1 and 3

3. Mobile Phone
4. 1,2 and 3

5. None of these

DIRECTIONS for questions 20 - 22: Analyze the following passage and provide appropriate answers
for the question that follows.
Ideas involving the theory of probability play a decisive part in modern physics. Yet we still lack a
satisfactory, consistent definition of probability; or, what amounts to much the same, we still lack a
satisfactory axiomatic system for the calculus of probability. The relations between probability and experience
are also still in need of clarification. In investigating this problem we shall discover what will at first seem an
almost insuperable objection to my methodological views. For although probability statements play such a
vitally important role in empirical science, they turn out to be in principle impervious to strict falsification.
Yet this very stumbling block will become a touchstone upon which to test my theory, in order to find out
what it is worth. Thus we are confronted with two tasks. The first is to provide new foundations for the
calculus of probability. This I shall try to do by developing the theory of probability as a frequency theory,
along the lines followed by Richard von Mises, but without the use of what he calls the 'axiom of
convergence' (or 'limit axiom'), and with a somewhat weakened 'axiom of randomness'. The second task is to
elucidate the relations between probability and experience. This means solving what I call the problem of
decidability of probability statements. My hope is that these investigations will help to relieve the present
unsatisfactory situation in which physicists make much use of probabilities without being able to say,
consistently, what they mean by 'probability'.
20.

The statement, "The relations between probability and experience are still in need of clarification",
implies that:
1. probability of an event can always be checked with experience.
2. probability of an event can only be gauged historically.
3. probability is mathematical while experience is real;
4. probability statements can become difficult to disprove without experience.
5. probability is futuristic.

Bulls Eye

XAT PAPER 2014


7

www.hitbullseye.com

21.

Author has talked about the two tasks in the above passage. Choose the best option from the following
statements relevant to the tasks.
1. The first task is sufficient to become the touchstone for the author to test his theory.
2. The second task is sufficient to become the touchstone for the author to test his theory.
3. Either of the tasks is sufficient for the author to test his theory.
4. None of the tasks is sufficient for the author to test his theory.
5. Both the tasks would be important for the author to test his theory.

22.

Which one of the following statements can be inferred from the passage?
1. Physics is the only subject that borrows from the theory of probability.
2. Physics is the only subject where the theory of probability is inaccurately applied.
3. The theory of probability may be inaccurately applied in other subjects.
4. Physics is highly mathematical.
5. Experience relates to physical objects only.

DIRECTIONS for questions 23 - 25: Analyze the following passage and provide appropriate answers
for the question that follows.
The ways by which you may get money almost without exception lead downward. To have done anything by
which you earned money merely is to have been truly idle or worse. If the laborer gets no more than the wages
which his employer pays him, he is cheated, he cheats himself. If you would get money as a writer or lecturer,
you must be popular, which is to go down perpendicularly. Those services which the community will most
readily pay for, it is most disagreeable to render. You are paid for being something less than a man. The State
does not commonly reward a genius any more wisely. Even the poet laureate would rather not have to
celebrate the accidents of royalty. He must be bribed with a pipe of wine; and perhaps another poet is called
away from his muse to gauge that very pipe. The aim of the laborer should be, not to get his living, to get "a
good job," but to perform well a certain work; and, even in a pecuniary sense, it would be economy for a town
to pay its laborers so well that they would not feel that they were working for low ends, as for a livelihood
merely, but for scientific, or even moral ends. Do not hire a man who does your work for money, but him who
does it for love of it.
The community has no bribe that will tempt a wise man. You may raise money enough to tunnel a mountain,
but you cannot raise money enough to hire a man who is minding his own business. An efficient and valuable
man does what he can, whether the community pays him for it or not. The inefficient offer their inefficiency to
the highest bidder, and are forever expecting to be put into office. One would suppose that they were rarely
disappointed. God gave the righteous man a certificate entitling him to food and raiment, but the unrighteous
man found a facsimile of the same in God's coffers, and appropriated it, and obtained food and raiment like
the former. It is one of the most extensive systems of counterfeiting that the world has seen. I did not know
that mankind was suffering for want of gold. I have seen a little of it. I know that it is very malleable, but not
so malleable as wit. A grain of gold will gild a great surface, but not so much as a grain of wisdom.
23.

Which of the following would the author disagree most with?


1. Setting up a factory in a rural area
3. Studying in a business school
5. Working for an investment bank

24.

2. Advertising for tooth paste


4. Betting in a casino

Which of the following could be a good title for the above passage?
1. Money and Work
4. In Search for God

2. God Rush
5. God is Gold

3. Work is Worship

Bulls Eye

XAT PAPER 2014


8

25.

www.hitbullseye.com

The author of the passage went on to say: "We are provincial, because we do not find at
home our standards; because we do not worship truth, but the reflection of truth; because we are
warped and narrowed by an exclusive devotion to trade and commerce and manufactures and
agriculture and the like, which are but means, and not the end."
Which of the following, as per author, could have been the end (last words in the lines above)?
1. Economic growth of society
4. Strong and powerful nation

2. Realization of self
5. Distribution of wealth

3. Happy family life

DIRECTIONS for questions 26 - 28: Analyze the following passage and provide appropriate answers
for the question that follows.
Either explicitly or implicitly, our informants suggest that the objects that transfix them are hoped to be
conduits to, rather than surrogates for, love, respect, recognition, status, security, escape, or attractiveness.
These are the social relations we desire, consciously or subconsciously, beneath the objects that we find so
compelling. The value of the objects that we focus our longing upon inheres less in the object or in a Lacanian
search for childhood love than in the culture. The hope for hope that an altered state of being may result keeps
the cycle of desire moving. Desires are nurtured by self-embellished fantasies of a wholly different self, and
they may be stimulated by external sources, including advertising, retail displays, films, television programs,
stories told by other people, and the consumption behavior of real or imaginary others. But we find that the
person who feels strong desire has almost always actively stimulated this desire by attending, seeking out,
entertaining, and embellishing such images. The desires that occupy us are vivid and riveting fantasies that we
participate in nurturing, growing, and pursuing, through self-seduction.
The social nature of desire implies that preferences of consumers are far from being independent. Yet, choice
models assume that preferences of consumers act as individuals. The mimetic aspect of desire creates
difficulties for using individual attitude or intention measures to predict adoption of new products whose use
will be visible. The notion of desire we have derived suggests that the appeal of the desired object is not
inherent in the object itself. Models that begin with preferences for product attributes or benefits are therefore
problematic. The consumer, individually and jointly, has a role in constructing the object of desire, within a
social context. What makes consumer desire attach to a particular object is not so much the object's particular
characteristics as the consumer's own hopes for an altered state of being, involving an altered set of social
relationships.
26.

Consider the statement given below as true:


"The failure of men to transition from being shoppers and consumers to producers and creators has
implications about their manliness. "
Which of the following statements would concur with the above idea and the theme of the main
paragraph?
1. Manliness is no longer socially desirable.
2. Boys will be boys and will always consume more.
3. Men will be men and will always create and produce.
4. Products that fulfill the desire will sell more.
5. Consumers would like to buy more do-it-yourself kits.

Bulls Eye

XAT PAPER 2014

www.hitbullseye.com

27.

Consider the statement given below as true:


"Men use the plasticity of consumer identity construction to forge atavistic masculine identities based
upon an imagined life of self-reliant, pre-modern men who lived outside the confines of cities,
families, and work bureaucracies. "
Which of the following statements would concur with the above idea and the theme of the main
paragraph?
1. Pre-modern man was anti-social.
2. Products that fulfill the desire of anti-social behavior will sell more.
3. Modern men are always anti-social.
4. Consumer will satisfy their desire of masculine identity through socially visible consumption.
5. Men will fulfill their desires for maintaining their identity.

28.

Consider the statement given below as true:


"By appropriating fashion discourse, consumers generate personalized fashion narratives and
metaphoric and metonymic references that negotiate key existential tensions and that often express
resistance to dominant fashion norms in their social milieu or consumer culture at large."
Which of the following statements would concur with the above idea and the theme of the main
paragraph?
1. Females in human society are creating personalized fashion narratives.
2. It is socially desirable to resist all dominant norms.
3. Feminists consider it socially desirable to resist dominant fashion norms that typify females in
human society.
4. Consumers often resist dominant fashion norms.
5. Attractiveness increases if one is unique.

Bulls Eye

XAT PAPER 2014


10

www.hitbullseye.com

Section B: Decision Making


DIRECTIONS for questions 29 - 32: Read the following case-let and answer the questions that follow.
Krishna Reddy was the head of a pharmaceutical company that was trying to develop a new product. Reddy,
along with his friend Prabhakar Rao, assessed that such products had mixed success. Reddy and Rao realized
that if a new product (a drug) was a success, it may result in sales of 100 crores but if it is unsuccessful, the
sales may be only 20 crores. They further assessed that a new drug was likely to be successful 50% of times.
Cost of launching the new drug was likely to be 50 crores.
29.

How much profit can the company expect to earn if it launches the new drug (suppose there
are no additional costs)?
1. 12 crores

30.

2. 10 crores

3. 24 crores

4. 11 crores

5. 11.5 crores

Now, Reddy and Rao were in a quandary whether the company should go ahead and market the drug.
They contacted Raj Adduri, a common friend for advice. Adduri was of the opinion that given the
risky nature of launch, it may be a better idea to test the market. Rao and Reddy realized test
marketing would cost 10 crores. Adduri told them the previous test marketing results have been
favorable 70% of times and success rate of products favorably tested was 80%. Further, when test
marketing results were unfavorable; the products have been successful 30% of the times.
How much profit can the company expect to make if the product is launched after favorable
test marketing results (assume there are no additional costs)?
1. 11.5 crores

31.

2. 10 crores

3. 24 crores

5. 6 crores

What is the probability of product failure if Reddy and Rao decides to test market it?
1. 0.21
4. 0.28

32.

4. 13.8 crores

2. 0.35
5. Cannot be computed

3. 0.14

If Rao and Reddy decides to launch the product despite unfavorable test marketing, how
much profit can the company expect to earn?
1. -13.2crores

2. -36.8 crores

3. -46.8 crores

4. -16 crores

5. 10 crores

DIRECTIONS for questions 33 - 35: Read the following case-let and answer the questions that follow.
Ms. Banerjee, class teacher for 12th standard, wants to send teams (based on past performance) of three
students each to district, state, national and international competition in mathematics. Till now, every student
of the class has appeared in 100 school level tests. The students had following distribution of marks in the
tests, in terms of "average" and "number of times a student scored cent per cent marks".
Student

Average

1
2
3
4
5
6
7
8
9

70
60
65
70
65
65
65
60
65

Number of times a student


scores cent per cent
7
15
8
1
6
10
4
12
3

Bulls Eye

XAT PAPER 2014


11

www.hitbullseye.com

10
11
12
13
14
15

60
70
65
70
60
65

8
1
6
2
20
5

Ms. Banerjee has carefully studied chances of her school winning each of the competitions. Based on in-depth
calculations, she realized that her school is quite likely to win district level competition but has low chances of
winning the international competition. She listed down the following probabilities of wins for different
competitions. Prize was highest for international competition and lowest for district level competition (in that
order).
Competition Probability of win
District
0.95
State
0.6
National
0.1
International
0.05
All the students are studying in the school for last twelve years. She wanted to select the best team for all four
competitions (Ms. Banerjee had no other information to select students).
33.

Which of three members should form the team for the International competition?
1. 4, 11, 14

34.

2. 2, 8, 14

5. 1,3,4

2. 1,4,11
5. Any team can win the competition

3. 4, 5, 6

Ms. Banerjee has to select the team for national competition after she has selected the team
for international competition. A student selected for international competition cannot be a part of
national competition. Which is the best team for the national competition?
1. 1,7,4
4. 3, 6, 1

DIRECTIONS for questions 36 - 39:


provided.
36.

4. 13,14,15

Which of the following members should constitute the team for the district level competition?
1. 4, 11, 14
4. 4, 11, 13

35.

3. 1,6, 12

2. 8, 9, 10
3. 2, 8, 14
5. Any of remaining students, as it would not matter
Read the following question based on information that is

The main issues of interpretation arising from the work of professionally trained anthropologists are
that they are late in colonial/post-colonial trajectories, because professional training shapes their
interpretations. However, within field of interest and training, their works are most thorough and
systematic.
The best conclusion drawn from the above paragraph is analogous to:
1. Heisenberg uncertainty principle, which states that speed and position cannot be determined
simultaneously.
2. Cultural relativism, which states that two or more than two cultures cannot be compared.
3. Personal relativism, which states that one should not study anthropological phenomenon for
personal gains.

Bulls Eye

XAT PAPER 2014


12

www.hitbullseye.com

4. Conclusive relativism, which states that anthropologists should not knowingly color their
findings.
5. Communicative relativism, which states that anthropologists should not be selective in
communicating their findings.
37.

Consider merit pay for teachers. Schools face constant pressure to change their management
approaches to improve performance, which is usually assessed by standardized reading, math,
and science scores. In most schools teachers' pay is determined by seniority, years of total
teaching experience, and credentials. Pay is rarely based on performance, which is contrary to
the belief among parents and private sector. Parents and business leaders lament that there are no
carrots/sticks used to motivate teachers. Consequently, there has been greater push to implement
some form of merit pay to improve the motivation.
Which of the following statements will disprove the claim of the parents and business leaders?
1. A recent study suggested that teachers are self- motivated.
2. Teachers are largely motivated, by financial incentives; so pay for performance will induce
greater and more effective effort.
3. Learning cannot be measured reliably and accurately by a test given once a year.
4. Teaching is a solo-activity, there is a little interdependence with other co-curricular and extra
curricular activities in the school.
5. To err is human. And hence stick should be used to reduce errors.

38.

In a new study by Harvard School of Public Health (HSPH), researchers explored how caffeine can
serve as a "mild anti-depressant". They concluded that "drinking several cups of coffee daily appears
to reduce the risk of suicide". Data pulled in from three large studies in the US showed that the suicide
risk of those who drank two to four cups of caffeinated coffee a day was about half of those who
drank decaffeinated coffee or very little or no coffee. In the studies, a respondent's caffeine
consumption was assessed every four years through a questionnaire. The respondents were all adults,
and the study was published online. The authors, however, cautioned the public from increasing
coffee intake as it could result in "unpleasant side effects". "Overall, our results suggest that there is
little further benefit for consumption above two to three cups/day or 400 mg of caffeine/day," wrote
the researchers. The authors observed that there was no major difference in suicide risk for adults who
drank two to three cups a day from those who drank four cups or more.
Which of the following shaped graph would best capture the above paragraph (x-axis represents
"coffee intake" and Y-axis represents "suicidal tendency")?
1. A straight line

39.

2. Saw tooth curve 3. S shaped curve

4. U shaped curve 5. L shaped curve

A group of nine runners will finish the 400 meters race in a certain order. The runners are:
Ashok, Benjamin, Chetan, Divya, Eshant, Faneesh, Girish, Himani, and Irravaty. They all finish
at different times, and their finishing order is as follow:

Faneesh finishes before Ashok.


Divya finishes before Benjamin and Eshant.
Irravaty finishes after Chetan.
Girish finishes after Ashok.

Which is the best position Girish can finish?


1. First

2. Second

3. Third

4. Fourth

5. Fifth

Bulls Eye
www.hitbullseye.com

XAT PAPER 2014


13

DIRECTIONS for questions 36 - 39: Read the following case-let and answer the question that follows.
Rajinder Singh was 32 years old from the small town of Bhathinda, Punjab. Most of the families living there
had middle class incomes, with about 10% of the population living below the poverty level. The .population
consisted of JJ) percent small traders, 30 .percent farmers, besides others. Rajinder liked growing up in
Bhathinda, where people knew and cared about each other.
Even as a youngster it was clear that Rajinder was smart and ambitious. Neighbors would often say,
"Someday you're going to make us proud!" He always had a job growing up at Singh's General Store - Uncle
Balwant's store. Balwant was a well-intentioned person. Rajinder loved being at the store and not just because
Balwant paid him well. He liked helping customers, most of who were known by the nicknames. Setting up
displays and changing the merchandise for different seasons and holidays was always exciting. Uncle Balwant
had one child and off late, his interest in business had declined. But he had taught Rajinder 'the ins and outs of
retailing'. He had taught Rajinder everything, including ordering merchandise, putting on a sale, customer
relations, and keeping the books.
The best part about working at the store was Balwant himself. Balwant loved the store as much as Rajinder
did. Balwant had set up the store with a mission to make sure his neighbors got everything they needed at a
fair price. He carried a wide variety of goods, based on the needs of the community. If you needed a snow
shovel or piece of jewelry for your wife, it was no problem - Singh's had it all. Rajinder was impressed by
Balwant's way of handling and caring for customers. If somebody was going through "hard times", Balwant
somehow knew it. When they came into the store, Balwant would make them feel comfortable, and say
something like, "you know Jaswant, let's put everything on credit today". This kind of generosity made it easy
to understand why Balwant was loved and respected throughout the community.
Rajinder grew up and went to school and college in Bhathinda. Later on, he made it to an MBA program in
Delhi. Rajinder did well in the MBA course and was goal oriented. After first year of his MBA, the career
advisor and Balwant advised Rajinder for an internship at Bigmart. That summer, Rajinder was amazed by the
breadth and comprehensiveness of the internship experience. Rajinder got inspired by the life story of the
founder of Bigmart, and the values the founder held. Bigmart was one of the best companies in the world.
The people that Rajinder worked for at Bigmart during the internship noticed Rajinder's work ethic,
knowledge, and enthusiasm for the business. Before the summer ended, Rajinder had been offered a job as a
Management Trainee by Bigmart, to start upon graduation. Balwant was happy to see Rajinder succeed. Even
for Rajinder, this was a dream job - holding the opportunity to move up the ranks in a big company. Rajinder
did indeed move up the ranks quickly, from management trainee, to assistant store manager, to store manager,
to supervising manager of three stores, to the present position - Real Estate Manager, North India. This job
involved locating new sites within targeted locations and community relations.
One day Rajinder was eagerly looking forward to the next assignment. When he received email for the same,
his world came crashing down. He was asked to identify next site in Bhathinda. It was not that Rajinder didn't
believe in Bigmart's expansion. What was printed in the popular press, especially the business press, only
reinforced Rajinder's belief in Bigmart. An executive viewed as one of the wisest business persons in the
world was quoted as saying, "Bigmart had been a major force in improving the quality of life for the average
consumer around the world, offering great prices on goods, giving them one stop solution for almost
everything." Many big farmers also benefitted as Bigmart's supply chain facilities helped reducing the waste.
Consumers also benefitted through low prices, as middlemen were removed. At the same time, Rajinder knew
that opening a new Bigmart could disrupt small businesses in Bhathinda. Some local stores in small towns
often went out of business within a year of the Bigmart's opening.
In Bhathinda, one of those local stores was Singh's, now run by Balwant's son, although Balwant still came in
every day to "straighten out the merchandise." As Rajinder thought about this assignment, depression set in,
and nightmares followed. Rajinder was frozen in time and space. Rajinder's nightmares involved Balwant

Bulls Eye

XAT PAPER 2014


14

www.hitbullseye.com

screaming something - although Rajinder could not make out what Balwant was saying. This especially
troubled Rajinder, since Balwant never raised his voice.
Rajinder didn't know what to do - who might be helpful? Rajinder's spouse, who was a housewife? Maybe
talking it through could lead to some positive course of action. Rajinder's boss? Would Bigmart understand?
Could Rajinder really disclose the conflict without fear? Uncle Balwant? Should Rajinder disclose the
situation and ask for advice? He wanted a solution that would make all stakeholders happy.
40.

Who is the best person for Rajinder to talk to?


1. Wife
3. Let time take its own course
5. His colleagues in the office

41.

2. Boss
4. Balwant

After deliberation with many people and a lot of research, Rajinder came across a study
published in leading journal, which stated that most local farmers benefited because Bigmart
bought agricultural produce directly from the farmers.
Which of the following actions would you prefer Rajinder to take, after he got this fresh information?
1. Since farmers benefit from Bigmart's presence, Rajinder should be happy with Bigmart's
presence in Bhathinda.
2. Rajinder should undertake a new study to find out influence of Bigmart on local farmers.
3. Rajinder should see this as an opportunity to take concerns of local traders to the top management
of Bigmart in India. It may help Bigmart to develop new solutions that can benefit Bigmart, retailers,
farmers and consumers.
4. Quit job at Bigmart and start another big retail chain to compete with Bigmart.
5. Request his boss to transfer him to a new location, which has a low percentage of mom and
pop stores. This would reduce Rajnder's discomfort.

42.

Which is the right ascending order, in terms of proportion of population, benefitting from.
Bigmart, in and around Bhathinda?
1. Consumers, farmers, retailers
3. Retailers, farmers, consumers
5. Consumers, retailers, farmers

2. Farmers, consumers, retailers


4. Farmers, retailers, consumers

DIRECTIONS for questions 43 - 45: Read the following case-let and answer the question that follows.
Intercontinental Business Manufacturing (IBM) was doing a roaring business. Demand of the products was
high and supply of raw-material was abundant. IBM was manufacturing three different products. Some
customers bought two types of products and some bought only one. The three products were "quickie-quick",
"run-of-the-mill", and "maxi-max". Customers were not complaining loudly.
Ram, the product manager, was confused! Demand for "quickie-quick" was increasing. Raw material
suppliers wanted to supply lower quality at cheaper price. It was profitable for the company to increase
production. Quality department was not happy with the product. Ram met Rahim, the CEO, who, as always,
wanted higher profits. He said that IBM will set up a committee for improving the quality.
43.

What must not be done by Ram?


1. Only produce "quickie-quick" as it gives highest profits.
2. Increase the production for "quickie-quick" and ask the supplier for better quality raw material.

Bulls Eye

XAT PAPER 2014

www.hitbullseye.com

15

3. Maintain the level of production for "quickie-quick" and ask the supplier for better quality raw
material.
4. Reduce the production of "quickie-quick" till committee submits its report.
5. Stop production of "quickie-quick" till committee submits its report.
44.

Rahim set up a "brand" committee comprising of a few selected managers, headed by Robert.
The committee proposed that IBM should continue to manufacture the three differently branded
products. It also proposed to recruit a new brand manager for improving brand image of the
products. It agreed with Rahim that the company should increase the price. Rocket Singh, head
of sales, was confused because he realized that customers were miffed with "run-of-the-mill" and
"maxi-max".
What should Rocket Singh do?
1. Launch one more product to increase sales.
2. Stop production of quickie-quick.
3. Launch a campaign on social media to increase awareness about company's products.
4. Send an anonymous email to all the employees highlighting customer dissatisfaction.
5. Conduct a research study to find out the reason for customers' dissatisfaction.

45.

Some of the managers in production department were discussing the problems faced in
shipping products in time. They complained that they had to undertake responsibility of creating
financial and marketing plans in addition to responsibility of production planning. At the same
time, finance and marketing managers were to be involved in preparing production plans. It was
expected that this will reduce customer complaints. It was rumored that these changes were
initiated by the managers educated in the U.S.
Which of the following is the best possible course of action available to the affected managers?
1. Create a union to safeguard their rights.
2. File a law suit against the company.
3. Create a forum for discussion and resolution of issues.
4. Register a complaint with the human rights body.
5. Go for further education to the U.S.

DIRECTIONS for questions 46 - 49: Please read the three reports (newspaper articles) on ranking of
different players and products in smart phones industry and answer the question that follow.
Report l; (Feb, 2013): Apple nabs crown as current top US mobile phone vendor
Apple's reign may not be long, as Samsung is poised to overtake Apple in April, 2013. For the first time since
Apple entered the mobile phone market in 2007, it has been ranked the top mobile phone vendor in the US.
For the latter quarter of 2012, sales of its iPhone accounted for 34 percent of all mobile phone sales in the
USincluding feature phonesaccording to the latest data from.Strategy Analytics.
While the iPhone has consistently been ranked the top smartphone sold in the .US, market research firm NPD
noted that feature phone sales have fallen off a cliff recently,.to the point where 8 out of every 10 mobile
phones sold in the US are now smartphones. That ratio is up considerably from the end of 2011, when
smartphones had just cracked the 50 percent mark.
Given this fact, it's no surprise that Apple, which only sells smartphones, has been able to reach the top of the
overall mobile phone market domestically. For the fourth quarter of 2012, Apple ranked number one with 34
percent of the US mobile market, up from 25.6 percent year over year. Samsung grew similarly, up to 32.3
percent from 26.9 percentbut not enough to keep from slipping to second place. LG dropped to 9 percent

XAT PAPER 2014


16

Bulls Eye
www.hitbullseye.com

from 13.7 percent, holding its third place spot. It should be noted that Samsung and LG both sell a variety of
feature phones in addition to smartphones.
Looking only at smartphones, the ranking is a little different according to NPD. Apple holds the top spot with
39 percent of the US smartphone market, while Samsung again sits at number two with 30 percent. Motorola
manages to rank third with 7 percent, while HTC dropped to fourth with 6 percent. In the US smartphone
market, LG is fifth with 6 percent. Note how the percentages aren't all that different from overall mobile
phone market sharefor all intents and purposes, the smartphone market is the mobile phone market in the
US going forward.
Still, Samsung was the top mobile phone vendor overall for 2012, and Strategy Analytics expects Samsung to
be back on top soon. "Samsung had been the number one mobile phone vendor in the US since 2008, and it
will surely be keen to recapture that title in 2013 by launching improved new models such as the rumored
Galaxy S4." And while Apple is the top vendor overall among smartphones, its iOS platform is still second to
the Android platform overall. Samsung is the largest vendor selling Android-based smartphones, but
Motorola, HTC, LG, and others also sell Android devices, giving the platform a clear advantage over iOS both
domestically and globally.
Report2: Reader's Response (2013, Feb)
I don't actually believe the numbers for Samsung. Ever since the debacle in early 2011, when Lenovo called
into question the numbers Samsung was touting for tablet shipments, stating that Samsung had only sold
20,000 of the 1.5 million tablets they shipped into the US the last quarter of 2010, Samsung (who had no
response to Lenovo) has refused to supply quarterly sales numbers for smartphones or tablets. That's an
indication that their sales aren't what analysts are saying.
We can look to several things to help understand why. In the lawsuit between Apple and Samsung here last
year, both were required to supply real sales numbers for devices under contention. The phones listed turned
out to have sales between one third and one half of what had been guessed by IDC and others. Tablet sales
were even worse. Of the 1.5 million tablets supposedly shipped to the US during that time, only 38,000 were
sold.
Then we have the usage numbers. Samsung tablets have only a 1.5% usage rate, where the iPad has over 90%.
Not as much a difference with the phones but it's still overwhelmingly in favor of the iPhone.
The problem is that with Apple's sales, we have actual numbers to go by. The companies who estimate can
calibrate what they do after those numbers come out. But with Samsung and many others, they can't ever
calibrate their methods, as there are no confirming numbers released from the firms. A few quarters ago, as a
result, we saw iSupply estimate Samsung's smartphone sales for the quarter at 32 million, With estimates from
others all over the place up to 50 million. Each time some other company reported a higher number for that
same quarter, the press dutifully used that higher number as THE ONE. But none of them was the one.
Without accurate self-reporting of actual sales to the end users, none of these market share charts" are worth a
damn!
Report3: Contradictory Survey (Feb, 2013): iPhone5 Ranks Fifth In U.S. Customer Satisfaction Survey
in Share
The iPhone5 ranks fifth in customer satisfaction according to the results of a recent survey from On Device
Research, a mobile device research group. In the poll, they asked 320,000 smartphone and tablet users from
six different countries, how satisfied they were with their devices. According to 93,825 people from the US,
Motorola Atrix HD is the most satisfying and Motorola's Droid Razr took second spot. HTC Corp
(TPE:2498)'s Rezound 4G and Samsung Galaxy Note 2 took third and fourth spots, while Apple's iPhone5
landed in fifth spot.

Bulls Eye

XAT PAPER 2014

www.hitbullseye.com

17

It appears, that Apple may be lagging in consumer interest. OnDevice Research, Sarah Quinn explained,
"Although Apple created one of the most revolutionary devices of the past decade, other manufactures have
caught up, with some Android powered devices now commanding higher levels of user satisfaction."
Despite the lower rankings, things aren't looking too bad for Apple Inc. (NASDAQ:AAPL) elsewhere. In the
United Kingdom, they ranked second place, right after HTC One X. Interesting enough, Apple did take top
spot for overall satisfaction of mobile device, whereas Google Inc. (NASDAQrGOOG) ranked second.
Motorola Mobility Holdings Inc. (NYSE:MMI), HTC Corp (TPE:2498), and Nokia Corporation
(NYSE:NOK) took third, fourth, and fifth places respectively, while Sony Ericsson trailed behind at sixth
place.
The survey sampled mobile device users in the following countries: United States, United Kingdom, France,
Germany, Japan, and Indonesia. Although OnDevice didn't share the full list of devices mentioned in the
survey, it does show some insight to what customers want. Unfortunately, there were still many questions
regarding the survey that were left unanswered. Everyone wants to know why Google Inc.
(NASDAQrGOOG) was on the list when they are not an actual smartphone maker and why was Samsung
Electronics Co., Ltd. (LON:BC94) on the bottom of the satisfaction list when the brand is leading elsewhere.

Fortunately, those questions were answered by OnDevice Research's representative. He explained that the
survey was conducted on mobile web where the survey software could detect the taker's device and since
users rate their satisfaction levels on a 1 to 10 scale, thanks to the Nexus device, Google was included.
46.

If you analyze the three reports above, which of the following statements would be the best inference?
1. Newspapers publish completely wrong data.
2. All three reports have been bought by the companies.
3. Newspapers may report selectively.
4. Apple is leading but its dominance in Korean market is definitely under threat.
5. Customer's response, in report 2, is neutral and objective.

47.

Assume report 1 and report3 are both correct, which of the following can be a reason for seemingly
divergent ranks of different companies and their products?
1. iPhone4, another product from Apple, could have been the best seller in the U.S.
2. HTC-One was the most preferred device in Japan.
3. OnDevice included Google in the list because it is an important player in Android market.
4. Customer satisfaction never leads to higher customer loyalty.
5. None of the above.

Bulls Eye

XAT PAPER 2014


18

48.

www.hitbullseye.com

If report 1 and report3 are both correct, which of the following can be a reason for seemingly
divergent ranks reported for different companies and their products?
1. OnDevice has been paid by Motorola to publish.
2. Customers in France, in report3, have voted Samsung as the favorite company.
3. Customers in Japan, in report3, have voted Samsung as the favorite company.
4. Sales and satisfaction may not be perfectly related with each other.
5. None of the above

49.

Which of the following is more likely to be a correct conclusion about the respondent/reader in
report2?
1. The reader is an Apple user.
2. If the reader is an Apple user, his usage rate is higher.
3. The reader is a frequent user of HTC One.
4. The reader does not like Android platform.
5. The reader wants more objective data to be released by the companies.

DIRECTIONS for questions 46 - 49: Read the following case-let and choose the best alternative for the
questions that follow.
Ajay was thinking deeply about a problem that his organization, a business consulting company, faced.
Globalization had affected his company like many other companies. Despite the downturn, the current
revenues remained healthy. However, Ajay knew it was inevitable that the company could not do business the
same way. The complexity of managing the business had increased with time. Consultants were under
pressure to deliver good and innovative solutions. The organization had consultants from different age groups
having a good mix of domain and industry expertise. It was a flat organization with three levels. The biggest
challenge for Ajay was to have consultants with latest knowledge who would also earn revenues. Getting
additional business was a challenge as all the consultants were busy and it was very difficult to hire new
consultants.
50.

Some of the consultants were adept at applying old solutions to new problems. Ajay was not
very sure if this would work for long. Some of the clients had complained about the performance
of old and reputed consultants. Ajay was mulling over the following five solutions to tackle this
problem.
A. Decrease time spent on client interaction and increase time spent for generating solutions.
B. Increase support staff to help consultants to remain updated.
C. Decrease the number of simultaneous projects handled by consultants.
D. Make it compulsory for consultants to work on inter-industry and inter-domain problems.
E. Recruit more consultants.
Which of the following would be the best sequence of decisions taken by Ajay (starting from
immediate to distant)?
1. C,B,A,E,D

51.

2. D,C,B,A,E

3. E,D,C,B,A

4. B,D,E,A,C

5. A,E,D,C,B

After Ajay implemented some of the steps mentioned above, consultants wanted to
renegotiate their contract with the organization. It seems that the organization had never
mentioned that consultants have to work across industries and domains. Some of the old
consultants were reluctant to change their ways, while many of the younger consultants were
willing to follow Ajay's advice. Which of the following decision can be taken by Ajay to handle
this situation?

Bulls Eye

XAT PAPER 2014


19

www.hitbullseye.com

1. Retrench old consultants and recruit young consultants.


2. Pressurize non-conformists by giving preferential treatment to the conformists.
3. Negotiate with the old consultants and communicate that the new rules would apply to the
new consultants only.
4. Do nothing and wait for a right solution to emerge, as with time resistance would die down.
5. Discuss the issue in an open house and let solutions emerge democratically.
52.

Ajay was to retire in five years and he wanted to leave behind a legacy. Order the following
activities, from the most important to the least important, that Ajay should undertake in next five
year:
A. Do nothing.
B. Set a future direction for the organization in these challenging times.
C. Benchmark performance with respect to the best consulting company in industry.
D. Empower senior consultants and at the same time seek opinion of all others for handling future
challenges.
E. Infuse fresh thinking by hiring outsiders.
Choose the best option from the following sequences
1. A,B,C,D

2. B,C,D,E

3. C,D,B,A

4. B,E,C,A

5. B,E,D,C

Bulls Eye

XAT PAPER 2014


20

www.hitbullseye.com

Section C: Quantitative Ability


53.

Consider the formula, S

*
, where all the parameters are positive integers. If is increased
*

and , and are kept constant, then S:


1. increases
3. increases and then decreases
5. cannot be determined
54.

Prof. Suman takes a number of quizzes for a course. All the quizzes are out of 100. A student can get
an A grade in the course if the average of her scores is more than or equal to 90. Grade B is awarded
to a student if the average of her scores is between 87 and 89 (both included). If the average is below
87, the student gets a C grade. Ramesh is preparing for the last quiz and he realizes that he must score
a minimum of 97 to get an A grade. After the quiz, he realizes that he will score 70, and he will just
manage a B. How many quizzes did Prof. Suman take?
1. 6

55.

2. 7

3. 8

4. 9

5. None of these

A polynomial "ax3 + bx2 + cx + d" intersects x-axis at 1 and -1, and y-axis at 2. The value of b is:
1. -2
4. 2

56.

2. deceases
4. decreases and then increases

2. 0
5. Can't be determined

3. 1

Consider four natural numbers: x, y, x + y, and x - y. Two statements are provided below:
I. All four numbers are prime numbers.
II. The arithmetic mean of the numbers is greater than 4.
Which of the following statements would be sufficient to determine the sum of the four numbers?
1. Statement I.
3. Statement I and Statement II.
5. Either Statement I or Statement II.

57.

2. Statement II.
4. Neither Statement I nor Statement II.

Triangle ABC is a right angled triangle. D and E are mid points of AB and BC respectively.
Read the following statements.
I. AE=19
II. CD = 22
III. Angle B is a right angle.
Which of the following statements would be sufficient to determine the length of AC?
1. Statement I and Statement II.
3. Statement II and III.
5. All three statements.

58.

2. Statement I and Statement III.


4. Statement III alone.

There are two circles C1 and C2 of radii 3 and 8 units respectively. The common internal tangent, T,
touches the circles at points P1 and P2 respectively. The line joining the centers of the circles intersects
T at X. The distance of X from the center of the smaller circle is 5 units. What is the length of the line
segment P1P2?
1.< 13

2. > 13 and < 14

3. > 14 and < 15

4. > 15 and < 16

5. > 16

Bulls Eye

XAT PAPER 2014


21

www.hitbullseye.com

59.

x, 17, 3x - y2 - 2, and 3x + y2 - 30, are four consecutive terms of an increasing arithmetic sequence.
The sum of the four numbers is divisible by:
1. 2

60.

2. 3

2. > 12 and < 14


5. cannot be determined

2. 1

2. 5

2. > 8 and < 11

5. None of these

2. > 9 m and < 9.5 m


4. > 10 m and < 10.5 m

5. > 10.5 m

3. 7

4. 9

5. None of these

3. > l1 and < 14

4. > 14 and < 17

5. > 17

Diameter of the base of a water-filled inverted right circular cone is 26 cm. A cylindrical pipe, 5 mm
in radius, is attached to the surface of the cone at a point. The perpendicular distance between the
point and the base (the top) is 15 cm. The distance from the edge of the base to the point is 17 cm,
along the surface. If water flows at the rate of 10 meters per minute through the pipe, how much time
would elapse before water stops coming out of the pipe?
1. < 4.5 minutes
3. > 4.8 minutes but < 5 minutes

66.

4. 3

There are two squares S1 and S2 with areas 8 and 9 units, respectively. S1 is inscribed within S2, with
one corner of S1 on each side of S2. The corners of the smaller square divides the sides of the bigger
square into two segments, one of length 'a' and the other of length 'b', where, b > a. A possible value
of 'b/a', is:
1. > 5 and < 8

65.

3. 2

Amitabh picks a random integer between 1 and 999, doubles it and gives the result to Sashi. Each
time Sashi gets a number from Amitabh, he adds 50 to the number, and gives the result back to
Amitabh, who doubles the number again. The first person, whose result is more than 1000, loses the
game. Let 'x' be the smallest initial number that results m a win for Amitabh. The sum of the digits of
'x' is:
1. 3

64.

3. > 14 and < 16

There are two windows on the wall of a building that need repairs. A ladder 30 m long is placed
against a wall such that it just reaches the first window which is 26 m high. The foot of the ladder is at
point A. After the first window is fixed, the foot of the ladder is pushed backwards to point B so that
the ladder can reach the second window. The angle made by the ladder with the ground is reduced by
half, as a result of pushing the ladder. The distance between points A and B is
1.< 9m
3. > 9.5 m and < 10 m

63.

5. 11

The sum of the possible values of X in the equation |X + 7| + |X - 8| = 16 is:


1. 0

62.

4. 7

In quadrilateral PQRS, PQ = 5 units, QR = 17 units, RS = 5 units, and PS = 9 units. The length of the
diagonal QS can be:
1. > 10 and < 12
4. > 16 and <18

61.

3. 5

2. > 4.5 minutes but < 4.8 minutes


4. > 5 minutes but < 5.2 minutes

E. > 5.2 minutes

Aditya has a total of 18 red and blue marbles in two bags (each bag has marbles of both colors). A
marble is randomly drawn from the first bag followed by another randomly drawn from the second
bag, the probability of both being red is 5/16. What is the probability of both marbles being blue?
1. 1/16

2. 2/16

3. 3/16

D. 4/16

E. None of these

Bulls Eye

XAT PAPER 2014


22

67.

www.hitbullseye.com

Read the following instruction carefully and answer the question that follows:
13
x
1
can also be written as
Expression
13!
n 1 n
What would be the remainder if x is divided by 11 ?

1. 2
68.

2.4

4. 9

5. None of these

A rectangular swimming pool is 48 m long and 20 m wide. The shallow edge of the pool is 1 m deep.
For every 2.6 m that one walks up the inclined base of the swimming pool, one gains an elevation of 1
m. What is the volume of water (in cubic meters), in the swimming pool? Assume that the pool is
filled up to the brim.
1. 528

69.

3. 7

2. 960

3. 6790

4. 10560

5.12960

3. 1

4. 10

5. 100

The value of the expression:


100

log!100! is :
i2

1. 0.01
70.

2. 0.1

The probability that a randomly chosen positive divisor of 1029 is an integer multiple of 1023 is: a2/b2,
then 'b - a' would be:
1. 8

71.

5. 45

2. 1 6

3. 1 3

4. 1

5. None of these

2. > 10 and < 10.5 3. > 10.5 and < 11 4. > 11 and < 11.5 5. > 11.5

Two numbers, 297B and 792B belong to base B number system. If the first number is a factor of the
second number then the value of B is:
1. 11

74.

4. 23

Consider a rectangle ABCD of area 90 units. The points P and Q trisect AB, and R bisects CD. The
diagonal AC intersects the line segments PR and QR at M and N respectively. What is the area of the
quadrilateral PQMN?
1. > 9.5 and < 10

73.

3. 21

Circle C1 has a radius of 3 units. The line segment PQ is the only diameter of the circle which is
parallel to the X axis. P and Q are points on curves given by the equations y = ax and y = 2ax
respectively, where a < 1. The value of a is:
1. 1 6

72.

2. 15

2. 12

3. 15

4. 17

5. 19

A teacher noticed a strange distribution of marks in the exam. There were only three distinct scores: 6,
8 and 20. The mode of the distribution was 8. The sum of the scores of all the students was 504. The
number of students in the in most populated category was equal to the sum of the number of students
with lowest score and twice the number of students with the highest score. The total number of
students in the class was:
1. 50

2. 51

3. 53

4. 56

5. 57

Bulls Eye

XAT PAPER 2014


23

www.hitbullseye.com

DIRECTION for the questions 75 to 77: Based on the following information given below.

The exhibit given below compares the countries (first column) on different economic indicators (first row),
from 2000-2010. A bar represents data for one year and a missing bar indicates missing data. Within an
indicator, all countries have same scale..

Table given below contains data of (GDP) in constant 2000 US Dollar (in billions)
Country
Brazil
China
India
Malaysia
Philippines
Thailand
Unites States
Vietnam

75.

2000
645
1,198
475
94
81
123
9,899
31

2002
671
1,416
518
99
86
132
10,190
36

2. China

2004
717
1,715
603
112
97
150
10,814
41

2005
740
1,909
659
118
101
157
11,146
45

2006
769
2,151
720
125
107
165
11,443
48

2007
816
2,457
790
133
114
174
11,661
53

2008
858
2,693
821
140
119
178
11.619
56

2009
855
2,940
888
137
120
174
11,209
59

2010
919
3,246
973
147
129
187
11,548
63

3. India

4. Brazil

5. Thailand

Which country (and which year) has witnessed maximum year-to-year decline in "industry as
percentage of GDP"? Given that the maximum value of industry as percentage of GDP is 49.7% and
the minimum value of industry as percentage of GDP is 20.02%, in the chart above.
1. United States in 2002-3
4. Malaysia in 2008-9

77.

2003
678
1,558
559
105
91
141
10,450
38

Which of the following countries, after United States, has the highest spending on military as % of
GDP, in the period 2000-2010?
1. Vietnam

76.

2001
653
1,298
498
94
83
125
10,007
33

2. Brazil in 2006-7
5. China in 2008-9

3.India in 2009-10

Which of the following countries has shown maximum increase in the "services, value added as % of
GDP" from year 2000 to year 2010?
1. Brazil

2. India

3. United States

4. Philippines

5. None of these

Bulls Eye

XAT PAPER 2014


24

www.hitbullseye.com

DIRECTION for the questions 78 to 80: Based on the trends lines from the following graphs:

Note: Left side of X axis represents countries that are "poor" and right side of X axis represents countries that
are "rich ", for each region. GDP is based on purchasing power parity (PPP). These are World Bank (WB)
estimates

78.

Which of the following could be the correct ascending order of democratic regions for poor?
1. North America, C and E Europe, South America, Middle East, Asia Pacific
2. Scandinavia, Western Europe, Africa, Asia Pacific, Middle East
3. Scandinavia, Western Europe, North America, C and E Europe, Middle East
4. C and E Europe, Africa, South America, Western Europe, Scandinavia
5. Africa, South America, Western Europe, North America, Scandinavia

79.

Which region has the highest disparity, of democratic participation, between rich and poor?
1. North America

80.

2. C and E Europe 3. Africa

4. South America

5. Western Europe

The maximum GDP of African region is higher than the maximum GDP of South American region by
factor of:
1. 10

2. 100

3. 2

4. 4

5. None of these

Bulls Eye

XAT PAPER 2014


25

www.hitbullseye.com

DIRECTION for the questions 81 to 83: based on the given data on the tourism sector in India.

Year

Foreign Tourist
Arrivals in
India (in
millions)

Foreign Exchange
Earnings from
Tourism in India
(in US $ million)

Foreign Exchange
Earnings from
Tourism in India
(in Rs. Crore)

1997
1998
1999
2000
2001
2002
2003
2004
2005
2006
2007
2008
2009
2010
2011

2.37
2.36
2.48
2.65
2.54
2.38
2.73
3.46
3.92
4.45
5.08
5.28
5.17
5.78
6.29

2889
2948
3009
3460
3198
3103
4463
6170
7493
8634
10729
11832
11136
14193
16564

10511
12150
12951
15626
15083
15064
20729
27944
33123
39025
44360
51294
53700
64889
77591

81.

2. 2005 and 2006


5. 2004, 2005 and 2006

3. 2005 and 2007

In which of the following years was the rupee cheapest with respect to the dollar?
1.2001

83.

Number of
Domestic Tourist
Visits to all
States/UTs (in
millions)
159.88
168.2
190.67
220.11
236.47
269.6
309.04
366.27
392.01
462.32
526.56
563.03
668.8
747.7
850.56

In which of the following years the percentage increase in the number of Indians going abroad was
greater than the percentage increase in the number of domestic tourists?
1. 2004 and 2005
4. 2006 and 2008

82.

Number of Indian
Nationals
Departures from
India (in
millions)
3.73
3.81
4.11
4.42
4.56
4.94
5.35
6.21
7.18
8.34
9.78
10.87
11.07
12.99
14.21

2.2002

3. 2007

4. 2010

5. 2011

Let 'R' be the ratio of Foreign Exchange Earnings from Tourism in India (in US $ million) to Foreign
Tourist Arrivals in India (in million). Assume that R increases linearly over the years. If we draw a
pie chart of R for all the years, the angle subtended by the biggest sector in the pie chart would be
approximately:
1. 24

2. 30

3. 36

4. 42

5. 48

Bulls Eye

XAT PAPER 2014


26

www.hitbullseye.com

PART II: General Knowledge


84.

'Long Walk To Freedom' is the autobiography of the following Bharat Ratna recipient:
1. Jawaharlal Nehru
3. Khan Abdul Ghaffar Khan

85.

2. Medhatithi

2. Leh and Ladakh


5. Andaman and Nicobar Islands

2. 15

3. 19

3. Sumeria

2. Wangari Muta Maathai


4. Doris Lessing

5. Shirin Ebadi

The correct ascending order of countries based on the latest Gross Domestic Product data (Purchasing
Power Parity) from IMF is:
2. Germany, Japan, India, China, USA
4. Germany, Russia, Japan, China, USA

The distance covered in half marathon is approximately:


1. 21 km

93.

5. 12

The following women have been honored with The Nobel Peace Prize, except:

1. Brazil, Japan, India, China, USA


3. Brazil, India, Japan, China, USA
5. Russia, India, Japan, China, USA
92.

4. 25

2. Syria
5. None of the above

1. Mother Teresa
3. Aung San Suu Kyi
91.

3. Coromandel

The ancient civilization of Sumer flourished in a region, which is a part of the following country in
the modern world:
1. Iraq
4. Egypt

90.

2. Vienna (Austria)
4. New York (USA)

The number of officially recognized languages according to the Eighth Schedule of the
Indian Constitution is
1. 22

89.

4. Bhaskaracharya 5. Panini

Which Indian region is the only place in the world where the Asiatic Wild Ass is found?
1. Sunderban
4. Rann of Kutch

88.

3. Brahmagupta

Which city houses the headquarters of International Atomic Energy Agency (IAEA)?
1. Tokyo (Japan)
3. Geneva (Switzerland)
5. Brussels (Belgium)

87.

5. C. Rajgopalachari

Which seventh century Indian mathematician was the first in the world to treat 'zero' as a number and
show its mathematical operations?
1. Aryabhatta

86.

2. S. Radhakrishnan
4. Nelson Mandela

2. 42 km

3. 11 km

4. 7.5 km

5. 5 km

3. Eritrea

4. East Timor

5. South Sudan

The newest country in the world is:


1. Palau

2. Kosovo

Bulls Eye

XAT PAPER 2014


27

www.hitbullseye.com

94.

The G-7 is a group consisting of:


1. the heads of the states of US, UK, France, Germany, Italy, Canada and Japan
2. the finance ministers of US, UK, France, Germany, Italy, Canada and Japan
3. the heads of the states of US, UK, France, Germany, Russia, Canada and Japan
4. the finance ministers of US, UK, France, Germany, Russia, Canada and Japan
5. the heads of the states of US, UK, France, Germany, Russia, Italy and Japan

95.

The correct match between countries and their official languages is:
1. Brazil-Spanish, Barbados-English, Haiti-Portuguese, Chile-French
2. Brazil-Spanish, Barbados-Portuguese, Haiti-French, Chile-Spanish
3. Brazil-Portuguese, Barbados-English, Haiti-French, Chile-Spanish
4. Brazil-Portuguese, Barbados-French, Haiti-Spanish, Chile-French
5. Brazil-Spanish, Barbados-French, Haiti-Portuguese, Chile Spanish

96.

The correct match between the following mountain ranges and their highest peaks is:
1. Alps: Mont Blanc, Rockies-Mount Albert, Andes- Mount Aconcagua, Karakoram-K2
2. Alps: Mount Albert, Rockies-Aconcagua, Andes -Mont Blanc, Karakoram-K2
3. Alps: Mount Albert, Rockies-Aconcagua, Andes -K2, Karakoram-Mont Blanc
4. Alps-Mont Blanc, Rockies-Aconcagua, Andes-Mount Albert, Karakoram-K2
5. Alps- Aconcagua, Rockies-Mount Albert, Andes -Mont Blanc, Karakoram-K2

97.

The acronym "YAHOO" stands for:


1. A song sung by Shammi Kapoor
2. Yet Another Hierarchical Object Orientation
3. Yet Another Hierarchical Officious Oracle
4. Young American Hi-tech Organization in Ohio
5. None of these

98.

The list of persons not containing an RBI governor is:


1. Raghuram Rajan, D. Subbarao, I.G Patel, R. N Malhotra
2. Y. V Reddy, Bimal Jalan, C. Rangarjan, S. Venkitaraman
3. Manamohan Singh, A. Ghosh, D. Subbarao, K. R Puri
4. Raghuram Rajan, D. Subbarao, Y. V Reddy, James Taylor
5. None of the above

99.

The factor that least influences exchange rate fluctuations:


1. Industrial Production
4. Consumer Price Index

100.

2. Investor sentiments
5. Goods and Services Tax

3. Interest Rates

The sequence that contains a wrong match between car manufacturers and their country of origin:
1. Bugatti-France, Opel-Germany, Ferrari-Italy, Jaguar-England
2. Cherry Automobile-U.K, Rolls-Royce-U.K, Austin-U.K, Kia-Korea
3. First Automobile Works-China, The First Cars-Japan, Lancia-Italy, Saab-Sweden
4. Volvo-Sweden, Audi-Germany, Skoda-Czech Republic, Tatra-Austria
5. Volkswagen-Germany, Audi-Germany, Opel-Germany, Mercedes Benz-Germany

Bulls Eye

XAT PAPER 2014


28

101.

www.hitbullseye.com

As per the Constitution, which among the following is not a fundamental right granted to the citizens
of India?
1. Right to Equality
3. Right Against Exploitation
5. Right to Freedom of Religion

102.

Jawaharlal Nehru declared 'Poorna Swaraj' at:


1. Surat (1907)

103.

2. Right to Freedom
4. Right to Property

2. Belgaum(1924) 3. Lahore (1929)

4. Tripuri(1939)

5. Calcutta (1938)

Who among the following was known as the 'Saint of the Gutters' ?
1. Baba Amte
4. Jyotirao Phule

2. Mother Teresa
5. Dr. Bindeshwar Pathak

3. C.F. Andrews

Bulls Eye

XAT PAPER 2014


29

www.hitbullseye.com

XAT Paper-2014

Key & Explanations

Q.
1.

Key

Explanations

2.

3.

4.

5.

6.

7.

8.

9.

10.

11.

12.

13.
14.

4
2

15.

16.

Line 1 will be the opening line as it mentions the mechanism of electroweak symmetry as one of the
most important issues in Particle physics. Line 4 will form a logical pair as the SUSY further
elaborates the term(most important issue mentioned in Line 1.Line 3 refers to the fundamental Higgs
doublet(Higgs sector consists of 2 chiral superfields of Higgs doublets).Line 5 refers to the
minimal SUSY further describes the Higgs sector which consists of (H1 and H2 with opposite
hypercharges) Line 2 will follow as it further justifies the function of SUSY (required to give masses
for all quarks.gauge anomaly). So, the correct sequence will be 14352.
Line 1 is the opening line as it refers to the term no official collection of Shakespeares play appeared
until his death. Line 4 refers to Shakespeares collection of sonnets that appeared before 1609 just
before his death. Line 5&3 will form a logical pair as the term quarots is further elaborated in Line
3.Also,line 2 justifies the term quartos(were probably based on actors memories of the play).So the
correct sequence will be 14532
Since the word as is used ,so synonyms or the same tone options will be appropriate .In option
1,Blithely means without thought or regard; carefree; heedless or joyous: Undesirable means not
likeable. As both the options are contrastive in nature, so this option will be inappropriate .In option
2,Cavalierly means to be arrogant or haughty, disdainful, and Apocryphal means of doubtful
authorship .Vehemently means characterized by rancor or anger; So, the most appropriate is
B
The use of not just is suggestive of the fact contrastive option will eb the most appropriate .In option
1retribution means requital according to merits or deserts, especially for evil or retaliation.
Camaraderie means comradeship. Guilt means the fact or state of having committed a crime,
violation, or wrong, especially against moral offence. However, crime and religion do not fit in the
passage.In option E recrimination is a countercharge and insight is to a deep understanding of a
concept. So the most appropriate option will be A
Separate studies for the both types of excitations and now the last line will focus on when these two
theories related to their combined effect on atomic nuclei.
Precise instead of exact will be more appropriate with techniques on the basis of which, the
appropriate options will be C&D. Since ,we use increase for numbers/numerals and improvement
with qualities, so Improvement is the right word that goes with notions. SO the correct option is C.
Out of such as and like, the former is preferred because such as is used for a particular
characterstic and like is used for the process of execution. Since the worddisorder has been
mentioned, so such as will be more appropriate. So options A&E are left. Since it cannot be the
host of emotional and behavioural disorders, so the appropriate option is A.
Though but and yet are counter indicators, a small difference still exists .While yet is used in
situations where despite the likelihood, the event doesnt occur and vice versa. On the contrary,
but is used to suggest simple counter opinions. In the question but will be preferred over
yet, so options A&E are left. Since, this subsumption relation is specific, article the will be used
instead of a. SO the correct option will be E
2-pronoun as it refers to noun MBA Degree, 3- verb(helps) , 4- adjective(more important) , 7- verb
(tackle), 8- conjunction(and). Hence the answer is option 5.
The media paid high attention on char dham inspite of the fact the other areas were also badly
affected. Hence the answer is option 3
The given statement is correct. OLDER cannot come as no two things are c compared and not only
is followed by but also..
Statement 1 is an authors own perception regarding people achieving greatness by a few in their
chosen field therefore it is an assumption and it is true also as explained further. Statement 2 is a
question statement. 4 is a reason and the clue word is because. 7 is a conclusion.
All the statements are inductive inferences as all of them are based on a real incidence.
The author says that army like governments is inexpedient and there can be so many objections can be
raised against so many them. Hence answer is option 2
After reading the last few lines author emphasizes that government and hence legislators can deliver
justice or give fair treatment to a minority if decisions are conscientious. Hence answer is option 4
In the last few lines the author says we should be men first and subjects afterwards.. This refers to

XAT PAPER 2014


30

Bulls Eye
www.hitbullseye.com

17.

18.

19.

20.

21.

22.

23.

24.

25.

26.

27.

28.

29.

option A as the best.


As per the lines from the first Para the fact that most beer . Clearly something is wrong
somewhere points out that the answer is option 4.
The author has given thy example of Rutherford experiment is to prove that incomparable things
should not be compared.
None of the options fall under the category of psychophysical goods except for a concrete a little as it
is not concrete, the perception of which may or may not change after increasing or decreasing the
advertising budget. Hence answer is option 5.
The passage says that probability lacks consistency. There is a gap between probability and
experience as per the passage and given line. Therefore answer is option 4
The author says that we are confronted by two tasks and he goes on explaining these two tasks further
in the passage. Hence the both the task are required to test the theory.
From the first few lines and last few lines from the passage it can be inferred that theory may have
been inaccurately applied in other subjects.
As per the author one should undertake the career out of love for that particular area/work and not for
the money/remuneration. Betting in a casino is solely for sake of making money out of greed and that
is not desirable as per the author. Therefore author will not agree with the option 4.
In the entire passage the author is talking about the way one should pick up/choose the career/work
and it should not be solely based on wages one will get or if it is a lucrative opportunity. On the
contrary the choice of career/work should be as per the liking/love for that work/job. Finally he wants
to prove that work is of prime importance and not the money involved. Therefore the best tile will be
option 3 as when worship takes the front seat then other factors such as money will take the back
seat.
The author in the lines say that we are narrow when it comes to searching for truth instead we keep on
moving around the perceptions, which we consider truth where there is none. We consider only the
means and do not worry or think about the results. Therefore it should be realization of truth above all
rather than running after perishables. Hence the answer is option 2.
The passage highlights that it is a mistake to think that consumers preferences can be changed as
most of the business/ choice models think instead it is independent of models and more related to
desires of the consumers and they will try to find the desires in the products that they buy. Out of all
the options this is possible in the option 5.
The given lines points out option 4 as the passage says that consumer want to associated with the
desire attached to a product while buying that product and this is explicitly shown in the option 4.
As the passage also suggests that consumers go by their own desires and hope for an altered state of
being involving an altered set of social relationships and the lines in the given question are also
converging to the same ideas as the key words are personalized narratives, metamorphic and
metonymic references etc to go against the dominant fashion norms in their social environment or
consumer culture at large. The one of prime reason for doing this is to put across the want for
recognition etc.
If the company launches the product
the cost of launching the product = 50 crores
there are 50% chance of success hence 50 % of 100 crore = 50 crore
and 50% of chance of unsuccessful sales = 50% of 20 crore = 10

30.

hence profit =- 60 50 = 10 crores


The product whose results are favorable has success rate = 80%
Cost of product = launching+ marketing =Total cost = 50 + 10 crores
Sales = successful + unsuccessful
= 80% * 100 + 20 % *20 = 84 crore
Profit = 84 60 = 24 crores.

31.

Probability of product failure =


failure probability when product is favorable result but fails + probability when unfavorable and fails
= 0.70*0.20 + 0.30*0.70 = 0.14 + 0.21 = 0.35

Bulls Eye

XAT PAPER 2014

www.hitbullseye.com

32.

31

Despite unfavorable :the profit earned by company can be =


Cost of product = launching+ marketing = 60 crores
Now the product fails in the test 30% chance of success is still there
sales = 30% of 100 crore ( successful ) + 70 % of 20 crore ( unsuccessful )
= 30 + 14 = 44 crores
hence the profit = 44 -60 = -16 crores

33.

34.

35.

36.

37.

38.

39.

40.

41.
42.

3
3

43.

44.

The prime motive is to maximize the prize money .Since maximum prize money is fixed for
International level. So Ms. Banerjee, keeping in mind the probability of winning the competition, will
pick up those students who have hit the cent percent score maximum number of times hence rising up
the probability of winning. Now, we can observe that Students 2 , 6 , 8 and 14 are the top four students
hitting maximum cent percent score 15 , 10 , 12 and 20 times respectively . So looking at the options
the team of top 3 is formed, i.e 2,8 and 14.
Now for district level , Since winning probability is on higher side . Ms. Banerjee will choose those
students who have maintained a good average while the tests. Observing the average we can see that
students 1,4,11 and 14 , All have hit the average of 70. So a team of three should be formed from the
above four mentioned. So looking up from the options students 1, 4 and 11 are selected.
- Now national team has to be selected after the selection of International team i.e 2, 8 and 14 are
already selected, Now the students who have hit the cent percent score maximum number of times
after the international team will be selected,. Students 1 ,3,6 and 12 have hit the cent percent 7,8,10
and 6 times respectively. So the team of top 3 is formed from the above mentioned, I.e 3,6 and 1 are
selected.
As the issue is of interpretation of work of the anthropologists and same uncertainty is there in
predicting the position of the electrons. Hence the answer is option 1.
The author assumes that money is a great motivator and to weaken we have to hit this assumption.
This is clearly given in the option 1.
The curve will be of L shape as after certain number of cups of coffee, as given in the passage, there
will not be any effect on the rate of risk of suicide.
As we know that by statements
F > A >G ; D > B / E ; I > C
The best position for girish is when all others than Faneesh and Ashok finish after girish
Hence the answer is option 3
As he has been his mentor in his childhood days and Balwant is generous by nature. Balwant is not so
active in is business and his son is not so interested in a family business. When he got admission in
MBA, Balwant was very happy. So it is highly likely that he should talk to Balwant regarding the
issue.
This seems to be the middle path as this is going to be win-win situation for all the parties concerned.
The first person to benefit from the Bigmart is Consumer because ethically it is correct. The consumer
should be benefitted from the firm.
The second are the farmers because they are dealing with them
The Third will be the retailers because they are competitors of Bigmart and they will be in least focus.
Ascending order becomesthe least profiting first
retailer
farmers
consumers
The steps which should not be taken by Ram is to produce quick quick
because: 1. Although the demand for quick-quick is increasing; but we dont know the percentage
composition of sales
2. There are already quality issues in the product , it should be solved first before producing it
Rocket Singh being head of sales should look for improving brand image which cannot be solved by 1.
Launch one more product 2. Stop production 3. Media awareness is good but finding the reason for
customers miffed with products by creating a research study for their dissatisfaction is more important
hence the answer should be option 5

Bulls Eye

XAT PAPER 2014


32

www.hitbullseye.com

45.

Best course of action for affected managers are to discuss the issues they face and resolve them ;
which is mentioned in option 3 only

46.

47.

48.

49.

50.

51.

52.

After the analyses of 3 reports; As the data is not published for other countries and the reports are selective on
publishing reports . hence the answer is option C
The only reason for seemingly divergent ranks because while google may be an important player in the adroid
market in the other countries apart from US
Option A is wrong because the Motorola could have paid at other countries also . Option B & C are wrong
because there is no mention of samsung in the report
option D is wrong as sales have not been talked about anywhere . thats why answer should be E
The reader makes the clear point that data on Samsung is ambiguous, since the company does not release data on
phones sold. This makes option 5 correct. There is no justification for options 1 to 4 in report 2. Hence, the correct
option is answer 5.
As per given in the question the recruitment of consultant is not fixable , hence it should be the last decision taken
by Ajay ; Hence the appropriate step of sequence of decision = 4 3 2 1 5
The best way to discuss the solutions in open house so that the right solution should emerge and emerge
democratically. hence the answer is option E
Doing nothing should not be taken whereas infuse fresh thinking by hiring outsiders should be the least concerned
; hence the best possible sequence should be B

53.

54.

55.

56.

57.

As all parameters are positive.


As the value function is multiple function in numerator and addition function in numerator
hence the value of S will increase with the increase of value
Moreover it can be further re checked by assuming values .
Grade A 90 ;
Whereas Grade B = 87 to 89 ;
If ramesh is scoring 70 instead of 97 = hence the change of 27 marks in one subject , creating a change of Grade
from A to B which mean a overall change in average by 3 i,e ( minimum of grade A grade B 90- 87)
Hence the number of Subjects should be = 27/ 3 = 9. Hence the answer is option D
ax3 + bx2 + cx + d, intersects at x = 1, x = -1 Hence at x = 1, -1, ax3 + bx2 + cx + d = 0
Put x = 1, a + b + c + d = 0 ------------(1)
Put x = -1, -a + b - c + d = 0 -----------(2)
Put y = 2 that means at x = 0, y = 2 but that is not the third root.
at x = 0; a (0)3 + b(0)2 + c(0) + d = 2
d = 2 ------(3)
Hence using (1), (2), (3)
2b + 4 =0
b = -2, hence the answer is option A.
As per the question the natural numbers are x , y ,x+y , x-y
Using statement 1 alone; As all the numbers are prime hence either x or y has to 2 because otherwise x+ y cannot
be prime
now if x is 2 then x- y is not possible as prime; hence y = 2 and numbers x -2, x, x+2 (substituting value of y) will
be in prime as well as in series, hence the only possibility will be 2 3 5 7.
Whereas using statement 2 answer cannot be find out.
Hence the answer is option A
ABC is a right angle triangle. D and E are mid points of AB and BC.

As it is not given that which angle is 900. So without it we cannot find the value of AC. Hence the statement III is
compulsory.
Whereas using statement I and statement II alone we cannot find the value of AC. But using all the three
statements. We can find value of AB2 + BC2; Hence the value of AC2 = AB2 + BC2 can be find. (By Pythagoras
theorem)

Bulls Eye

XAT PAPER 2014


33

www.hitbullseye.com
58.

P1

O1

O2

X
P2

Given that O1P1 = 3, O2P2 = 8, O1 X = 5 units.


In O1 X P1 P1 X
As O1P1 x =

59.

60.

61.

62.

O1 X 2 O1 P1 2

25 9 16 4

x P2 O 2 P2
8
B2
X P2 = 4
10.66

3
3
x P1 O1 P1

Total distance P1P2 XP2 + P1 10.66 + 4 14.66 units. Hence Answer is option C.
The numbers x , 17 , 3x-y2 2 , 3x + y2 -30 are in A.P ; and one of the term is odd i.e 17 . and increasing
sequence , Hence either the common difference is odd or even If common difference is odd , hence two terms are
odd and two terms will be even ; hence sum has to be even and should be divisible by 2
If common difference is even, hence all the terms are odd ;hence sum has to be even and should be divisible by 2
By Simple property of triangle sum of two sides should be > third side and the difference of two sides is < third
side. In triangle PQR; QS should be 17 - 5 > 12. In triangle QRS QS should be < 9+4 = 14
Hence answer is option B
x + 7 + x 8 = 16
Taking + ve and ve of values of mode = 16. 2x -1 = 16 ; x = +8.5
Taking ve values = -2x +1 = 16 ; X = - 7.5
Sum of possible values of x = 8.5 7.5 = 1

Let length of Ladder is PA = 30m. PQ 26m. Hence the Length of QA Distance of ladder boot from wall
In PQA Right Angled at Q.

30 2 26 2 900 676 224 is 15m.


QA
1 15
0
Hence Angle PAQ Cos-1
cos 60
30
PA
1
PBQ = PAQ = 300
2
QA =

As Length of PB = PA = 30m.
Length of QB = PB cos 300 = 30

63.

3
25.98
2

Hence Length of AB = 25.98 - 15 10.98 11.


Let the smallest initial Number = x.
Let sequence of operation Represented by AM (Amitabh) and (Sashi)
Step operation
AM = 2x
SA = 2x + 50
AM = 4x + 100
SA = 4x + 150
AM = 8x + 300
SA = 8x + 350
AM = 16x + 700
SA = 16x + 750 Hence SA should lose that means.
16x + 750 > 999

Bulls Eye

XAT PAPER 2014


34

www.hitbullseye.com

16x + 750 > 999 (for lowest possible)


16x + 750 > 249
x>
64.

249
taking minimum possible. x > 15. Hence x = 16 And sum of digit should be 7
16

3a 2 2

3
R
A

The figure of the question will be as shown above S1 is the outer square of area 9 units. Hence its side will be 3
units S2 is the inner square of area 8 units. Hence its side is
In PDS
b2 + a2 = 8
b2 + (3 b2 = 8
b2 + 9 + b2 6b = 8
2b2 6b + 1 = 0

2 2 units.

6 36 8 6 28 3 7
=

4
4
2
3 7 63 7 3 7
a = 3 b = 3

2
2
2
3 7
b
2 3 7 15.9 Hence option D is the answer.

a 3 7 3 7
b=

2
65.

13
A

8
17

15
5
01

C
5 mm

The total volume discharged by the pipe = volume of Frustum Formed


As it is given that point of contact is 17cm alongside and 15cm from the base. Hence AB

17 2 15 2 64 8cm
If AB = 8 then OA = O1C = 13-8 = 5cm
Volume of Frustum = volume discharged

Bulls Eye

XAT PAPER 2014


35

www.hitbullseye.com

1
259
5
M(132+52+135) = M
= 5.1 minutes.
Minute for discharge (T). T mins. =
3
50
100
66.

Probability of being both red = 5/16


That means both Bags give red balls.
Probability of being both red = Probability of red from 1st Probability of red from 2nd

5 5 1
only possible as probability cannot be 1 or > 1 (as given in the question that each bag has
16 8 2
marbles of both colors)

5
probability of red ball from 1st bag
8
Probability of Blue Ball from 1st bag =

3 3

8 8

Similarly probability of red ball from 2nd bag =

1 1

2 2
1 3 3
Hence probability of both blue
2 8 16

probability of blue ball from 2nd bag =

67.

x
1

n 1 n
13!
1 1 1
1
x

Expanding
1 2 3
13 13!
13! 13!
13! x
13!

2
3
13 13!
13

When x is divided by 11

13! 13!
13!

2
3
13
That means
11
13!
All quantity have multiple of 11 except
11
13!
13!
13!

So
11 2 11
13 11
13!

All will give remainder O except the term =


Remainder of x divided by 11

13!
11 11!

1 2 3 8 9 10 12 13
11

10!12 13 10 1 2
2

9 is the remainder
11
11
11
68.

2.6

2.6
2.6
2.6

2.6

2.6

48

20

Bulls Eye

XAT PAPER 2014


36

www.hitbullseye.com

As the for each 2.6m walk; 1m deep step is there the number of total steps =

48
19 approx.
2.6

That means steps will be in depth of 1m, 2m, 3m, ----19m.


Volume of tank (Depth) Length Width.
Length of each step = 2.6, width = 20m, Depth = (1-----19)
2.620(1+2+3+4---19)

2.6 20 19 20
approx 10600 = volume of water in tank
2
100
1
1
1
1
10

t 2 log;100!
log 2 100! log 3 100! log 4 100!
log100 100!
1
log a b (As we know)
log b a

69.

70.

log100!2 +log100!3 + log100!4------log100! 100


Also loga b + loga c loga b c
Hence log100! 2345-----100
log100!100! = 1.
Multiple of 1023 which and divisor of 1029
Number of factor of 1029 229 529 = 30 30 = 900
Factor of 1029 multiple of 1023 1029 = 1023 26 56 7 7 = 49
Hence

49
a2
7

a/b
2
900
30
b

Hence b a = 30 7 = 23.

71.

Diameter PQ will be 6 units. Y coordinates will be same along PQ as line is parallel to x axis. There will be a gap
of 6 units in x coordinate of P and Q.
Equating the y coordinate in both the equations we get
ax = 2ax+6

1 a x 6
x
2
a
1
a6 =
2
1

a=
72.

Let the breadth be 3x and the length be y


3xy = 90 xy = 30

A
x
P
x
2
x
2

W
Q
x
B

3x
2

E
T

V
F

3x
2

Bulls Eye

XAT PAPER 2014


37

www.hitbullseye.com

V is midpoint of WR PW||EV EV =

PW
2

WQ
PQ x
EF =

2
2
2
Now MPA MEV
Height of MPA with respect WAP: Height (h1) of MEV with respect to EV
x
= AP : EV = x : = 4:1
2
Let the height of MPA = 4K and height (h1) of MEV= K.
y
y
4K + K = 5K = K =
2
10
y
Height (h1) of MEV =
10
1
1 x y
30
Area (MEV) = EV h1 =

0.375
2
2 4 10 80
Similarly VFN CRN
Height (h2) of VFN with respect to VF: Height of CRN with respect of CR
x 3x
= VF : CR = :
1: 6
4 2
Let height (h2) of VFN = m and height of CRN = 6m
y
m+6m = 7m =
2
y
m=
14
y
Height (h2) of VFN =
14
1
1 x y
30
Area (VFN) = FV h2 =

0.27
2
2 4 14 1 / 2
y 1
x y 3xy 90
1
Area ( PQEF) = (PQ + EF) x

11.25 sq. units


2 2
2 2
8
8
2

Similarly FV =

73.

A ( PQMN) = A ( PQEF) Area ( MEV) + Area ( VFN) = 11.25-0.375+0.27 11.145


Hence answer is option 4.
792B 297B belong to base B number system.
If 297B is the factor of 792B
297B 2B2 + 9B+7
792B 7B2 + 9B + 2

7 B 2 9B 2
must be integer.
2B 2 9B 7
5B 2 5
1
2B 2 9B 7

74.

75.

76.

Hence (5B2 5) = (2B2 + 9B + 7) K must be the factor.


Put K = 2
5B2 5 = 4B2 + 18B + 14
B2 18B 19
(B 19) (B + 1) = 0
B = 19
Let x, y, z, the number of students getting 6, 8 and 20 marks respectively
According to question 6x + 8y + 20z = 504 ---(1) and y = x + 2z -----(2)
(2) can be written as x = y 2z 6x = 6y 12z ---(3)
Put (3) in (2) we get 6y 12z + 8y + 20z = 504
14y + 8z = 504 Further solving only y = 32, z = 7 and x = 18 satisfies above equation.
total number of students = x + y + z = 18 + 32 + 7 = 57.
As seen from the sixth column of the first graph given, the percentage after USA in almost all the years happens to
be in case of India, hence that is the answer.
In case of United states in the year 2002-03, there is a small decline. Brazil in the year 2006-07 is almost equal, so
no decrease. India in the year 2009-10 is again very close to each other, hardly any decline. In case of Malaysia in
the year 2008-09 the bar graph is decreasing significantly proportionately. It seems between 20- 25%. In China in
the year 2008-09 the decline is close to 10%. Hence it is Malaysia in the year 2008-09.

Bulls Eye

XAT PAPER 2014


38

www.hitbullseye.com

77.

78.

79.

80.

81.

82.

83.

In this case, one by one all the countries are to be seen. In the question it is clearly stated that out of the following
countries, he have to answer, and that becomes India. Overall it seems that China would give the highest such
increase. Hence answer is option B.
Just have a look at all the graphs and try n find the lowest value, it happens to be in case of C & E Europe, having
a look at the options, it is given in case of 4th option. Then checking all the remaining countries given, all are in
the ascending order only. Hence it is the answer.
Just have a look at slopes of all the graphs ALONG WITH the differences of gridlines given. In case of C and E
Europe the slope is quite increasing and the difference of grid lines is also very high i.e. starting from 5 it goes to
around 20. As this is the highest among all, this is the answer. Hence option B.
Maximum GDP of African region is 9.5 also Maximum GDP of south American region is 9.5. Hence none of
these is the answer.
The question states to find where the first increase in percentage is higher than the second increase. The year 2005
is given in four options out of 5, that should be our starting point. Finding it actually is higher the answer 4th
option is rejected. Checking for 2004 after that, and finding it wrong, the possible options left are B and C only.
Checking for 2006, which is wrong. You can mark the answer as option C.
Finding the values of $ in Rs. terms for all the years given, it happens to be as follows. Year 2001 = 45.16, 2002
= 48.5, 2007 = 4.13, 2010 = 45.7, 2011 = 46.8. Now as in year 48.5 Rs. Is equivalent to 1 dollar, it is cheapest in
that year.

6170
16564
1783 and R for the year 2011 is
2633 .
3.46
6.29
360
As total Pie is divided in 15 years and the average sector becomes
24 0 . Also R increase linearly so angle
315

R for the year 2004 is

for the middle year i.e. 2004 is 240. Now equating the middle year with average sector degrees. Angle for 2011 i.e.

year of maximum angle is

84.
85.
86.
87.
88.
89.
90.
91.
92.
93.
94.
95.
96.
97.
98.
99.
100.
101.
102.
103.

4
3
2
4
1
1
4
2
1
5
2
3
1
3
5
4
4
4
3
2

2633
24 0 36 0 . Hence the answer.
1783

Bulls Eye

XAT 2015

www.hitbullseye.com

SECTION I
Verbal and Logical Ability
1.

Number of questions: 28

The first and the last sentences of the paragraph are numbered 1 & 6. The others, labelled as P, Q, R
and S, are given below:
1. The world of cinema is indeed a strange one and baffles many a critic.
P. But there are incorrigible optimists who see a bright future.
Q. The Pundits still predict doom and they insist that it is the end of the road for cinema.
R. At the temples of the box office, fortunes are made and unmade.
S. The world of cinema has, they say, its own attraction.
6. Perhaps a positive outlook is not unwarranted. A doomsday approach is far too fatal at this stage.
Which of the following combinations given below is the most logically ordered?
1. 1SQPR6

2.

3. 1RQPS6

4. 1QSPR6

5. 1QPSR6

4. childlike

5. awful

Which of the followings is not a term of 'disapproval'?


1. infantile

3.

2. 1RSPQ6

2. charlatan

3. imbecile

Read the following sentences and choose the option that best arranges them in a logical order.
I. It is certainly true that the criticsthose persons whom the dictionary describes as "skilled in judging
the qualities or merits of some class of things, especially of literary or artistic work" -have long
harboured murderous thoughts about the conditions of our drama, but their ineffectuality as public
executioners is legendary.
II. But not close enough, it would seem, for this "marriage" constitutes the case of an absolute desire
encountering a relative compliance.
III. The reviewers, by contrast, come close to being the most loyal and effective allies the commercial
theatre could possibly desire.
IV. Perhaps the greatest irony in a situation bursting with ironies is the reiterated idea that the
critics are killing the theatre.
V. We all know that when theatre people or members of the public refer to the critics, they nearly
always mean the reviewers.
I. V, IV, III, II, I

4.

2. IV, V, I, III, II

3. IV, I, V, II, III

4. II, V, IV, I, III

5. I, IV, V, II, III

In the traditions of many religions throughout the world (including Judaeo-Christian beliefs), there has
long been a sustained belief that the Universe as we know it today did not exist forever in the past, and
that there was a spontaneous act which gave birth to all that has been, all that is, and all that will be. In
other words, the Universe itself has not been eternal as our senses might indicate at first glance,
Which of the following options can meaningfully complete the above sentence?
1. but has a limited lifespan after its creation.
2. but our senses give us the right knowledge.
3. however, on second glance, our religious beliefs are right.
4. however, it is a ball of intense energy.
5. however, it could not have been created.

5.

Bulls Eye

XAT 2015

www.hitbullseye.com

It is a curious historical fact that modern quantum mechanics began with two quite different
mathematical formulations: the differential equation of Schroedinger, and the matrix algebra of
Heisenberg. The two, apparently dissimilar, approaches were proved to be mathematically equivalent.
Which of the following sentences would most meaningfully follow the above paragraph?
1. The two approaches did not start with the same mathematical formulations.
2. These two points of view were destined to complement one another and were ultimately synthesized
in Dirac's transformation theory.
3. A third mathematical formulation given by Feynman combines the matrix algebra of Heisenberg and
Integral calculus of Leibniz.
4. Quantum mechanics evolved in the twentieth century and came very close to particle physics,
especially after the CERN experiments in Switzerland.
5. Earlier, the two formulations were mathematically similar.

6.

Ranu is an ordinary sportsperson. In the last two university sprint events, her performances in the heats
were pathetic.
Which of the followings, if true, weakens the above argument the most?
1. She had participated in the college swimming competition and finished last.
2. She is a national shot-put champion.
3. The last two times, Ranu had to compete with national level runners. Had she been in other
heats, she would have reached quarterfinals.
4. Ranu was the only player who represented her college in the sprint events.
5. In the college sprint events, Ranu always won.

7.

Six words are given below:


I. Cacophonic
II. Cacographic
III. Calamitous
IV. Catastrophic
V. Contraindicative VI. Cataclysmic
Which of the above words have similar meanings?
1. IV & VI only
4. III, IV & VI only

8.

2. I, II & V only
5. III, IV, V & VI only

3. II, V & VI only

Read the four sentences given below:


I. He is the most ______ of the speakers to address us today.
II. The belief in ________ justice is the essence of his talk.
III. This hall would have been full but for the ______ rain.
IV. Many in the audience have achieved ______ in their respective fields.
Which of the following sequence of words would most appropriately fit the blanks?
1. I. Eminent, II. Imminent, III. Immanent, IV. Eminence
2. I. Immanent, II. Imminent, III. Imminence, IV. Eminence
3. I. Eminent, II. Immanent, III. Imminent, IV. Eminence
4. I. Eminent, II. Immanent, III. Imminent, IV. Imminence
5. I. Immanent, II. Imminence, III. Eminent, IV. Eminence

Bulls Eye

XAT 2015

www.hitbullseye.com

9.

In the following pages, I shall demonstrate that there is a psychological technique which makes it
possible to interpret dreams, and that on the application of this technique, every dream will reveal itself
as a psychological structure, full of significance, and one which may be assigned a specific place in the
psychic activities of the waking state. Further, I shall endeavour to elucidate the processes which
underlie the strangeness and obscurity of dreams, and to deduce from these processes the nature of the
psychic forces whose conflict or cooperation is responsible for our dreams. This done, my investigation
will terminate, as it will have reached the point where the problem of the dream merges into more
comprehensive problems, and to solve these we must have recourse to material of a different kind.
Which of the followings would be closest to the ideas expressed in the first two sentences of the above
passage?
1. Overt causes can have only overt effects.
2. Overt causes have only covert effects.
3. Covert effects have only covert causes.
4. You can't judge a book by its cover.
5. Overt effects can have covert causes.

10.

Identify the correct sequence of words that aptly fit the blanks in the following passage.
It is ______(i) ________ that the accused had _________ (ii) ___________ (iii) __________ from all
criminal activities by adopting the ___________ (iv) ________ of a sanyasi. However, despite repeated
requests from the counsel for prosecution, the court has ______(v) ____ a lie detector to ascertain the
truth.
1. (i) inferred, (ii) feigned, (iii) separation, (iv) deportment, (v) prescribed
2. (i) inferred, (ii) forged, (iii) palling, (iv) deportment, (v) proscribed
3. (i) implied, (ii) faked, (iii) separation, (iv) demeanour, (v) proscribed
4. (i) implied, (ii) feigned, (iii) separation, (iv) demeanour, (v) prescribed
5. (i) inferred, (ii) faked, (iii) cessation, (iv) deportment, (v) proscribed

DIRECTIONS for question 11: Read the following statements carefully:


11.

Statement 1: If you want to understand the causes that existed in the past, look at the results as they are
manifested in the present.
Statement 2: Murali did not work as hard as his friends but had secured Ist rank in the examination.
Which of the following options is correct with respect to the above two statements?
1. If Statement 2 is right, Statement 1 is invalid.
2. Statement 1 and Statement 2 are contradictory to each other.
3. Statement 2 supplements Statement 1.
4. Statement 2 is a rare occurrence and hence irrelevant.
5. Statement 1 will hold true even if Statement 2 is valid.

DIRECTIONS for question 12 to 15: Analyse the following passage and provide appropriate answers for
questions that follow.
Alone - he was alone again - again condemned to silence - again face to face with nothingness! Alone! - never
again to see the face, never again to hear the voice of the only human being who united him to earth! Was not
Faria's fate the better, after all - to solve the problem of life at its source, even at the risk of horrible suffering?
The idea of suicide, which his friend had driven away and kept away by his cheerful presence, now hovered like
a phantom over the abbe's dead body.

Bulls Eye

XAT 2015

www.hitbullseye.com

"If I could die," he said, "I should go where he goes, and should assuredly find him again. But how to die? It is
very easy," he went on with a smile; "I will remain here, rush on the first person that opens the door, strangle
him, and then they will guillotine me." But excessive grief is like a storm at sea, where the frail bark is tossed
from the depths to the top of the wave. Dantes recoiled from the idea of so infamous a death, and passed
suddenly from despair to an ardent desire for life and liberty.
"Die? oh, no," he exclaimed - "not die now, after having lived and suffered so long and so much! Die? yes, had
I died years ago; but now to die would be, indeed, to give way to the sarcasm of destiny. No, I want to live; I
shall struggle to the very last; I will yet win back the happiness of which I have been deprived. Before I die I
must not forget that I have my executioners to punish, and perhaps, too, who knows, some friends to reward.
Yet they will forget me here, and 1 shall die in my dungeon like Faria." As he said this, he became silent and
gazed straight before him like one overwhelmed with a strange and amazing thought. Suddenly he arose, lifted
his hand to his brow as if his brain were giddy, paced twice or thrice round the dungeon, and then paused
abruptly by the bed.
"Just God!" he muttered, "whence comes this thought? Is it from thee? Since none but the dead pass freely from
this dungeon, let me take the place of the dead!" Without giving himself time to reconsider his decision, and,
indeed, that he might not allow his thoughts to be distracted from his desperate resolution, he bent over the
appalling shroud, opened it with the knife which Faria had made, drew the corpse from the sack, and bore it
along the tunnel to his own chamber, laid it on his couch, tied around its head the rag he wore at night around
his own, covered it with his counterpane, once again kissed the ice-cold brow, and tried vainly to close the
resisting eyes, which glared horribly, turned the head towards the wall, so that the jailer might, when he brought
the evening meal, believe that he was asleep, as was his frequent custom; entered the tunnel again, drew the bed
against the wall, returned to the other cell, took from the hiding-place the needle and thread, flung off his rags,
that they might feel only naked flesh beneath the coarse canvas, and getting inside the sack, placed himself in
the posture in which the dead body had been laid, and sewed up the mouth of the sack from the inside.
12.

How was the protagonist planning to resolve his problem?


1. To give up and surrender.
2. To commit suicide in the dungeon.
3. To fight the jailor and escape.
4. To kill those who came to carry the corpse.
5. To exchange places with the dead.

13.

Which one of the following options is nearest in meaning to that implied by the phrase 'sarcasm of
destiny' in this passage?
1. Destiny makes one a laughing stock.
2. Destiny ultimately asserts itself.
3. Triumph of the struggles gone through.
4. A mockery of the forces of destiny.
5. Let the enemy have the last laugh.

14.

Among the options given below, which phrase specifically captures the change of mood of the
protagonist?
1. To be or not to be
3. Depression to daring
5. Loathing to yearning

2. Despair and hope


4. Darkness to light

Bulls Eye

XAT 2015

www.hitbullseye.com

15.
Words
I. Counterpane
II. Dungeon
III. Guillotine
IV. Shroud

Related Words
A. Burial
B. Bed
C. Execution
D. Cell

Which of the above 'related words' on the right-hand side are correctly matched with 'words' on the lefthand side?
1. i-b, ii-d, iii-c, iv-a
4. i-d, ii-b, iii-a, iv-c

2. i-a, ii-d, iii-b, iv-c


5. i-b, ii-a, iii-c, iv-d

3. i-a, ii-d, iii-c, iv-b

DIRECTIONS for question 16 to 20: Analyse the following passage and provide appropriate answers for
questions follow.
The understanding that the brain has areas of specialization has brought with it the tendency to teach in ways
that reflect these specialized functions. For example, research concerning the specialized functions of the left
and right hemispheres has led to left and right hemisphere teaching. Recent research suggests that such an
approach neither reflects how the brain learns, nor how it functions once learning has occurred. To the contrary,
in most 'higher vertebrates' brain systems interact together as a whole brain with the external world. Learning is
about making connections within the brain and between the brain and the outside world.
What does this mean? Until recently, the idea that the neural basis for learning resided in connections between
neurons remained a speculation. Now, there is direct evidence that when learning occurs, neuro-chemical
communication between neurons is facilitated, and less input is required to activate established connections
over time. This evidence also indicates that learning creates connections between not only adjacent neurons but
also between distant neurons, and that connections are made from simple circuits to complex ones and from
complex circuits to simple ones.
As connections are formed among adjacent neurons to form circuits, connections also begin to form with
neurons in other regions of the brain that are associated with visual, tactile, and even olfactory information
related to the sound of the word. Meaning is attributed to 'sounds of words' because of these connections. Some
of the brain sites for these other neurons are far from the neural circuits that correspond to the component
sounds of the words; they include sites in other areas of the left hemisphere and even sites in the right
hemisphere. The whole complex of interconnected neurons that are activated by the word is called a neural
network.
In early stages of learning, neural circuits are activated piecemeal, incompletely, and weakly. It is like getting a
glimpse of a partially exposed and blurry picture. With more experience, practice, and exposure, the picture
becomes clearer and more detailed. As the exposure is repeated, less input is needed to activate the entire
network. With time, activation and recognition become relatively automatic, and the learner can direct her
attention to other parts of the task. This also explains why learning takes time. Time is needed to establish new
neural networks and connections between networks. This suggests that the neural mechanism for learning is
essentially the same as the products of learning. Learning is a process that establishes new connections among
networks. The newly acquired skills or knowledge are nothing but formation of neural circuits and networks.
16.

It can be inferred that, for a nursery student, learning will...


1. Comprise piecemeal ideas and disconnected concepts.
2. be a pleasant experience due to the formation of improved connections among neurons.
3. lead to complex behaviour due to formation of new connections among neurons.
4. be better if discrete subjects are taught rather than a mix of subjects.
5. be a happy experience.

17.

Bulls Eye

XAT 2015

www.hitbullseye.com

Read the following statements and answer the question that follows.
I. The two hemispheres of the brain are responsible for learning autonomously.
II. Simultaneous activation of circuits can take place in different areas of the brain.
III. There are specific regions of the brain associated with sight, touch and smell.
IV. The brain receives inputs from multiple external sources.
V. Learning is not the result of connections between neurons.
Which of the above statements are consistent with ideas expressed in the passage?
1. I, V

18.

2. II, III

3. III, V

4. IV, V

5. I, II, III

Which of the following proverbs best describes the passage?


1. When student is ready, the master appears.
2. Child is the father of the man.
3. All's well that ends well.
4. You can't teach old dog new tricks.
5. Many a mickle makes a muckle.

19.

A father and son aged 60 and 25 respectively, have been learning paragliding for quite some time.
Based on the passage above, which of the following would be true?
1. The son would always learn more.
2. The father might learn more, if both of them started at the same time.
3. The son would learn more, if both of them started at the same time.
4. If both of them have been learning since the age of 15, the son would learn more.
5. Both of them would always progress equally.

DIRECTIONS for question 20 to 23: Analyse the following passage and provide appropriate answers for
questions follow.
Certain variants of key behavioural genes, "risk allele" make people more vulnerable to certain mood,
psychiatric, or personality disorders. An allele is any of the variants of a gene that takes more than one form. A
risk allele, then, is simply a gene variant that increases your likelihood of developing a problem.
Researchers have identified a dozen-odd gene variants that can increase a person's susceptibility to depression,
anxiety, and antisocial, sociopathic, or violent behaviours, and other problems- if, and only if, the person
carrying the variant suffers a traumatic or stressful childhood or faces particularly trying experiences later in
life. This hypothesis, often called the "stress diathesis" or "genetic vulnerability" model, has come to saturate
psychiatry and behavioural science.
Recently, however, an alternate hypothesis has emerged from this one and is turning it inside out. This new
model suggests that it's a mistake to understand these "risk" genes only as liabilities. According to this new
thinking, these 'bad genes' can create dysfunction in unfavourable contextsbut they can also enhance function
in favourable contexts. The genetic sensitivities to negative experience that the vulnerability hypothesis has
identified, it follows, are just the downside of a bigger phenomenon: a heightened genetic sensitivity to all
experience.
This hypothesis has been anticipated by Swedish folk wisdom which has long spoken of "dandelion" children.
These dandelion children - equivalent to our "normal" or "healthy" children, with "resilient" genesdo pretty
well almost anywhere, whether raised in the equivalent of a sidewalk crack or a well-tended garden. There are
also "orchid" children, who will wilt if ignored or maltreated but bloom spectacularly with greenhouse care.
According to this orchid hypothesis, risk becomes possibility; vulnerability becomes plasticity and

Bulls Eye

XAT 2015

www.hitbullseye.com

responsiveness. Gene variants generally considered misfortunes can instead now be understood as highly
leveraged evolutionary bets, with both high risks and high potential rewards.
In this view, having both dandelion and orchid kids greatly raises a family's (and a species') chance of
succeeding, over time and in any given environment. The behavioural diversity provided by these two different
types of temperament also supplies precisely what a smart, strong species needs if it is to spread across and
dominate a changing world. The many dandelions in a population provide an underlying stability. The lessnumerous orchids, meanwhile, may falter in some environments but can excel in those that suit them. And even
when they lead troubled early lives, some of the resulting heightened responses to adversity that can be
problematic in everyday life - increased novelty-seeking, restlessness of attention, elevated risk-taking, or
aggression - can prove advantageous in certain challenging situations: wars, social strife of many kinds, and
migrations to new environments. Together, the steady dandelions and the mercurial orchids offer an adaptive
flexibility that neither can provide alone. Together, they open a path to otherwise unreachable individual and
collective achievements.
20.

The passage suggests 'orchids':


1. are insufficient in number.
3. end up weaker as compared to dandelions.
5. are always too delicate to survive.

21.

2. are limited to greenhouses.


4. thrive in anaesthetised conditions.

Which of the following statements correctly echoes the author's view?


1. Persons carrying risk allele end up being self-destructive and antisocial.
2. Orchids possess humankind's phenomenal adaptability and evolutionary success.
3. With a bad environment and poor parenting, all children will have a normal life.
4. Children born with genetic vulnerability need not necessarily be sociopaths.
5. Genes not only makes you sensitive to disorders; but are also responsible for failures of societies.

22.

The word 'diathesis' means:


1. susceptible to disease
3. connected with two kidneys
5. living in two different environments

23.

2. two-pronged hypothesis
4. missing parts of the body

Mr. Good and Mr. Evil were batch-mates during the college. Five years after graduating, Mr. Evil was
put behind bars for financial fraud while Mr. Good was running a successful NGO, working for
orphans. Mr. Good was raised in a protective environment while Mr. Evil was a self-made man.
Based on the above information, which of the following statements is definitely correct?
1. It can be concluded that Mr. Evil is a 'dandelion', but nothing can be concluded about Mr.Good.
2. It can be concluded that Mr. Evil is an 'orchid', but nothing can be concluded about Mr. Good.
3. It can be concluded that Mr. Good is a 'dandelion', but nothing can tie concluded about Mr.Evil.
4. It can be concluded that both Mr. Good and Mr. Evil are 'orchid'.
5. It is not possible to conclude about 'children typology' of the two batch mates.

Bulls Eye

XAT 2015

www.hitbullseye.com

DIRECTIONS for question 24 to 26: Analyse the following passage and provide appropriate answers for
questions that follow.
For private goods, competitive markets ensure efficiency despite the decentralized nature of the information
about individual's tastes and firms' technologies. Implicitly, market competition solves adverse selection
problems and the fixed-price contracts associated with exogenous prices solve moral hazard problems.
However, markets fail for pure public goods and public intervention is thus needed. In this case, the
mechanisms used for those collective decisions must solve the incentive problem of acquiring the private
information that agents have about their references for public goods. Voting mechanisms are particular
incentive mechanisms without any monetary transfers for which the same question of strategic voting, i.e., not
voting according to the true preferences, can be raised. For private goods, increasing returns to scale create a
situation of natural monopoly far away from the world of competitive markets. When the monopoly has private
information about its cost or demand, its regulation by a regulatory commission becomes a principal-agent
problem.
Note: Public goods are those in which individuals cannot be excluded from use and where use by one individual
does not reduce availability to others, while an individual can be excluded in case of private goods.)
24.

For which of the following goods, can markets not be efficient?


1. Packaged water
4. Petrol

25.

2. Electricity supply at home


5. All of the above

3. Air

Which of the following cannot be concluded from the above paragraph?


1. Public intervention is the panacea when market fails.
2. Adverse selection problems as well as moral hazard problems may not arise in competitive markets.
3. Strategic voting is nothing but a non-monetary incentive mechanism.
4. Lack of access to private information regarding preferences of agent leads to incentive problem.
5. Public regulations may address problems associated with natural monopoly.

26.

Read the following statements carefully:


Statement 1: In India factories dump their waste in the nearby water bodies.
Statement 2: Government is thinking of granting tax benefits to factories which adopt eco-friendly
practices.
Which of the following options best captures the relationship between Statement 1 and Statement 2?
1. Statement 1 is an example of market failure and Statement 2 corroborates Statement 1.
2. Statement 1 is an example of 'adverse selection problem' and Statement 2 is an example of
'moral hazard problem'.
3. Statement 1 is an example of market failure while Statement 2 suggests one way of reducing the
problem.
4. Statement 1 is an example of public good and Statement 2 is an example of private good.
5. In Statement 1 the principal is 'factory' and in Statement 2 the principal is 'government'.

Bulls Eye

XAT 2015

www.hitbullseye.com

DIRECTIONS for question 27 to 30: Analyse the following passage and provide appropriate answers for
questions that follow.
Creative thinking can be used by management teams to produce actions that will potentially increase innovation
and identify opportunities. Brainstorming is one technique that can enhance creativity. Brainstorming is usually
regarded as a method to be used with groups of people. Although, it can be employed with individuals, the
benefit of involving a group is that one person's idea can help to stimulate even more ideas by other group
members.
Underlying brainstorming is the idea that people's creativity is restricted because they tend to reject ideas at too
early a stage. This can be because they may be imposing imaginary constraints on a problem or making false
assumptions. Alternatively, they may be unable to see a problem from multiple perspectives or they may be
stereotyping problems and possible solutions and hence failing to see their wider potential. Involvement of
people with different perspectives enriches the idea generation.
27.

Pick the option that best captures the relationship between the two paragraphs above.
1. The first paragraph describes a technique and the second is an example that supports it.
2. The first paragraph describes a process and the second paragraph contradicts the description.
3. The first paragraph describes a technique. The first part of the second paragraph contradicts it
and the second part of the second paragraph makes untested claims.
4. In the first paragraph, the author conveys the understanding of a subject and in the second paragraph
the author complements the first.
5. In the first paragraph the author describes a technique and in the second paragraph the author
provides explanation of its advantages.

28.

Which of the following options would be closest to the main argument in the second paragraph above?
1. Viewing students as customers, future alumni, brand ambassadors, potential recruiters etc., would
make engineering colleges more successful.
2. Good students, after completing (he MBA, should play multiple roles in an organization to become
successful leaders.
3. India does better in team sports like cricket than in individual sports like swimming.
4. All departments of the organization, including marketing, should give inputs to generate new ideas
for improving customer satisfaction.
5. Compared to small entrepreneurial firms, large organizations will definitely generate more ideas.

10

Bulls Eye

XAT 2015

www.hitbullseye.com

SECTION II
Verbal and Logical Ability

Number of questions: 23

DIRECTIONS for question 29 to 31: Answer the basis of information given in the following case.
Mr. Dipangshu Barua, a young IT professional, came early to office to assist his boss in the preparation for an
important client presentation. When he switched on his computer, he saw an email from Mr. Patel. The email
was as follows:
January 2, 2015
Dear Mr. Barua,
This email serves as a follow-up of my conversation with you on December 1, 2014. I have already conveyed
need for improvement in your behaviour as desired by your project leader and colleagues. They are yet to notice
any visible improvements. I am apprehensive that your failure to act may warrant further action leading to
dismissal. I will continue to monitor and assess your performance over the next three months to determine
whether improvements meet the expectations. At the same time, I would like to re-affirm that you are very
valuable for our organization.
Best Wishes,
Mr. A. Patel
HR Director
29.

Initially, the e-mail distracted Dipangshu but he decided to focus on the job. Which of the following
options might best explain his decision to do so?
1. Mr. Patel would soon be transferred to another department.
2. Last week, Mr. Dipangshu has been assigned to a new team in the same project.
3. Three days back, Mr. Dipangshu has been assigned a new project similar to his final year
engineering project.
4. His friend has been hospitalized for the last three months.
5. Failing to perform in the client meeting might further complicate things.

30.

The scheduled presentation went off smoothly. Back in his cabin, Dipangshu read Mr Patel's e-mail
once more and pondered over it. During the last meeting he tried hard to put forward his explanation
but Mr. Patel had not allowed him to speak. Dipangshu was thinking of meeting Mr. Patel once again
but was doubtful whether that would help. Incidentally, he had a job offer from a start-up with a
comparable salary. If Dipangshu was to join the new job, he had to accept the offer within the next two
weeks. However, he cannot think of a life without a job. Dipangshu was confused!
Which of the following options would be the best move for Dipangshu?
1. Talk to Mr. Patel and highlight the initiatives he has taken but at the same time start applying for
other jobs.
2. Reject the offer from the start-up. Use the next three months to find a better job, but continue in
the present job.
3. Resign from this organization right now.
4. Accept the offer, only if the start-up gives a salary hike, else keep prospecting.
5. Accept the offer with a request to give him a 10% salary hike

Bulls Eye

XAT 2015

11

www.hitbullseye.com

31.

After a couple of weeks, Mr. Patel came to know that Dipangshu's project leader Mr. John, a very
competent senior executive, may have wilfully influenced his team members to file a wrong
complaint against Dipangshu. Mr. John may have done it because Dipangshu has refused to tow
John's line. Mr. Patel also came to know that Dipangshu was thinking of quitting this job. He felt
regretful about his letter to Dipangshu. He wanted to resolve the complicated situation. He was
contemplating following five actions in his mind.
I. Talk to Mr. John about Dipangshu and convey to him that losing a bright employee would cost the
organization dearly.
II. Catch up with Mr. John during coffee break and convey that Dipangshu has a very good track
record.
III. Chat with Dipangshu during coffee break.
IV. Catch up with Dipangshu during coffee break and convey that the organization values him.
V. Arrange a meeting among Mr. John, Dipangshu and himself to sort out the differences.
Which of the following is the best sequence of actions for resolving the problem?
1. I, III, V

2. II, III, V

3. I, II, IV

4. I, IV, V

5. III, IV, V

DIRECTIONS for question 32 to 33: Answer questions on the basis of information given in the following
case.
MBA entrance examination comprises two types of problems: formula-based problems and application-based
problems. From the analysis of past data, Interesting School of Management (ISM) observes that students good
at solving application-based problems are entrepreneurial in nature. Coaching institutes for MBA entrance
exams train them to spot formula-based problems and answer them correctly, so as to obtain the required
overall cut-off percentile. Thus students, in general, shy away from application-based problems and even those
with entrepreneurial mind-set target formula-based problems.
Half of a mark is deducted for every wrong answer.
32.

ISM wants more students with entrepreneurial mind-set in the next batch. To achieve this, ISM is
considering following proposals:
I. Preparing a question paper of two parts, Part A and Part B of duration of one hour each. Part A and
Part B would consist of formula-based problems and application- based problems, respectively. After
taking away Part A, Part B would be distributed. The qualifying cut-off percentile would be
calculated on the combined scores of two parts.
II. Preparing a question paper comprising Part A and Part B. While Part A would comprise formulabased problems, part B would comprise application-based problems, each having a separate
qualifying cut-off percentile.
III. Assigning one mark for formula-based problems and two marks for application-based problems as
an incentive for attempting application-based problems.
IV. Allotting one mark for formula-based problems and three marks for application- based problems,
without mentioning this in the question paper.
Which of the following proposal (or combination of proposals) is likely to identify students with best
entrepreneurial mind-set?
1. II

2. I & II

3. I & III

4. II & III

5. II & IV

12

33.

Bulls Eye

XAT 2015

www.hitbullseye.com

ISM conducts a common entrance examination every year. This year, the question paper would
comprise 60 questions with an equal mix of formula-based problems and application-based
problems. All questions would carry equal marks. Balaji is appearing for the examination. Before,
appearing for the examination he gets the following information from coaching institutes:
I. Application-oriented problems take more time to solve in an examination hall.
II. Chances of silly mistakes would be low in application-based problems.
III. ISM would assist the students with bank loans to start a new venture.
IV. Options are generally confusing for formula-based problems.
V. Practice makes a man perfect' can apply only to formula-based problems.
Based on above information, which of the following options would help him to be better prepared for
the examination?
1. I & II

2. I, III & V

3. II, III & VI

4. IV, V & VI

5. I, II, IV & V

DIRECTIONS for question 34: Answer question on the basis of information given in the following case.
34.

Innovative Institute of Business (IIB) has decided to be the first green campus in India. IIB
Administration has advised all campus residents to reduce carbon footprints. IIB faculty members
did a brainstorming and came up with the following suggestions:
I. Replacing electricity source for street lights with solar panels.
II. Replacing the existing buildings with environment friendly buildings.
III. Organizing a seminar on 'Towards a Sustainable Future' involving all students, staff, and experts
from around the country.
IV. Introducing a compulsory course on sustainability to increase awareness among students.
V. Conducting an initial energy audit to explore where IIB can reduce carbon footprints.
Which of the following options would be the most preferred sequence of actions to reduce carbon
footprints on campus?
1. II, IV, V

2. IV, V, III

3. V, I, II

4. V, I, III

5. V, III, I

DIRECTIONS for question 35 to 38: Answer questions on the basis of information given in the following
case.
The Disciplinary Committee of National Political Party (NPP) is meeting today to decide on the future of two of
their party members, Mr. Loyal and his son Mr. Prodigal. Mr. Prodigal is the prime accused in the brutal murder
of Mr. Victim, an opposition party leader. Mr. Prodigal is in police custody and his appeal for bail has got
rejected. Mr. Loyal claims that his son is innocent and Mr. Victim's death was the result of internal rivalry in
the opposition party. Though Mr. Loyal is not accused in this case, his weakness for his son is well known. The
media is blaming him for influencing key witnesses to protect his son. Severe criticism of this father-son duo,
both by the media and some social activists, is damaging the image of the party. However, Mr. Loyal has
significant followers within the party and is considered an asset to the party.-Any harsh decision against Mr.
Loyal would adversely affect the future of NPP and could even lead to a split in the party. This would benefit
the opposition.
35.

Which of the following actions would adversely affect both NPP and Mr. Loyal, the most?
1. Take no action against Mr. Loyal.
2. Suspend Mr. Prodigal from the party with immediate effect.
3. Expel Mr. Loyal from the party with immediate effect.
4. Ban Mr. Loyal from entering party premises till completion of the court proceedings.
5. Initiate an internal inquiry to find the truth.

Bulls Eye

XAT 2015

13

www.hitbullseye.com

36.

At the Disciplinary Committee meeting, members came up with the following suggestions. Which of
the following suggestions would harm the party, the least?
1. Maintain status-quo.
2. Expel Mr. Prodigal from the party with immediate effect to maintain party's clean image.
3. Initiate an internal inquiry to find the truth.
4. Suspend Mr. Prodigal from the party with immediate effect but announce that he will be taken
back if the court declares him innocent.
5. Suspend both Mr. Loyal and Mr. Prodigal from the party with immediate effect.

37.

Mr. Opportunist, a veteran member of NPP, stakes his claims to be nominated as an NPP candidate
in the upcoming election. Mr. Opportunist presented the following arguments in favour of his
candidature to the NPP Executive Committee.
I. Mr. Loyal's candidature in the upcoming election will adversely impact NPP's chances. Hence,

the party should not nominate him.


II. The party should call a press conference to disown Mr. Loyal. This would enhance the party's image.
III. The party would not be able to take any strong disciplinary action against Mr. Loyal, if he gets
re-elected.
IV. I have a lot of goodwill and significant followers in the constituency.
V. None of my close relatives are into active politics.
Which of the following combinations would best strengthen the claim of Mr. Opportunist?
1. I & III
38.

2. I & IV

3. II & III

4. III & V

5. IV & V

The Disciplinary Committee has decided to suspend Mr. Loyal from the party because they felt he
was influencing the judicial process. However, Mr. Loyal feels that the committee is biased and he
is being framed. Now, election has been announced. The last time, Mr. Loyal had won with a
majority on account of his good work. Which of the following options is most likely to resurrect
Mr. Loyal's immediate political career?
1. The main opposition party has invited Mr. Loyal to join the party and contest the election.
Chance of winning is high.
2. Not participating in the campaign and instructing his followers to stay away from the
campaigning process.
3. Ask his followers to support the NPP nominated candidate and display his loyalty to NPP.
4. Mr. Loyal should contest as an independent candidate. But because of a split in votes, his chances of
winning would be low.
5. Influence the nomination process through his followers within NPP, to get one of his close associates
nominated.

DIRECTIONS for question 39 to 41: Answer question on the basis of information given in the following
case.
Bright Engineering College (BEC) has listed 20 elective courses for the next term and students have to choose
any 7 of them. Simran, a student of BEC, notices that there are three categories of electives: Job-oriented (J),
Quantitative-oriented (Q) and Grade-oriented (G). Among these 20 electives, some electives are both Job and
Grade-oriented but are not Quantitative-oriented (JG type). QJ type electives are both Job and Quantitativeoriented but are not Grade-oriented and QG type electives are both Quantitative and Grade-oriented but are not
Job-oriented. Simran also notes that the total number of QG type electives is 2 less than QG type electives.
Similarly, the total number of QG type electives is 2 less than JG type and there is only 1 common elective
(JQG) across three categories. Furthermore, the number of only Quantitative-oriented electives is same as only

Bulls Eye

XAT 2015

14

www.hitbullseye.com

Job-oriented electives, but less than the number of only Grade-oriented electives. Each elective has at least one
registration and there is at least one elective in each category, or combinations of categories.
39.

On her way back Simran met her friend Raj and shared the above information. Raj is preparing for XAT
and is only interested in Grade-oriented (G) electives. He wanted to know the number of G- type
electives being offered. Simran replied, "You have all the information. Calculate the number of G-type
electives yourself. It would help your XAT preparation". Raj calculates correctly and says that there can
be ___possible answers.
Which of the following options would best fit the blank above?
1. 3

40.

3. 8

4. 9

5. 11

Simran prefers J-type electives and wants to avoid Q-type electives. She noted that the number of only
J-type electives is 3. Raj's preference is G-type electives followed by Q-type electives. However, they
want to take as many common electives as possible. What is the maximum number of electives that can
be common between them, without compromising their preferences?
1. 3
4. 7

41.

2. 5

2. 4
3. 5
5. Not possible to answer from the above information

Vijay and Raj want to avoid each other. Vijay is interested in J-type electives and wants to avoid Q-type
electives. Raj's preference is G-type electives followed by Q-type electives. Raj noted that the number of
only G-type electives is 2. Is there a possibility that they would not share any common elective(s)?
1. Yes. There is a possibility.
2. No. They would meet in one elective.
3. No. They would not be able to avoid in two electives.
4. No. They would meet in five electives.
5. Cannot be solved with the information given.

DIRECTIONS for question 42 to 44: Answer questions on the basis of information given in the following
case.
A few years back Mr. Arbit and Mr. Boring started an oil refinery business. Their annual earning is currently just
50,000 million rupees. They are now exploring various options to improve the business. Mr. Xanadu, a
salesperson from Innovative Technology Solutions (ITS), is trying to sell a new oil refinery technology to Mr.
Arbit and Mr. Boring. This technology could potentially enhance their annual earning to 150,000 million rupees
within a year. But they have to make onetime investment of 100,000 million rupees to implement the technology.
If the technology is not successful, the investment would be lost. Mr. Arbit and Mr. Boring arc discussing about
possible risks of the investment.
42.

Mr. Arbit is enthusiastic about this investment idea but Mr. Boring is a little sceptical. This impasse
makes them approach a consultant. The consultant makes some observations. Which of the following
observations, made by the consultant, might reduce Mr. Arbit's enthusiasm for the new investment idea?
1. Investment is warranted only when benefits outweigh costs.
2. Technology investments give higher earnings in future.
3. Investment in technology leads to reduction of costs in the long run.
4. Technology risks can be controlled.
5. Business is all about taking risky decisions.

Bulls Eye

XAT 2015

15

www.hitbullseye.com

43.

In order to sell the technology to Mr. Arbit and Mr. Boring, Mr. Xanadu is thinking of five possible
sales pitches. Which of the following sales pitches would reduce uncertainties the most for Mr.Arbit and
Mr. Boring?
1. All other competitors are aggressively investing in risky technologies.
2. If the technology succeeds, the annual earnings would grow 3 times from the next financial year
and they would be able to recover the invested money within 1 year.
3. Preliminary studies indicate that success rate of the technology is 85%.
4. The R&D team of ITs is working to counter any possible downside of the technology.
5. Business is all about taking risky decisions.

44.

Mr. Arbit and Mr. Boring did not invest in the new technology, but the new technology is a big success.
Repentant, they are now estimating the additional amount they would have earned (i.e. forgone earnings)
had they invested in the new technology. However, the two owners differed on expected lifespan of the
new technology. Mr. Arbit expected lifespan to be 5 years, whereas, Mr. Boring expected it to be 2
years. After the technology gets out-dated, the earnings from the business would drop back to 50,000
million rupees.
What would be the difference between two expected foregone earnings after 5 years of the technology
investment, if yearly earnings are deposited in a bank @ 10%, compounded annually?
Note: Forgone Earnings = (Earnings from business with new technology) - (Earnings from business
without new technology)
1. 231,200 million rupees
3. 400,510 million rupees
5. 464,100 million rupees

2. 331,000 million rupees


4. 431,000 million rupees

DIRECTIONS for question 45 to 48: Answer questions on the basis of information given in the following
case.
Life Saving Pharmaceuticals (LSP) is an India-based pharmaceutical company. Their business mostly revolves
around a couple of generic drugs and a few patented drugs. LSP operates in 30 odd countries and more than 50%
of their sales volume is from outside India.
45.

If more than 50% of their sales volume is from generic drugs, which of the following options is
definitely correct? {Note: All percentages figures are with respect to total sales volume)
1. If sales volume of patented drugs in India is 43%, the sales volume of generic drugs in India will be
less than 43%.
2. If the sales volume of generic drugs in foreign countries is at least 24%, the sales volume of patented
drugs in India will be above 24%.
3. If the sales volume of patented drugs in India is 54%, the sales volume of generic drugs in foreign
countries will be above 54%.
4. If the sales volume of patented drugs in India is 29%, the sales volume of generic drugs in foreign
countries will be above 29%.
5. If the sales volume of generic drugs in India is at least 60%, the sales volume of patented drugs in
foreign countries will be above 60%.

16

46.

Bulls Eye

XAT 2015

www.hitbullseye.com

Mr. Sinha, a senior executive of LSP, observes that their business in India is not vibrant. LSP faces
stiff competition from Indian and global players, except in rural areas. Interestingly, most of their
sales in the rural areas are from cough syrup, used as sedatives by teenagers. Mr. Sinha is planning
the following actions to improve business in the long run.
I. Invest in development of new drugs.
II. Increase sales of cough syrup in the rural markets.
III. Try and cut costs.
IV. Recruit more medical representatives in the rural areas.
Which of the following sequences is best arranged in the descending order of appropriateness?
1. I, III, II

47.

2. II, I, III

3. II, III, I

4. IV, II, III

5. IV, III, I

Mr. Rastogi, HR head of LSP, is contemplating of transferring Mr. Jose, from India to their Luxembourg
office. Mr. Jose's wife is also with the HR department of LSP. The couple is expecting their first child
within next four months and hence they want to be together. Mr. Rastogi is wondering whether Mr. Jose
would accept the transfer. If he doesn't, Mr. Rastogi would have to send a less competent person for this
job as early as possible. The office in Luxembourg is very important for the company's future. It is at its
nascent stage and does not yet have an HR department. Hence, it is not possible to transfer Mrs. Jose to
Luxembourg.
Which of the following options would be most appropriate, from the organization's perspective, to
resolve the issue?
1. Giving a salary hike to Mr. Jose with a promise to transfer Mrs. Jose to Luxembourg in the near
future.
2. Giving Mrs. Jose option to work from home while in Luxembourg so that she can be with Mr.
Jose.
3. Giving Mr. Jose option to work from India for the time being so that he can be with Mrs. Jose in
India.
4. Giving a salary hike to Mr. Jose to compensate for Mrs. Jose's salary so that she can join Mr. Jose at
Luxembourg, even with loss of pay.
5. Asking Mr. Jose to accept the offer right now but give him up to six months to join Luxembourg
office.

48.

Mr. Khan used to work as the Vice President of LSP India. However, he had resigned from LSP India
for a better job in New York. In the meantime, his wife was promoted to head the HR of LSP India. Mrs.
Khan had straggled hard to reach this position and was quite popular and respected within the
organization. Mrs. Khan was contemplating whether she should give up her career and join him in New
York. Mrs. Khan is considering the following actions:
I. Take a break for the time being and focus on personal life. Given her reputation, she can always get
back to the same job, if required.
II. Go to New York, on leave without pay for two months to help Mr. Khan settle down. After that she
can come back and resume her responsibility in LSP India.
III. Request Mr. Khan to look for an equivalent job in India.
IV. Resign from LSP India, join Mr. Khan in New York, and look for a similar job there.
V. Request LSP India for a similar position in LSP USA and follow Mr. Khan to New York.
Which of the following sequence of actions can be immediately taken by Mrs. Khan to maintain her
work-life balance?
1. I & II

2. I & III

3. I & IV

4. II & V

5. III & V

Bulls Eye

XAT 2015

17

www.hitbullseye.com

DIRECTIONS for question 49 to 51: Answer questions on the basis of information given in the following
case.
Mohan's was a popular fast-food joint at Connaught Place, Delhi. Initially Mohan handled his business alone.
His sons, Ram and Kishan, joined the business after graduating from college. Ram was entrepreneurial in
nature. Subsequently, another branch of Mohan 's was opened in Panipat. Mohan had chosen Ram to head the
Panipat branch. Though Ram increased sales in a short time, he had stopped using premium quality organic
vegetables, the speciality of Mohan's. Mohan and Kishan were not happy with his way of doing business.
Now, the foremost challenge for Mohan was to sort out this issue with Ram. Mohan knew that replacing Ram
with Kishan was difficult as Kishan did not want to leave Delhi. However, giving a freehand to Ram might
have long term negative consequences. Mohan was confused about the future course of actions.
49.

Mohan sought the help of five consultants, who gave the following opinions:
I. Organic vegetables might be a big success at Connaught place but awareness about organic
vegetables is low among Panipat customers.
II. The Connaught place model can be implemented in Panipat provided the business is prepared to face
the consequences.
III. Many high end restaurants in Panipat use organic vegetables. So, using organic vegetables will not
be a differentiating factor.
IV. Selling prices of their dishes in Panipat are significantly lower. Using organic vegetables will bring
down profits.
V. Premium quality organic vegetables are not easily available in Panipat.
Which of the following set of options would support Ram's argument of not using organic vegetables?
1. I, III, IV

50.

2. II, IV, V

3. I, III, IV, V

4. II, HI, IV, V

5. All of the above

Mohan sought feedback from a few of his businessmen friends, who were familiar with both the
branches. Here is what they said:
Businessman 1: Customers of Connaught place and Panipat are very different.
Businessman 2: Customers in Panipat are extremely happy with Ram's behaviour.
Businessman 3: Panipat branch does not use the same quality of ingredients but maintains good hygiene
and taste.
Businessman 4: Who knows, tomorrow the customers of Panipat might also appreciate what Connaught
place customers appreciate today!
If Mohan thinks all these are valid concerns, which of the following actions would be best for the
business?
1. Training Kishan to replace Ram in a few months.
2. Not worrying about ingredients as long as business grows.
3. Bringing Ram to Connaught place branch.
4. Naming the Panipat branch as 'Ram's', and changing it back to Mohan's, when needed.
5. Asking Kishan to run the Panipat branch.

51.

After discussing with a few customers, Mohan realised that compromising on the quality of ingredients
at Panipat branch may not be a good idea but at the same time he also realised that Panipat branch had
grown fast. He was contemplating following five actions. Which of the following actions would be the
best for the future of his business?
1. Creating awareness campaign for organic vegetables in Panipat.
2. Mohan himself should look after the Panipat branch.
3. Close down the Panipat branch.
4. Send Kishan to Panipat branch and bring Ram to Connaught place permanently.
5. Hire a new person to run the Panipat branch

18

Bulls Eye

XAT 2015

www.hitbullseye.com

SECTION III
Quantitative Ability and Data Interpretation
52.

What is the sum of the following series?


-64,-66,-68 . - 100
1. - 1458
4. - 1664

53.

Number of questions: 32

2. -1558
5. None of the above

3. -1568

Ramesh plans to order a birthday gift for his friend from an online retailer. However, the birthday
coincides with the festival season during which there is a huge demand for buying online goods and
hence deliveries are often delayed. He estimates that the probability of receiving the gift, in time, from
the retailers A, B, C and D would be 0.6, 0.8, 0.9 and 0.5 respectively.
Playing safe, he orders from all four retailers simultaneously. What would be the probability that his
friend would receive the gift in time?
1. 0.004

54.

2. 0.006

3. 0.216

4. 0.994

5. 0.996

The figure below has been obtained by folding a rectangle. The total area of the figure (as visible) is
144 square meters. Had the rectangle not been folded, the current overlapping part would have been a
square. What would have been the total area of the original unfolded rectangle?

6m

14 m

1. 128 square meters


3. 162 square meters
5. None of the above
55.

The Maximum Retail Price (MRP) of a product is 55% above its manufacturing cost. The product is
sold through a retailer, who earns 23% profit on his purchase price. What is the profit percentage
(expressed in nearest integer) for the manufacturer who sells his product to the retailer? The retailer
gives 10% discount on MRP.
1. 31%

56.

2. 22%

3. 15%

4. 13%

5. 11%

A solid metal cylinder of 10 cm height and 14 cm diameter is melted and re-cast into two cones in the
proportion of 3:4 (volume), keeping the height 10 cm. What would be the percentage change in the flat
surface area before and after?
1. 9%

57.

2. 154 square meters


4. 172 square meters

2. 16%

3. 25%

4. 50%

5.None of the above

A circular road is constructed outside a square field. The perimeter of the square field is 200 ft. If the
width of the road is 7 2 ft. and cost of construction is Rs. 100 per sq. ft. Find the lowest possible cost
to construct 50% of the total road.

1. Rs. 70,400
4. Rs. 235,400

2. Rs. 125,400
5. None of the above

3. Rs. 140,800

Bulls Eye

XAT 2015

19

www.hitbullseye.com

58.

Product M is produced by mixing chemical X and chemical Y in the ratio of 5:4. Chemical X is
prepared by mixing two raw materials, A and B, in the ratio of 1:3. Chemical Y is prepared by mixing
raw materials, B and C, in the ratio of 2:1. Then the final mixture is prepared by mixing 864 units of
product M with water. If the concentration of the raw material B in the final mixture is 50%, how much
water had been added to product M?
1. 328 units
4. 616 units

59.

2. 368 units
5. None of the above

Find the equation of the graph shown below.

2. y = 2x2 - 40
5. x = 2y2 + 3y - 19

1. y = 3x - 4
4. y = 2x2 + 3x - 19
60.

3. x = 2y2 - 40

Two diagonals of a parallelogram intersect each other at coordinates (17.5, 23.5). Two adjacent
points of the parallelogram are (5.5, 7.5) and (13.5, 16), Find the lengths of the diagonals.
1. 15 and 30
4. 17 and 40

61.

3. 392 units

2. 15 and 40
5. Multiple solutions are possible

3. 17 and 30

In the diagram below, CD = BF = 10 units and CED = BAF = 30. What would be the area of
triangle AED? (Note: Diagram below may not be proportional to scale.)
A

F
C

1. 100

2 3

2. 100/

3 3

3. 50/

34

4. 50

34

5. None of these

62.

Bulls Eye

XAT 2015

20

www.hitbullseye.com

The tax rates for various income slabs are given below.
Income Slab (Rs.)
500
> 500 to 2000
> 2000 to 5000
> 5000 to < 10000

Tax Rate
Nil
5%
10%
15%

There are 15 persons working in an organization. Out of them, 3 to 5 persons are falling in each of the
income slabs mentioned above. Which of the following is the correct tax range of the 15 persons? (E.g.
If one is earning Rs. 2000, the tax would be: 500 0 + 1500 0.05)
1. 1350 to 7350, both excluded
3. 2175 to 7350, both excluded
5. None of the above
63.

If a, b, c and d arc four different positive integers selected from 1 to 25, then the highest possible value
of ((a + b) + (c + d)) /((a + b) + (c - d)) would be:
1. 47

64.

3. 51

4. 96

5.None of the above

2. 7

3. 8

4. 9

5.None of the above

In the beginning of the year 2004, a person invests some amount in a bank. In the beginning of 2007,
the accumulated interest is Rs. 10,000 and in the beginning of 2010, the accumulated interest becomes
Rs. 25,000. The interest rate is compounded annually and the annual interest rate is fixed. The principal
amount is:
1. Rs. 16,000

66.

2. 49

If f(x2 - 1) = x4 - 7x2 + k, and f(x3 - 2) = x6 9x3 + k2 then the value of (k2 - k1) is
1. 6

65.

2. 1350 to 9800, both included


4. 2175 to 9800, both included

2. Rs. 18,000

3. Rs. 20,000

4. Rs. 25,000

5.None of the above

Devanand's house is 50 km West of Pradeep's house. On Sunday morning, at 10 a.m., they leave their
respective houses.
Under which of the following scenarios, the minimum distance between the two would be 40 km?
Scenario I: Devanand walks East at a constant speed of 3 km per hour and Pradeep walks South at
a constant speed of 4 km per hour.
Scenario II: Devanand walks South at a constant speed of 3 km per hour and Pradeep walks East
at a constant speed of 4 km per hour.
Scenario III: Devanand walks West at a constant speed of 4 km per hour and Pradeep walks East
at a constant speed of 3 km per hour.

1. Scenario 1 only 2. Scenario II only 3. Scenario III only 4. Scenario 1 and II 5.None of the above

Bulls Eye

XAT 2015

21

www.hitbullseye.com

67.

The median of 11 different positive integers is 15 and seven of those 11 integers are 8, 12, 20, 6, 14, 22,
and 13.
Statement I: The difference between the averages of four largest integers and four smallest integers is
13, 25.
Statement 11: The average of all the 11 integers is 16.

Which of the following statements would be sufficient to find the largest possible integer of these
numbers?
1. Statement I only.
2. Statement II only.
3. Both Statement 1 and Statement II are required.
4. Neither Statement I nor Statement II is sufficient.
5. Either Statement I or Statement 11 is sufficient.
68.

The parallel sides of a trapezoid ABCD are in the ratio of 4:5. ABCD is divided into an isosceles
triangle ABP and a parallelogram PBCD (as shown below). ABCD has a perimeter equal to 1120
meters and PBCD has a perimeter equal to 1000 meters, bind Sin ABC, given 2DAB = BCD.
B

1. 4/5
4. 24/25
69.

2. 16/25
5. A single solution is not possible

3. 5/6

An ascending series of numbers satisfies the following conditions:


I. When divided by 3, 4, 5 or 6, the numbers leave a remainder of 2.
II. When divided by 11, the numbers leave no remainder.
The 6th number in this series will be:
1. 242

70.

3. 3542

4. 4202

5.None of the above

In an examination, two types of questions are asked: one mark questions and two marks questions. For
each wrong answer, of one mark question, the deduction is 1/4 of a mark and for each wrong answer, of
two marks question, the deduction is 'A of a mark. Moreover, 1/2 of a mark is deducted for any
unanswered question. The question paper has 10 one mark questions and 10 two marks questions. In the
examination, students got all possible marks between 25 and 30 and every student had different marks.
What would be the rank of a student, who scores a total of 27.5 marks?
1. 5

71.

2. 2882

2. 6

3. 7

4. 8

5.None of the above

For a positive integer x, define f(x) such that f(x + a) = f(a x), where a is an integer and f(1) = 4. If the
value of f(1003) = k, then the value of 'k' will be:
1. 1003

2. 1004

3. 1005

4. 1006

5.None of the above

72.

www.hitbullseye.com

The centre of a circle inside a triangle is at a distance of 625 cm. from each of the vertices of the
triangle. If the diameter of the circle is 350 cm. and the circle is touching only two sides of the
triangle, find the area of the triangle.
1. 24000

73.

Bulls Eye

XAT 2015

22

2. 387072

3. 48000

4. 506447

5.None of these

If the last 6 digits of [(M)!(N)!] are 999000, which of the following option is not possible for (M)
(M-N)?
Both (M) and (N) are positive integers and M > N. (M)! is factorial M.
1. 150

74.

2. 180

3. 200

4. 225

5. 234

A person is standing at a distance of 1800 meters facing a giant clock at the top of a tower. At 5.00
p.m., he can see the tip of the minute hand of the clock at 30 degree elevation from his eye-level.
Immediately, the person starts walking towards the tower. At 5.10 pm., the person noticed that the tip of
the minute hand made an angle of 60 degrees with respect to his eye-level. Using three- dimensional
vision, find the speed at which the person is walking. The length of the minutes hand is 200 3 meters
( 3 = 1.732).
1. 7.2 km/hour

75.

3. 7.8 km/hour

4. 8.4 km /hour

5.None of the above

A three-digit number has digits in strictly descending order and divisible by 10. By changing the places
of the digits a new three-digit number is constructed in such a way that the new number is also divisible
by 10. The difference between the original number and the new number is divisible by 40. How many
numbers will satisfy all these conditions?
1. 5

76.

2. 7.5 km /hour

2. 6

3. 7

4. 8

5.None of the above

Three pipes are connected to an inverted cone, with its base at the top. Two inlet pipes, A and B, arc
connected to the top of the cone and can fill the empty cone individually in 8 hours and 12 hours,
respectively. The outlet pipe C, connected to the bottom, can empty a filled cone in 4 hours.
When the cone is completely filled with water, all three pipes are opened. Two of the three pipes
remain open for 20 hours continuously and the third pipe remains open for a lesser time. As a result, the
height of the water inside the cone comes down to 50%. Which of the following options would be
possible?
1. Pipe A was open for 19 hours.
2. Pipe A was open for 19 hours 30 minutes.
3. Pipe B was open for 19 hours 30 minutes.
4. Pipe C was open for 19 hours 50 minutes.
5. The situation is not possible.

Bulls Eye

XAT 2015

23

www.hitbullseye.com

DIRECTIONS for question 77 to 80: Answer questions on the basis of information given below:
Twitter allows its users to post/share and read short messages known as tweets. Tweets can be of three types Positive Tweets (in support), Negative Tweets (against) and Neutral Tweets. The following table presents the
Number of Votes and Tweets received by certain political parties.

Parties
Year 2000

Number of Votes
Year 2005 Year 2010

Total No.
of tweets
131,021
108,128
96.620
41,524
32,724
15,000

Tweets (Year 2010)


Positive
Negative
Tweets (%) Tweets (%)
33.3%
35.4%
30.4%
29.7%
32.5%
26.6%
30.6%
36.1%
21.6%
41.0%

A
329,700
343,200
364,450
B
133,450
154,000
241,325
C
196,250
123,200
162,525
D
27,475
48,400
54,175
E
30,800
49,250
Others
98,125
180,400
113,275
Parties*
* Any party which has secured less than 2% of the total votes falls under 'Other Parties' category. For
example, Party E secured less than 2% of total votes, in the year 2000.

Note: If the vote share (%age of total votes) of a party changes from 15% to 40%, gain in vote share
would be 25% (= 40% - 15%).
77.

Which of the following options correctly arranges the political parties in descending order of gain in
vote share from the year 2005 to the year 2010?
1. EBDCA

78.

2. EBCDA.

5. BCEAD

2. Party C
3. Party D
5. One of the parties categorized under 'Other Parties'

Between 2000 and 2010, in terms of gain in vote share which of the following cannot be a possible
value (approximated to one decimal place) for any party?
1. 2.0%

80.

4. BCEDA

Which of the following parties received maximum number of "neutral tweets" in the year 2010?
1. Party B
4. Party E

79.

3. EBCAD

2. 2.5%

3. 3.5%

4. 4.5%

5. 7.5%

In 2010, which of the following options has maximum difference between the vote share and tweet
share?
1. Party B

2. Party C

3. Party D

4. Party E

5. Other Parties

24

XAT 2015

Bulls Eye
www.hitbullseye.com

DIRECTIONS for question 81 to 84: Answer questions on the basis of information given below:
As a part of employee improvement programs, every year an organization conducts a survey on three factors: 1.
Number of days (in integers) of training undergone, 2. Amount of bonus (in lacs) received by an employee and
3. Employee effectiveness score (on the scale of 1 to 10). Survey results for last two years are given below for
the same seven employees.

Bulls Eye
www.hitbullseye.com

XAT 2015

25

81.

Bulls Eye

XAT 2015

26

www.hitbullseye.com

In Survey 1, what was the average bonus earned by employees who underwent training for more than
17 days?
1. Between 16 and 17 lacs
3. Between 18 and 19 lacs
5. None of the above

82.

Identify the number of employees whose employee effectiveness score was higher than 7 in Survey 1,
but whose bonus was lower than 20 lacs in Survey 2.
1. 2

83.

2. 3

3. 4

4. 5

5.None of the above

From Survey 1 to Survey 2, how many employees underwent more days of training but their annual
bonus decreased?
1. 1

84.

2. Between 17 and 18 lacs


4. Between 19 and 20 lacs

2. 2

3. 3

4. 4

5. None of these

From Survey 1 to Survey 2: for how many employees training days increased along with an increase of
employee effective score by at least 1,0 rating?
1. 2

2. 3

3. 4

4. 7

5.None of the above

Bulls Eye

XAT 2015

27

www.hitbullseye.com

SECTION IV
General Knowledge
85.

Number of questions: 30

Which of the following statements is correct about Union Budget 2014?


1. Current Account Deficit was around 5% of GDP.
2. Current Account Deficit was around 6.2% of GDP.
3. Current Account Deficit was around 3.5 % of GDP.
4. Current Account Deficit was around 1.7% of GDP.
5. Current Account Deficit was around 0.5% of GDP.

86.

How many smart cities have been announced in India?


1. 50

87.

2. 60

3. 70

4. 80

5.None of the above

What is "Qualified Institutional Placement"?


1. An agency that helps in providing jobs to students
2. It is business school method of providing jobs
3. It is a way of starting business in the U.S
4. It is a way of raising money in India
5. None of the above

88.

What is "Quantitative Easing"?


1. Reducing excise duty so as to promote higher demand.
2. Restricting the number of units produced by a factory.
3. Allowing companies to sell as much as possible.
4. A way of influencing money supply by the central bank.
5. None of the above

89.

What is "Bachpan Bachao Andolan"?


1. It is a movement to provide primary education to all children.
2. Supreme Court's directive to schools to reduce the weight of school bags.
3. A movement in India protecting the rights of children.
4. A movement in Pakistan to protect girl child.
5. None of the above

90.

Which of the following authors has not been awarded Pulitzer prize?
1. Paul Harding

91.

2. Donna Tartt

3. Adam Johnson

4. Jhumpa Lahiri

5. Arundhuti Roy

III. Honeycomb

IV. Kit Kat

V. Lollipop

Consider the following names:


I. Cadbury

II. Garlic Bread

Which of the following options lists all the correct names of Android operating system?
1. I, II & IV

2. I, IV & V

3. I, III & IV

4. II, IV & V

5. III, IV & V

28

92.

3. Salam Bombay

4. Swadesh

5. Water

2. Vishakhapatnam 3. Porbandar

4. Chennai

5. Paradip

Which of the following towns is not associated with coal/lignite mining?


1. Neyveli

95.

2. Mother India

Which of the following cities is least likely to experience a cyclonic disturbance?


1. Mumbai

94.

www.hitbullseye.com

Which of the following Indian movies did not get nominated for 'Academy Awards' in the 'Best
Foreign Language Film' category?
1. Lagan

93.

Bulls Eye

XAT 2015

2. Ramgarh

3. Jharia

4. Raniganj

5. Brahmapur

Four possible industrial belts are given below:


I. Bengaluru-Coimbatore-Madurai
II. Mumbai-Pune
III. Ahmedabad-Vadodara
IV. Hugli Region
Which of the above are well known industrial belts (regions) in India?
1. I, II, III

96.

2. I, I, IV

3. II, III, IV

4. I, II, III, IV

5.None of the above

Consider the following statements related to Ukraine


I. It supplies gas to Russia.
II. Russia sells gas to Ukraine.
III. Sochi is not located in Ukraine.
IV. In a referendum, people of Crimea decided to be part of Ukraine.
V. Simferopol is the capital of Crimea.
Which of the following options contains wrong statement(s) related to Ukraine?
1. I, II

97.

2. I, IV

3. II, III

4. II, V

5. III, IV, V

Consider the following statements about the Syrian crisis:


I. It was initially related to Arab spring.
II. Many Syrian refugees flee to Jordan and Lebanon.
III. Bashar Al Assad is involved in Syria crisis.
IV. There has been a single party political government in Syria for over a decade.
V. Syrian crisis is not related to Tunisia.
Which of the following options does not contain false statement pertaining to Syrian crisis?
1. I, II, III, IV

2. I, II, IV, V

3. I, III, IV, V

4. II, III, IV, V

5. I, II, III, V

Bulls Eye

XAT 2015

www.hitbullseye.com

98.

Consider the following statements related to Brazil:


I. Brazil's GDP is higher than Italy's GDP.
II. Dilma Rouseff was elected as President of Brazil.
III. Maracana stadium hosted world cup football final.
IV. Of all of Brazil's neighbours, Argentina has the longest border.
V. Real is the currency of Brazil.
Which of the following options does not contain false statement(s) related to Brazil?
1. I, II, III, IV

99.

2. I, II, IV, V

3. I, III, IV, V

4. I, II, III, V

5. II, III, IV, V

Which of the following options correctly lists all the dignitaries, who visited India in 2014?
1. Ji Xinping, Tony Abbott, Joachim Gauck, David Johnston, Shinzo Abe
2. David Cameron, Tony Abbott, Ji Xinping, David Johnston, Joachim Gauck
3. Kim Jong Un, Tony Abbott, Joachim Gauck, David Johnston, Shinzo Abe
4. Kim Jong Un, Tony Abbott, Joachim Gauck, David Johnston, Francois Hollande
5. Kim Jong Un, David Cameron, Joachim Gauck, David Johnston, Francois Hollande

100.

Which of the following statements is closely associated with Boko Haram?


1. It is an organization opposed to Western education in Nigeria.
2. It is fighting for freedom against the oppressive Nigerian regime.
3. It wants to defend the pristine culture of Nigeria.
4. It is an arm of the Nigerian government to promote Islam.
5. It is an organization set up for abducting women.

101.

Where is Taxila located?


1. Bihar
3. North West Frontier Province, Pakistan
5. Baluchistan, Pakistan

102.

From which of the following space stations was Mangalyaan launched?


1. Sriharikota

103.

2. Gujarat
4. Punjab, Pakistan

2. Balasore

3. Thumba

4. Wheeler Island

5. Mahendragiri

Which of the following statements is right?


1. Number of organised labour is more than number of unorganized labour in India.
2. Number of unorganised labour is more than number of organized labour in India.
3. Organised labour in India is difficult to count.
4. Organized and Unorganised labour in India is approximately equal.
5. There is no unorganised labour in India.

104.

'Hudhud', the name of recent cyclone, is associated with:


1. National bird of Israel
3. Bangladeshi fowl

2. Malaysian pet dog


4. Telugu language

5. Mandarin

29

30

105.

Bulls Eye

XAT 2015

www.hitbullseye.com

Which of following is the correct list of sportspersons awarded gold medal at Incheon Asian games?
1. Jitu Rai, Yogeshwar Dutt, Tintu Luka, Mary Kom
2. Saurav Ghoshal, Mary Kom, Sania Mirza, Krishna Punia
3. Krishna Punia, Rajat Chauhan, Saurav Ghoshal, Mary Kom
4. Seema Punia, Vikas Gowda, Sania Mirza, Jitu Rai
5. Sania Mirza, Vikas Gowda, Yogeshwar Dutt, Rajat Chauhan

106.

As per 2011 census, which is the second most urbanized State in India (in percentage terms)?
1. Kerala

107.

2. Mizoram

3. Gujarat

4. Tamil Nadu

5. Punjab

4. SM Pollock

5. Wasim Akram

Christine Lagarde is:


1. a famous tennis player of yesteryears
2. an American stage and screen actress
3. the Managing Director of the International Monetary Fund
4. the Chief Executive Officer of Nestle
5. a British political activist

108.

Consider the following cricketers:


I. A Kumble

2. CA Walsh

3. GD McGrath

Arrange the above cricketers in descending order of test wickets taken:


1. II, III, IV, V, I

109.

2. II, IV, V, I, III

3. I, III, II, IV, V

4. IV, III, II, V, I

5. V, III, II, IV, I

III. India

IV. Indonesia

V. Philippines

Consider the following nations:


I. Bangladesh

II. Brazil

Arrange the above nations in descending order of rice production:


1. II, IV, V, I, III

110.

2. II, III, IV, V, I

3. IV, III, II, V, I

4. V, III, II, IV, I

5. III, IV, I, V, II

Consider the following Tennis Grand Slams:


I. Australian Open 2. French Open

3. US Open

4. Wimbledon

Arrange the above Grand Slams in the order of their occurrence in a calendar year:
1. IV, III, II, I

111.

2. III, II, IV, I

3. I, II, IV, III

4. II, IV, I, III

Consider the following dynasties:


I. Chola Dynasty
III. Hoysala Dynasty
V. Pallava Dynasty

II. Chalukya Dynasty


IV. Pala Dynasty
VI. Kushana Dynasty

Arrange the above dynasties in chronological order:


1. I, VI, V, II, IV, III
3. I, II, IV, III, VI, V
5. II, III, IV, I, VI, V

2. I, II, III, VI, V, IV


4. II, IV, I, III, V, VI

5. II, III, IV, I

Bulls Eye

XAT 2015

31

www.hitbullseye.com

112.

Which of the following parties had contested the maximum number of seats in the 2014 Indian
General Election?
1.A am Aadmi Party
3. Bahujan Samaj Party
5. Indian National Congress

113.

2. All India Trinamool Congress


4. Bharatiya Janata Party

Which of following set of cities/regions is associated with Indian Super League (ISL) teams?
1. Sikkim, Kochi, Bengaluru, Delhi
2. North East, Kerala, Goa, Delhi
3. Goa, Delhi, Mumbai, Bengaluru
4. Chennai, Goa, Chandigarh, Jaipur
5. West Bengal, Bengaluru, Kerala, Hyderabad

114.

Consider the following list of some Countries and Capitals:

Countries

Capitals

A. Guyana
B. Uzbekistan
C. Estonia
D. Guinea

I. Dili
II. Tashkent
III. Tallinn
IV. Georgetown

Match the countries with their capitals:


1. A-I, B-II, C-III, D-IV
3. A-III, B-IV, C-III, D-V, F-I
5. A-IV, B-II, C-III, D-V, F-I

2. A-IV, B-II, C-III, D-V, E-I


4. A-IV, B-V, C-I, D-V, E-III

Bulls Eye

XAT 2015

32

www.hitbullseye.com

Answer key & Explanations


Set
B

Key

Explanations

1.

2.

3.

4.

5.

6.

7.

8.

9.

10.

11.

12.

13.

14.

15.

16.

17.

18.

S and P are a mandatory pair because they in S refers to optimists in P. Further S should come immediately before
statement 6 because 6 talks about positive outlook, which is the subject of discussion in S. Opening statement, should
be R only as it very well explains why the world of cinema as mentioned in statement 1, is strange and baffles a critic.
Infantile is an unacceptable childish behavior; similarly charlatan, which means, deception is also unacceptable;
imbecile, meaning idiot, is also unacceptable; awful means unacceptable;
The only positive word is childlike that means an innocent behavior as of a child.
3 should be followed by 2, as 3 talks about loyal and effective allies and 2 talks about this marriage which is referring
to loyal and effective allies. So we are left with answer choices
A and B. Out of the two B is better as the opening statement 4 is more appropriate than 5.
The key line/words is/are has not been eternal. So the next line will hint at limited lifespan. This is reflected in
option A.
The paragraph talks about two different formulations, which were responsible for the genesis of a common theory. The
only sensible option is B because it talks about how they both complemented each other and were used together by
somebody else who might have used the combined theory to generate a new one.
We have to prove that Ranu is a god sportsperson. Option A, C, D and E are talking about Ranus running/sprinting
skills at different levels but does not clearly indicate her superior skills in sports. But B clearly states that she is national
champion.
Calamitous, catastrophic and Cataclysmic all mean disastrous. Cacophonic means harsh sounds; cacographic means
bad handwriting; contraindicative means to indicate the inadvisability of something, such as a medical treatment.
Speakers can be eminent- meaning outstanding and well recognized; immanent means inherent, permanent etc and this
can be filled in the second blank as the belief in permanent justice makes sense. 3rd blank can be filled with Imminent
as it means something about to happen. Last blank will surely take eminence.
The paragraph is talking about a technique to interpret dreams. Dreams are definitely covert entities and the
interpretation can be termed as overt. E fits in this explanation very well as dreams- the covert causes- according to the
passage can lead to overt interpretations.
The last blank will surely take proscribe- meaning to forbid- as it starts from however, despite repeated requests.;
this structure clearly reflects that the court rejected the appeal i.e. the court proscribed a lie detector test. In the second
last blank only demeanor- can fit in as it means outlook/ behavior/ appearance whereas deportment means expulsion
and of cousre cannot be filled in here. Thats why answer is C.
It is difficult for us to understand that how come murali topped inspite of not working hard and to understand this we
need to understand the causes that existed in past. This is reflected in the option E.
it is clearly mentioned in the third paragraph that the protagonist decides to get out of his confinement by getting in the
place of the dead as it is only the dead that can pass freely from this dungeon. (Read the first line of 3rd para). The
subsequent lines mentions the way he executes his plan.
As per the passage first he decided to end his life and was searching ways to end life but somehow at the time of final
decision he decided not to go for. This can be understood, as per the passage by the phrase sarcasm of destiny that if it
is not there in destiny, then it will not happen. So if it is not in the destiny of a person to commit suicide to end
sufferings then it will not happen and instead he will live and fight against the sufferings of life. In other words to give
way to the sarcasm of life.
In the first paragraph the protagonist is thinking about suicide but in the subsequent paras he decides not to play in the
hands of destiny and plans and finally daringly executes his way out of the prison and escapes from his miserable life.
Thats why answer is from depression to daring.
Counterpane is given in 7th sentence last paragraph; the sentence says covered it with his counterpane... The only
option that fits in this context is bed as he made the corpse made sit on the couch and covered it with his beddings/bed.
Dungeon means cell according to the context of the last statement of 2nd paragraph paced twice or thrice round the
dungeon and then paused abruptly by the bed.
Guillotine means execution/killing according to the 8th line of the 1st paragraph, wherein it is stated that I will remain
hererush on the first person. And they will guillotine.
it is clearly mentioned in the very last paragraphs 1st line that in early stages of learning, neural circuits are
.weakly and in the subsequent paras it is mentioned that neural connections lead to learning. S it is clear that in
the younger age because the neural connections happen in a piecemeal manner, the learning is also piecemeal.
Statement 1 is wrong because there is no mention that the two hemispheres learn autonomously. Instead its suggested in
the 4th line that it is not clear how the two sides learn things.
Statement 2 is right as the 3rd para suggests that the neurons and circuits keep forming connections among themselves,
a phenomenon that suggests that simultaneous activation f circuits can take place.
Statement 3 is right as it is clearly mentioned in the 3rd line of the 1st para.
Statement 4 is not mentioned anywhere
Statement 5 is absolutely opposite to the idea mentioned in the paragraph that learning is the result of the neuron
connection.
The entire passage is based on the fact that learning is based on the connections between the neurons and forming of

Bulls Eye

XAT 2015

33

www.hitbullseye.com

circuits between different parts of the brain. The only phrase that captures this essence is E because it means that many
small things make up a huge thing; a fact similar to many small connections between neurons leading to huge thing as
learning.
19.
20.

B
D

21.

22.
23.

A
E

24.

25.

26.

27.

28.

29.

30.

31.
32.

D
D

33.
34.

E
C

35.
36.

C
D

37.

38.
39.

A
B

The lines and even when they lead in ... show that they possibly thrive in anaesthetized (ADVERSE/controlled)
conditions The critical element is the meaning of the word anaesthetised, which has an implied meaning of
controlled in the given sense. We can infer from the passage that orchids survive in a controlled environment.
In the third paragraph the lines recently, however suggest that it is not always true that children born with genetic
vulnerability will be sociopaths etc.
The means susceptible to disease.
It is not possible to conclude about children typology about the situation given as there could be changes in the
behavioral pattern, while growing up, due to environmental surroundings.
As AIR is not a private commodity so markets will not be efficient in this case. Remember, the paragraph states that
public goods are ones for which no individual can be excluded. Air is the only example for which an individual cannot
be excluded. It is required by all.
In the passage it is mentioned that public intervention is needed when market fails. But it cannot be concluded that
public intervention is remedy for all ills related to market failure. As there is not fact to support this conclusion.
Statement 1 is a clear market problem and statement 2 provides a possible solution for it. By introducing such a
measure, the government is effectively pushing industries not to dump their waste.
The first thing that you need to identify here is that the two paragraphs do not contradict each other and they are talking
of the same subject. Thus, the first three options are ruled out. In the first paragraph, the author talks about the
technique of brainstorming and in the second paragraph he explains its benefits.
As per the last line of the passage involvement of the people with different perspectives enriches the idea generation.
This is reflected in option 4.
Mr. Patel has been given notice on behavioral grounds. Hence, he must focus on his performance to avoid being too
vulnerable to be dismissed.
As the offer made Is comparable to the present job in terms of salary so he should accept this offer with some hike (at
least 10%, as per the passage). There is no point in explaining his point as he is still doubtful that it will work.
The HR should first talk to individual parties involved and then have a joint meeting of all to sort out differences
Option D is the best in this case as keeping separate cut-off would help in selecting students with entrepreneurial mind
set. Giving extra incentive for attempting application based problem will definitely encourage/attract students with
entrepreneurial mind -set.
Option III & VI are irrelevant to his achieving the desired score.
First energy audit should be conducted to find out where IIB can reduce carbon footprints. Course of action Stated in
the statement I can be immediately done without waiting for energy audit report. Finally existing buildings should be
replaced with environmental friendly buildings.
As Mr. Loyal is important resource for the party, expelling him will have most adverse effect on the party.
By suspending Mr. prodigal from the party will save the image of the party. By promising, that he will taken back in
party if he is proved innocent by the court , party will be able to take care of ego of father( one of the prominent
leaders) of Mr. Prodigal.
Mr. Opportunist wants himself to be in place of Mr. Loyal. Hence a combination of events, which removes Mr. loyal
and supports his chances of replacing him, is the best.
Immediate political career can be understood as winning the elections, which is only supported by A
As per the statements the following figure can be arrived at with the help of variables. The total is given to be 20 and all
QX
common is given to be 1.
JX

Y
1
Y+42

Y+22

More than X
GX
Now based on this the question states to find the possible values of G type electives. Taking the value of Y as 1, there
are three values of G that will be generated which will be 13, 15 and 17. Similarly taking the value of Y as 2, the
number of G electives is 14 and 16. On the same logic taking the value of Y as 3, the G electives generated are again
15, which is previously obtained. Hence in total there are five values of G electives possible which are 13, 14, 15, 16
and 17.

40.

Bulls Eye

XAT 2015

34
C

www.hitbullseye.com

As per the statements the following figure can be arrived at with the help of variables. The total is given to be 20 and all
common is given to be 1.

QX

JX
Y

1
Y+22

Y+42

More than X
GX
Based on this information in this question along with the number of Only J type given to be 3, the diagram will be

QX
JX

Y
1
Y+42

Y+22

Minimum 4
GX

41.

Now in this case the only value of Y satisfying this will be 1 so that the other two values become 3 and 5 and the total
becomes 20. Now Simran can take 3 only J and 5 both J and G. Similarly Raj can take All Gs and after that Qs. Now
the 5 courses which are common to J and G they both can have and that is the maximum number which can be common
to both.
As per the statements the following figure can be arrived at with the help of variables. The total is given to be 20 and all
common is given to be 1.
QX
J

Y+22

Y+42

More than X
GX

Based on this information in this question along with the number of Only G type given to be 2, the diagram will be

QX
JX
1

Y
1
Y+42

Y+22

2
GX

Bulls Eye

XAT 2015

35

www.hitbullseye.com

42.
43.
44.

A
C
B

45.
46.

D
A

47.

48.

49.
50.

C
D

51.

52.

53.

54.

If only G is given to be 2 only J and Only Q will become 1 each. Now the remaining total of 15 will be assigned to all
the variables of Y. Solving that the value of Y becomes 3 and then 5 and 7. It can be seen that Vijay can take 1 course
of Only J and Raj take can so many other courses and hence they will not share any common electives. Secondly Vijay
can have 1 and 5 common and then Raj can take Only G and then both G and Q. Even in this case, they will not have
any common electives. Hence first option is the answer.
A can reduce enthusiasm of Mr. Arbit as there the benefits =Costs in the case.
Only C reduces uncertainties as it promises 85 % success. B option is uncertain.
As per Mr. Arbit estimate he would have earned Rs. 100000 mn extra for the next five years extra. This foregone
earring would be fetch an interest of 10% compounded annually. As per Mr. Boring he would earn Rs. 100000 mn for
2 years only. Now for the first two years as they will earn same and invest at the same rate this amount would not create
any difference. But for the remaining three years earnings of Mr. Arbit would create a difference in principal as well in
the interest earned. His third year earning of Rs. 100000 mn will earn an interest @ 10% for two years and will become
100000 1.21 = 121000 mn. His fourth year earning of Rs. 100000 mn will earn interest for 1 year and would become
100000 1.1 = 110000 mn. His last year earning of Rs. 100000 mn will be earned at the end of fifth year only. Hence
there is a total difference of 121000 + 110000 + 100000 = Rs 331000 mn.
As outside India the sale is more than 50% i.e. option D is right answer.
IV choice is irrelevant as one, it does not go together with the idea of cutting cost, and second, it does not face
competition is rural markets. Hence choices I , III and II
Going by option 2, first balance can be maintained between personal life and professional and also they both can be
together.
Work life balance can be maintained if Mr. Khan stays back OR She should find equivalent position in LSP USA
and should go along with Mr. Khan.
Only statement II is not related to the question asked rest all the statements speak in favour Ram.
As customers are happy with Ram and business is going well so changing name of Panipat branch to Rams will not
affect the reputation of Mohans at Delhi. In case Rams starts losing business then the name of the food joint can be
changed to Mohans to capitalize on brand value.
For the future it will be better if Mohan himself takes care of the business in the Panipat as he knows that how to handle
and establish a brand.
First term, a1= -64
Last term, l=-100
Common difference, D = - 66 - (-64) = -2
So applying formula for nth term we get
-100 = -64 + (n-1) (-2)
-100 = -64 -2n + 2
-36 -2 = -2n
38
19
164
(-64 -100) =-19
n=
19 Sum of 19 terms =
= -1982 = -1558.
2
2
2
The probability that his friend receives the gift in time will be when his friend receives even one gift. That can be
calculated as the probability of his friend receiving at least one gift. The probability that none of the retailers sends in
time will be 0.4 0.2 0.1 0.5 = 0.004. Now the probability of his receiving at least one gift will be 1 0.004 =
0.996.

A
6m

6m
D

C
14m

55.

6m

6m
Area of figure given =144 square meter
As per said in question BCE becomes square when we will unfold it, so to find the complete area of the figure shown as
dotted after unfolding we need to add the area of triangle BCE.
1
Area of BCE = 66 = 18
2
So the final area of whole figure will become = 144 + 18 = 162 square meter.
Let Manufacturing Cost of the product= Rs.100
55
Maximum Retail Price(MRP) = 100 +
100 = Rs.155
100
Retailer gives 10% discounts. Therefore,
10
Retailers selling price =155 155 =139.5
100

Bulls Eye

XAT 2015

36

www.hitbullseye.com

As per question, The retailer earned 23% profit on his purchase price(say Rs. x).

123
x = 139.5
100

139.5
100
123
13950
x=
= 113.41
123
Now, the purchase price of retailer = x = selling price of Manufacturer
Profit earned by Manufacturer = 113.41 100 = 13.41 i.e. 13.41%
Volume of Cylinder= 49 10 = 490
Now, The solid metal cylinder is re-cast into two cones in the proportion 3 : 4 i.e. the volumes of cone 1 and cone 2 is
210 and 280 respectively.
So, Flat Surface area of cylinder before melting = 2 49 = 98
Volume of cone 1 = (1/3)r12h = 210, where h = 10
r1 = 37
Volume of cone 1 = (1/3)r22h = 280, where h = 10
r2 = 221
Flat surface area of cones = r12 + r22= (r12 + r22) = 147

x=

56.

Now, percentage change in surface area =


57.

147 98
100 50%
98

Perimeter of square ABCD = 200ft


200
AB =
50
4
DB = 50

B X = DY = 7 2 ft (width of road is given as 7 2 ft)

BO = 25 2 BO + BX = 25 2 + 7 2 = 32 2
Take R as radius of bigger circle.
Area of bigger circle = (32

58.

59.

50
50

50

2 )2 ft2 = 2048 ft2

And Area of smaller circle = (25 2 )2 = 1250 ft2


D
Y
22
Area of Road = 2048 1250 = 798 = 798
= 2508 ft2
7
But we have to calculate cost of construction of 50% road.
2508
= 1254 ft2
Required Construction =
2
Cost of 1ft = 100
Cost of 1254 ft = 1254 100 = Rs. 125400
M
5
:
4
X
+ Y
As final product contains M = 864 units
X = 480 and Y = 384
480 units of X and 384 units of Y.
X
1
:
3
A
+ B
120
:
360
Y
2
:
1
B
+ C
256
:
128
total quantity of B in the final product M = 360 + 256 = 616
As final product contains M = 864 units and B is 616 units, therefore
Remaining part of M is 864 616 = 248 units
As concentration of B in the final mixture is 50% or half in the final mixture, therefore
616 = B = Remaining part of M + quantity of water(say W)
616 = 248 + W W = 368 units
Going by options, it is clear from figure that
For only value of x, there are two values of y.
Options A, B and D are eliminated.
Check option C by putting y = 0

50

50
C

Bulls Eye

XAT 2015

37

www.hitbullseye.com

x = 2y2- 40 = 2 (0)2 - 40 = -40


But x is -19, so, option C is false.
Now, put y = 0 in option E, we get x = 2y2 + 3 y 19 = 2(0)2 + 3(0) 19 = -19. Only option E satisfies it.
60.

D
X

(17.5, 23.5)

B
(13.5, 16)

(5.5, 7.5)
Using distance formula

Distance, CX =

17.5 5.52 23.5 7.52

= 144 256 = 20

Distance AC = 2 CX = 40

Distance, BX =
61.

17.5 13.52 23.5 162 =

16 56.25 = 72.25 = 8.5

Distance BD = 2 BX = 17
In ABF
P AB
tan 600=
B 10

10 3 =AB

10 3

600
10
B
300
600

Similarly, ED = 10 3
Similarly in BFC, tan 600=
BC=

P 10

B BC

C
600 10

10
3

Height, AD = 10 3 +10+

30 10 3 10
3

300

300

10
3

40 10 3

10 3

3
40 10 3
1
=50 (4+ 3 )
10 3

2
3

Income Slab(Rs.) Tax rate Min Case


Nil
5
500
5%
4
>500 to 2000
10%
3
>2000 to 5000
> 5000 to <10000
15%
3

Required area of triangle AED =


62.

Max Case
3
3
4
5

Case I- Minimum Value


Tax of 5 employees in case 1 (Minimum case) = 0
(3+3) = 6 employees are having salaries more than Rs.2000,
5
6 =Rs. 450
So, Tax paid by them uptil Rs. 2000 = 1500
100
3 employees are having salaries more than Rs. 5000
10
So, Tax paid by them uptil Rs. 5000 = 3000
3 = Rs.900
100
Minimum total tax paid by 15 persons = 900 + 450 = Rs.1350
We have taken salaries of some employees more than 500, i.e. 501 to calculate the minimum tax.
Actual Value exceeds by some margins.
Case 2- Maximum Value
Tax of 3 employees in case 2 (Maximum case) = 0
(3+4+5) = 12 employees are having salaries more than Rs.2000
5
So, Tax paid by them uptil Rs. 2000 = 1500
12 = Rs.900
100
9 employees are having salaries more than Rs. 5000
10
9 = Rs. 2700
So, Tax paid by them uptil Rs. 5000 = 3000
100

Bulls Eye

XAT 2015

38

www.hitbullseye.com

5 employees are having salaries more than Rs. 5000 but less than Rs. 10000, So to calculate maximum tax, take salary
as Rs. 10,000
So, Tax paid by them uptil Rs. 10,000 = 5000

63.

64.

65.

66.

15
5 = Rs. 3750
100

Maximum total tax paid by 15 persons = 900 + 2700 + 3750 = Rs.7350


We have taken salaries of some employees as maximum of that group
Actual Value will be less by some margins.
To maximize, we have to reduce denominator, So we can make a + b + c = 26 and d = 25 (as maximizing d will give
denominator the least value).
a b c d 26 25
=
= 51
So required maximum value =
a bcd
26 25
f (x2 1) = x4 7x2 + k1
Put x2 = 1 to make it zero
f(0) = 1 7 + k1 = - 6 + k1 .. (1)
f (x3 2) = x6 9x3 + k2
Put x3 = 2 to make it zero
f(0) = (2)2 9(2) + k2 = 14 + k2 (2)
Equating (1) and (2), we get
- 6 + k1 = 14 + k2
k2 k1 = 14 - 6 = 8
In three years period from 2004 to 2006. The interest earned is Rs. 10000 on the principal. Now again three more years
from 2007 to 2009, the total accumulated interest is given to be Rs. 25000. This 25000 will be including the interest of
first three years on the principal i.e. Rs. 10000 for the years 2004 2006 and the interest on the original principal for
the next three years will be again the same i.e. Rs. 10000. This means the balance interest of Rs. 5000 (i.e. 25000
10000 10000) is interest on the Interest earned till 2006 end or 2007 beginning. That implies the rate of interest is 100
5000/10000 = 50% for three years combined. Now for the first three years the interest earned is given to be 10000,
which has to be 50% of the principal. Hence principal is 10000 100/50 = 20000.
Scenario I
D

50km
D 3km/hr
50km

P 4km/hr

After 6 hours, the distance would be minimum as 40 km between D and P.

32

40

67.

24m

Scenario II & Scenario III are ruled out as minimum distance in that case would be 50 kms as after that distance will
keep on increasing between the two.
6, 8, 12, 13, 14, 15, 20, 22
Statement 1
39
= 9.75
Four smallest = 6+8+12+13 =
4
Average four largest Average of four smallest = 13.25
Avg. four largest = 13.25 + 9.75 = 23
total of four numbers = 92
So we can easily allocate other three numbers different minimum values but more than 15 as 15 is median and
maximize the remaining one value
Statement I can answer
Statement 2
110
13.75
8
Avg. of 11 integers is 16

Avg. of these 8 no.s =

Bulls Eye

XAT 2015

39

www.hitbullseye.com

68.

Sum of 11 integers = 16 11 = 176


Remaining Three integer will have sum = 176 110 = 66
So we can easily allocate other three numbers different minimum values but more than 15 as 15 is median and
maximize the remaining one value
Statement II can answer
When BAP = ABP as triangle ABP is isosceles
ABC = 180 2x + x = 180 x
Sin ABC = sin (1800 x) = sin x (second quadrant)
As perimeter of PBCD is 10y = 1000 (Given)
y = 100

4y

1800-2x

120

2x
y

y
1800

1800-2x

2x
P

1y

4y

And perimeter of ABCD = AB + 10y = 1120 AB = 120


In ABP, applying cosine rule, we get
Cos x =

70.

71.

72.
73.

B
B

120
6

2 100 10

36
64
8
4

100
100 10 5
As the number when divided by 3, 4, 5, 6 leaves reminder 2.
Number will be of the form, 60k1 + 2 ..(i)
When divided by 11 it leaves 0 remainder so number will also be of the form, 11k2 (ii)
Hence equating (i) and (ii), we get,
60k1 + 2 = 11k2
60k1 11k2 = -2 or 11k2 60k1 = 2(iii)
It means 60k1 will leave remainder 9 when divide by 11.
In (iii) 60 leaves 5 as remainder when divided by 11
5k1
By remainder root
should leave remainder as 9 or -2
11
Possible values of K1 = 4, 15, 26, 37, 48, 59.
So, required value will be, 60 59 + 2 = 3540 + 2 = 3542
sin x =

69.

1202 1002 1002


2120100
1

Marks
30
All questions correct
Rank1
28.75
1 wrong of 1 mark
Rank 2
28.5
1 left unattempted of 1 mark
Rank 3
27.66
1 left unattempted of 2 mark
Rank 4
27.5
2 wrong of 1 mark
Rank 4
As 1 wrong of 2mark would result in 2.33 deduction (As negative in 2 marks question is 1/3 of a mark for every wrong
answer)
1 wrong of 1 mark lead to deduction of 1.25
1 unattempted of 1 mark lead to deduction of 1.5
1 unattempted of 2 mark lead to deduction of 3 marks
f(x+a) = f(ax)
Also, f (1) = 4 (given)
Now, f(1003)= k
f(1002+1) = f(10021)
f(1003) = f(1002)
Similarly, f(1002) = f(1001) = f (1000)=..= f(1) = 4
So, f(1003)= k = 4
M! N! = 999000
M (M N)
Going by option
A)

25 6 = 150
M = 25! 6 zeros at the last

XAT 2015

40

Bulls Eye
www.hitbullseye.com

N = 19! 3zeros at the last


M! N! = 999000, is feasible
B)

20 9 = 180
M = 20! 4 zeros at the last
N = 11! 2 zeros at the last
M! N! = 999000, is not feasible
C)

25 8 = 200
M = 25! 6 zeros at the last
N = 17! 3 zeros at the last
M! N! = 999000, is feasible
D)

25 9 = 225
M = 25! 6 zeros at the last
N = 16! 3 zeros at the last
M! N! = 999000, is feasible
E)

26 9 = 234
M = 26! 6 zeros at the last
N = 17! 3 zeros at the last
M! N! = 999000, is feasible
So option B is not possible.

74.
75.

D
B

76.

77.

Let the three digit number be abc = 100a + 10b + c is divisible by 10


So, c is zero. So number becomes ab0
After reconstructing the number we get, ba0 = 100b + 10a + 0
The difference of two number = 100b + 10a + 0 - 100a - 10b 0 = 90b - 90a = 90(b-a)
As the difference of the two numbers is divisible by 40 so, 90(b-a) = 40k
40
4
ba=
k b a = k
90
9
We know, (b-a) will be integer when k will beinteger. So possible value of k=9, 18, 27, 36
we can take k up to 18 as difference of b and a cannot be more than 9.
Case I. b a = 4
Possible values of a and b Possible three digit number
9, 5
950
8, 4
840
7, 3
730
6, 2
620
5, 1
510
Case II, b-a=8
Possible values of a and b Possible three digit number
9,1
910
1
1
A cone height becomes
, then volume would become
as radius will also become half by similar triangles. Let
2
8
flask has 24 litres capacity.
1
2r
Now the volume that remained will be 24 = 3 litres
8
24
Pipe As one hour work =
= 3 litres/hour
8
24
Pipe Bs one hour work =
= 2 litres/ hour
r
H
12
24
Pipe Cs one hour work =
= -6 litres/ hour
4
H/2
As option suggested 19 hours all will run simultaneous
(3+2-6)19 net effect =-19
After 19 hours, we left with 5 liters
As per condition
Run A and C together for 1 hour net effect=-3 litres
We left with 2 litre. Run B for 30 minutes to make it 3 litre.
Option C is the answer
Here, we need to find the vote share of all the parties. So, we make the following table:

Bulls Eye

XAT 2015

www.hitbullseye.com

Number of votes
Parties
A
B
C
D
E
Other Parties
Total

78.

A
B
C
D
E
Other Parties

80.

8184.

Year
2010
343200
154000
123200
48400
30800
180400
880000

Gain in vote
share(%)
-2
7
2.5
0
1.5
-9

From the above data, We can observe that the sequence in descending order of gain in vote share is BCEDA.
Following table can be made from the given data :
Parties

79.

Year
2005
343200
154000
123200
48400
30800
180400
880000

Vote share( %age of total


votes)
Year
Year
2005(%)
2010(%)
39
37
17.5
24.5
14
16.5
5.5
5.5
3.5
5
20.5
11.5

Number
of tweets in
2010
131021
108128
96620
41524
32724
15000

Neutral tweets (%)


=100 (Positive tweets % +
Negative tweets %)
31.3 %
39.9 %
40.9 %
33.3 %
37.4 %
100 %

Hence, Maximum number of neutral tweets in year 2010 is for party B.


Vote share ( %age of
Number of votes
total votes)
Parties
Year
Year
Year
Year
2000
2010
2000
2010
A
329700
364450
42 %
37 %
B
133450
241325
17 %
24.5 %
C
196250
162525
25 %
16.5 %
D
27475
54175
3.5 %
5.5 %
E
49250
< 2%
5%
113275
Other Parties
98125
12.5 %
11.5 %

Number of
neutral tweets
in 2010
41009
43143
39517
13841
12238
15000

Gain in vote
share (%)
5
7.5
8.5
2
4.5/

Total Votes
785000
985000
Now, we can clearly see from the table that the gain share can be 7.5% or 2.0%.
Now, for Party E, the vote share in year 2000 can be 0.5 or 1.5, Accordingly, the gain in vote share will be 4.5% or
3.5% respectively.
This gain in vote share can never be 2.5 as for Party E, the vote share in year 2000 can never be more than 2.
Therefore, between 2000 and 2010, the 2.5% gain in vote share cannot be possible for any party.
Here, we can make the following table :
Difference
Number of
Number
Vote
Tweet
between vote
Parties
votes (In
of tweets (In
Share(In
Share(In
share and
2010)
2010)
2010)
2010)
tweet share
B
241325
108128
24.5 %
25.4%
0.9%
C
162525
96620
16.5 %
22.7%
6.2%
D
54175
41524
5.5 %
9.8%
4.3%
E
49250
32724
5%
7.7%
2.7%
Other Parties
113275
15000
11.5 %
3.5%
8%
Total Votes
985000
425017
Now, clearly, the difference is maximum for other parties.
Employee
Days of Training
Effectiveness Score
Undergone
(Scale 1 to 10)
Employees Survey 1 Survey 2 Survey 1 Survey 2
1
9.1
8.5
17
27
2
5
9.5
10
21
3
4.5
7
12
15

Bonus Received in
lacs
Survey 1
31
27.5
15.5

Survey 2
35.5
22
13.5

41

XAT 2015

42

81.

82.

83.

84.

Bulls Eye
www.hitbullseye.com

4
8.8
5.5
18
9
21
18
5
8
6.5
20
18
18
31
6
6.5
8
15
13
23.5
25.5
7
7.3
4
13
25
12
17
Employee number 4 and 5 are going for more than 17 days of training. So bonus earned by Employee 4 and 5 in survey
1 is 21 and 18 lacs respectively. So required average =
Employees having employee effectiveness score higher than 7 in Survey 1 are 1, 4, 5 and 7 but out of these employees,
4 and 7 have bonus lower than 20 lacs in Survey 2.
From Survey 1 and 2, employee, 1, 2, 3 and 7 underwent more days of training but out of these employees, 2 and 3
bonus decreased from Survey 1 to Survey 2.
From Survey 1 and 2, employee, 1, 2, 3 and 7 underwent more days of training but out of these employees, 2 and 3
employee effectiveness score increased by at least 1 rating.

GK Answer key
Set B

Key

1.
2.
3.
4.
5.
6.
7.
8.
9.
10.
11.
12.
13.
14.
15.
16.
17.
18.
19.
20.
21.
22.
23.
24.
25.
26.
27.
28.
29.
30.

D
E
D
D
C
E
E
D
A
E
B
B
A
D
D
D
D
A
B
A
A
A
C
C
E
C
A
E
B
B

Bulls Eye

XAT 2016

www.hitbullseye.com

SECTION A
VERBAL & LOGICAL ABILITY
1.

Number of questions: 26

Assumptions are analogous to the basic ingredients in a gourmet recipe. Only the final product of the
recipe dictates whether the ingredients suffice
1. Good wine needs no advertisement! 2. The apple never falls far from the tree!
3. All is well that ends well!
4. As you sow, so shall you reap!
5. The Proof of the pudding is in the eating!

2.

Read the following poem and answer the question that follows:
I sought a soul in the sea
And found a coral there
Beneath the foam for me
An ocean was all laid bare.
Into my hearts night
Along a narrow way
I groped; and lo! the light,
An infinite land of day
Which of the following would best capture the ESSENCE of the poem above?
1. What lies outside is always deceptive.
2. Pursue the narrow path and avoid the broadways
3. External search is futile; explore the inner space for answers
4. Hearts pathways are broad and clear to find the destination
5. Light offers sight and insight

3.

Which of the following options best captures the relationship similar to INSPECT: VIVISECT?
1. Enquire: Observe
3. Investigate: Interrogate
5. Question: Respond

4.

2. Inquire: Explore
4. Query: Survey

The subject of this book is knavery, skulduggery, cheating, betrayal, unfairness, crime, sneakiness,
malingering, cuttingcorner, immorality, dishonesty, betrayal,graft, wickedness, and sin.
Which of the following options best captures ALL the italicized words above?
1. Aggressive behaviours
3. Deviant behaviours
5. Vetoed behaviours

5.

2. Illegal behaviours
4. Banned behaviours

Read the following conversation:


OINOS: I can comprehend you thus farthat certain operations of what we term Nature, or the
natural laws, will, under certain conditions, give rise to that which has all the appearance of creation.
Shortly before the final overthrow of the earth, there were, I well remember, many very successful
experiments in what some philosophers were weak enough to denominate the creation of animalculae.

XAT 2016

Bulls Eye
www.hitbullseye.com

AGATHOS: The cases of which you speak were, in fact, instances of the secondary creationand of
the only species of creation which has ever been, since the first word spoke into existence the first
law.
Which of the following options CANNOT be DEFINITELY inferred based on the above
conversation?
1. Agathos was explaining something related to creation to Oinos.
2. At the time of conversation there was nothing called Earth.
3. The creation of animalculae is a natural law.
4. Natural laws are creations of philosophers.
5. Law is a spoken word.
6.

Consider the two related statements below:


Statement I: Offices and positions for the marginalized sections should be open to those with greater
savings among them.
Statement II: Offices and positions must be open to everyone based on the principle of fair
opportunity.
Which of the following is true?
1. Statement I assumes that the marginalized sections are incapable of saving.
2. Statement II assumes that all citizens are equally exposed to all opportunities.
3. Statement II contradicts meritocracy.
4. Statement II assumes that all citizens are equally intelligent.
5. Statement I assumes that the marginalized sections always depend on subsidies.

7.

This season will pass. The Prime Minister may not win Lok Sabha elections, or she may; she may not
continue as Prime Minister, or she may. The country will survive whatever the texture of politics in
this decade or the next.
Which of the following, IF TRUE, will BEST reinforce the author's view?
1. The survival of any Prime Minister is dependent on the country's economic growth.
2. The country has a vibrant young working population.
3. The survival of the country depends on a dynamic, growth-oriented Prime Minister, not on the
texture of politics.
4. The previous season had also witnessed similar political uncertainty.
5. The survival of the Prime Minister is dependent on the political texture of the country.

8.

... there is a degree of convergence in the definition of dust which can be summarized as follows:
Trust is a particular level of the subjective probability with which an agent assesses that another agent
or group of agents will perform a particular action. When we say we trust someone or that someone is
trustworthy, we implicitly mean that the probability that he will perform an action that is beneficial to
us ....
Which of the following statements BEST COMPLETES the passage above?
1. is high enough for us to findout if he will cheat us.
2. is high enough for us to consider engaging in some form of cooperation with him.
3. is low enough for him not to engage in negative behaviour against us.
4. is high enough for us not to build defences against his possible aggression.
5. is low enough for us to attack him.

Bulls Eye

XAT 2016

www.hitbullseye.com

9.

The FIRST and the LAST sentences of the paragraph are numbered 1 & 6. The others, labelled as P,
Q, R and S are given below:
1. Suppose I know someone, Smith.
P. One day you come to me and say: "Smith is in Cambridge,"
Q. I inquire, and find you stood at Guildhall and saw at the other end a man and said: "That was
Smith."
R. I'd say: "Listen. This isn't sufficient evidence."
S. I've heard that he has been killed in a battle in this war. 6. If we had a fair amount of evidence he
was killed I would try to make you say that you're being credulous.
Which of the following combinations is the MOST LOGICALLY ORDERED?
1. 1SPQR6

10.

2. 1RSPQ6

3. 1PRSQ6

4. 1QSRP6

4. 1RQPS6

The FIRST and the LAST sentences of the paragraph are numbered 1 & 6. The others, labelled as P,
Q, R and S are given below:
1. The word "symmetry" is used here with a special meaning, and therefore needs to be defined.
P. For instance, if we look at a vase that is left-and-right symmetrical, then turn it 180 around the
vertical axis, it looks the same.
Q. When we have a picture symmetrical, one side is somehow the same as the other side.
R. When is a thing symmetrical - how can we define it?
S. Professor Hermann Weyl has given this definition of symmetry:a thing is symmetrical if one can
subject it to a certain operation and it appears exactly the same after operation.
6. We shall adopt the definition of symmetry in Weyl's more general form, and in that form we shall
discuss symmetry of physical laws.
Which of the following combinations is the MOST LOGICALLY ORDERED?
1. 1PQRS6

11.

2. 1QRSP6

3. 1RQPS6

4. 1RQSP6

5. 1SPQR6

In recent past, Indian football team has lost most of the matches in international football tournaments.
The most successful coaches in Indian club football tournaments are from Latin American countries.
In most of the Latin American countries, football is more popular sport than cricket.
From the passage above, choose the correct option:
1. It can be DEFINITELY concluded that "In India, cricket is more popular than football".
2. It can be DEFINITELY concluded that "Most Latin American countries are successful at football".
3. It can be DEFINITELY concluded that "In recent past, coaches of Indian football teams are not
from Latin America".
4. It can be DEFINITELY concluded that "European football coaches are less successful than their
Latin American counterparts for Indian national team".
5. It cannot be DEFINITELY concluded that "The more popular a sport the better the chance of
producing a successful coach in that sport".

12.

Choose the best pronunciation of the word, Sobriquet, from the following options:
1. soh-bruh-key
4. soub-rick-kaat

2. suub-rry-ka
5. Sobb-rik-kwet

3. sob-bee-ri-kwet

Bulls Eye

XAT 2016

www.hitbullseye.com

DIRECTIONS for questions 13 15: Analyse the following passage and provide appropriate answers for
the questions that follow:
Advances in economic theory in the 1970s and 1980s illuminated the limits of markets; they showed that
unfettered markets do not lead to economic efficiency whenever information is imperfect or markets are
missing (for instance, good insurance markets to cover the key risks" confronting individuals). And
information is always imperfect and markets are always incomplete. Nor do markets, by themselves,
necessarily lead to economic efficiency when the task of a country is to absorb new technology, to close the
"knowledge gap: a central feature of development. Today, most academic economists agree that markets, by
themselves, do not lead to efficiency; the question is whether government can improve matters.
While it is difficult for economists to perform experiments to test their theories, as a chemist or a physicist
might, the world provides a vast array of natural experiments as dozens of countries try different strategies.
Unfortunately, because each country differs in its history and circumstances and in the myriad of details in the
policies - and details do matter - it is often difficult to get a clear interpretation. What is clear, however, is that
there have been marked differences in performance, that the most successful countries have been those in
Asia, and that in most of the Asian countries, government played a very active role. As we look more
carefully at the effects of particular policies, these conclusions are reinforced: there is a remarkable
congruence between what economic theory says government should do and what the East Asian governments
actually did. By the same token, the economic theories based on imperfect information and incomplete risk
markets that predicted that the free flow of short-term capital - a key feature of market fundamentalist policies
- would produce not growth but instability have also been borne out.
13.

"...whether government can improve matters". Here 'matters' indicates


1. Economic efficiency
3. Knowledge gaps
5. Incomplete risk markets

14.

2. Information imperfectness
4. Good insurance markets

Which of the following options CANNOT be inferred from the above passage?
1. Free flow of short-term capital might fail to ensure economic growths
2. Insurance market is a proof that 'markets, by themselves, do not lead to efficiency'.
3. It is difficult to interpret the success of economic policies of Asian countries
4. Technology can impede market efficiency.
5. State intervention and imperfect information can never go hand-in-hand.

15.

Which of the following statements BEST captures the ESSENCE of the two paragraphs in the above
passage?
1. Paragraph I and Paragraph II are parallel arguments that are unrelated.
2. Paragraph I describes markets in general whereas Paragraph II describes market failures in Asian
economies in particular.
3. Paragraph I explains why markets fail. Paragraph II spells out why market based economic theories
fail to explain success of Asian economies.
4. Paragraph I raises question and Paragraph II answers it.
5. Paragraph I states an economic theory and Paragraph II cites a natural experiment to disprove it.

Bulls Eye

XAT 2016

www.hitbullseye.com

DIRECTIONS for questions 16 19: Analyse the following passage and provide appropriate answers for
the questions that follow:
An effective way of describing what interpersonal communication is or is not, is perhaps to capture the
underlying beliefs using specific game analogies.
Communication as Bowling: The bowling model of message delivery is probably the most widely held view
of communication. I think that's unfortunate. This model sees the bowler as the sender, who delivers the ball,
which is the message. As it rolls down the lane (the channel), clutter on the boards (noise) may deflect the ball
(the message). Yet if it is aimed well, the ball strikes the passive pins (the target audience) with a predictable
effect. In this one-way model of communication, the speaker (bowler) must take care to select a precisely
crafted message (ball) and practice diligently to deliver it the same way every time. Of course, that makes
sense only if target listeners are interchangeable, static pins waiting to be bowled over by our wordswhich
they aren't.
This has led some observers to propose an interactive model of interpersonal communication.
Communication as Ping-Pong: Unlike bowling, Ping-Pong is not a solo game. This fact alone makes it a
better analogy for interpersonal communication. One party puts the conversational ball in play, and the
other gets into position to receive. It takes more concentration and skill to receive than to serve because while
the speaker (server) knows where the message is going, the listener (receiver) doesn't. Like a verbal or
nonverbal message, the ball may appear straightforward yet have a deceptive spin. Ping-Pong is a back-andforth game; players switch roles continuously. One moment the person holding the paddle is an initiator; the
next second the same player is a responder, gauging the effectiveness of his or her shot by the way the ball
comes back. The repeated adjustment essential for good play closely parallels the feedback process described
in a number of interpersonal communication theories.
Communication as Dumb Charades The game of charades best captures the simultaneous and collaborative
nature of interpersonal communication. A charade is neither an action, like bowling a strike, nor an
interaction, like a rally in Ping-Pong. It's a transaction.
Charades is a mutual game; the actual play is cooperative. One member draws a title or slogan from a batch of
possibilities and then tries to act it out visually for teammates in a silent mini drama. The goal is to get at least
one partner to say the exact words that are on the slip of paper. Of course, the actor is prohibited from talking
out loud. Suppose you drew the saying "God helps those who help themselves." For God you might try
folding your hands and gazing upward. For helps you could act out offering a helping hand or giving a leg-up
boost over a fence. By pointing at a number of real or imaginary people you may elicit a response of them, and
by this point a partner may shout out, "God helps those who help themselves." Success.
Like charades, interpersonal communication is a mutual, on-going process of sending, receiving, and adapting
verbal and nonverbal messages with another person to create and alter the images in both of our minds.
Communication between us begins when there is some overlap between two images, and is effective to the
extent that overlap increases. But even if our mental pictures are congruent, communication will be partial as
long as we interpret them differently. The idea that "God helps those who help themselves" could strike one
person as a hollow promise, while the other might regard it as a divine stamp of approval for hard work.
Dumb Charade goes beyond the simplistic analogy of bowling and ping pong. It views interpersonal
communications as a complex transaction in which overlapping messages simultaneously affect and are
affected by the other person and multiple other factors.
16.

The meaning CLOSEST to 'interchangeable' in the 'Communication as Bowling paragraph is:


1. Complementary 2. Contiguous

3. Conforming

4. Compatible

5. Comparable

17.

XAT 2016

www.hitbullseye.com

Which of the following options is the CLOSEST to the necessary condition of communication:
1. Threshold overlap of shared images
3. Ability to stimulate affect
5. Ability to elicit a response

18.

Bulls Eye
2. Simultaneous exchange
4. Ability to enact a drama

The two inherent LIMITATIONS of Ping-Pong as a metaphor for communication are:


1. It is governed by conventions with possibility for appeal; it has clear rules.
2. The operating model is win-lose because only one individual or team can win; the receiver can
always predict the spin.
3. The number of players is limited as very few can be meaningfully engaged at a time; the rules of
the game are fixed by the regulators.
4. It demands more skills of the receiver than of the speaker; it is as passive as bowling.
5. Real life communications is like Dumb Charade with multiple players; there are multiple balls used
in Dumb Charade.

19.

Action, interaction and transaction is CLOSEST to:


1. Advertising, Buyer negotiating with a seller, Bidding for a player in Indian Premier League.
2. Preparing an election manifesto, Addressing a public gathering, Engaging in door to door
canvassing.
3. Preparing for MBA entrance exam, Writing the MBA entrance exam, Facing an interview for
business school.
4. Applying for learner licence, Negotiating with a driving school, Driving a Car.
5. Negotiating overseas posting, Applying for visa, Undertaking a journey.

DIRECTIONS for questions 20 22: Analyse the following passage and provide appropriate answers for
the questions that follow:
Each piece, or part, of the whole of nature is always merely an approximation to the complete truth, or the
complete truth so far as we know it. In fact, everything we know is only some kind of approximation, because
we know that we do not know all the laws as yet. Therefore, things must be learned only to be unlearned again
or, more likely, to be corrected.
The principle of science, the definition, almost, is the following: The test of all knowledge is experiment.
Experiment is the sole judge of scientific "truth." But what is the source of knowledge? Where do the laws
that are to be tested come from? Experiment, itself, helps to produce these laws, in the sense that it gives us
hints. But also needed is imagination to create from these hints the great generalizationsto guess at the
wonderful, simple, but very strange patterns beneath them all, and then to experiment to check again whether
we have made the right guess. This imagining process is so difficult that there is a division of labour in
physics: there are theoretical physicists who imagine, deduce, and guess at new laws, but do not experiment;
and then there are experimental physicists who experiment, imagine, deduce, and guess.
We said that the laws of nature are approximate: that we first find the "wrong" ones, and then we find the
"right" ones. Now, how can an experiment be "wrong"? First, in a trivial way: the apparatus can be faulty and
you did not notice. But these things are easily fixed and checked back and forth. So without snatching at such
minor things, how can the results of an experiment be wrong? Only by being inaccurate. For example, the
mass of an object never seems to change; a spinning top has the same weight as a still one. So a "law" was
invented: mass is constant, independent of speed. That "law" is now found to be incorrect. Mass is found to
increase with velocity, but appreciable increase requires velocities near that of light. A true law is: if an object
moves with a speed of less than one hundred miles a second the mass is constant to within one part in a
million. In some such approximate form this is a correct law. So in practice one might think that the new law
makes no significant difference. Well, yes and no. For ordinary speeds we can certainly forget it and use the

Bulls Eye

XAT 2016

www.hitbullseye.com

simple constant mass law as a good approximation. But for high speeds we are wrong, and the higher the
speed, the more wrong we are.
Finally, and most interesting, philosophically we are completely wrong with the approximate law. Our entire
picture of the world has to be altered even though the mass changes only by a little bit. This is a very peculiar
thing about the philosophy, or the ideas, behind the laws. Even a very small effect sometimes requires
profound changes to our ideas.
20.

Which of the following options is DEFINITELY NOT an approximation to the complete truth?
1. I know that I know.
3. I know what I know.
5. I know that others do not know.

21.

2. I know that I do not know.


4. I know what I do not know.

Consider the two statements from the passage:


Statement I: The mass of an object never seems \o change. Statement II: Mass is found to increase
with velocity.
Which of the following options CANNOT be concluded from the above passage?
1. Both statements I and II are approximation to the complete truth.
2. Both statements I and II are complete truth so far as we know.
3. Statement I is an approximation to the complete truth but Statement II is complete truth.
4. Statement I reveals that experimental physicists who imagine, deduce, and guess are
philosophically wrong.
5. Statement II shows that theoretical physicists can pinpoint the shortcomings of experimental
physicists.

22.

'Big Bang' is a popular theory related to the origin of the universe. It states that the universe was the
outcome of a big bang that released enormous energy.
Which of the following is the MOST PROBABLE inference about the big bang theory?
1. Big Bang Theory was first proposed by experimental physicists.
2. Big Bang Theory was first proposed by theoretical physicists.
3. Big Bang Theory was first proposed by experimental physicists and then deduced by theoretical
physicists.
4. Philosophers got the Big Bang theory wrong.
5. Big Bang theory is not an approximation of the complete truth.

DIRECTIONS for questions 23 26: Analyse the following passage and provide appropriate answers for
the questions that follow:
The base of Objectivism according to Ayan Rand is explicit: "Existence existsand the act of grasping that
statement implies two corollary axioms: that something exists which one perceives and that one exists
possessing consciousness, consciousness being the faculty of perceiving that which exists."
Existence and consciousness are facts implicit in every perception. They are the base of all knowledge (and
the precondition of proof): knowledge presupposes something to know and someone to know it. They are
absolutes which cannot be questioned or escaped: every human utterance, including the denial of these
axioms, implies their use and acceptance. The third axiom at the base of knowledgean axiom true, in
Aristotle's words,, of "being qua being"is the Law of Identity. This law defines the essence of existence: to
be is to be something, a thing is what it is; and leads to the fundamental principle of all action, the law of

XAT 2016

Bulls Eye
www.hitbullseye.com

causality. The law of causality states that a thing's actions are determined not by chance, but by its nature, i.e.,
by what it is.
It is important to observe the interrelation of these three axioms. Existence is the first axiom. The universe
exists independent of consciousness. Man is able to adapt his background to his own requirements, but
"Nature, to be commanded, must be obeyed" (Francis Bacon). There is no mental process that can change the
laws of nature or erase facts. The function of consciousness is not to create reality, but to apprehend it.
"Existence is Identity, Consciousness is Identification."
23.

Which of the following is DEFINITELY CORRECT according to the passage:


1. Only what can be perceived exists.
2. What exists is perceived.
3. All that exists does not have consciousness.
4. Consciousness makes perception of being possible.
5. Something to be known and someone to know are the conditio sine qua non for existence.

24.

Which of the following is the ESSENCE of 'The law of Causality'?


1. To be is to be something; 'being qua being'.
2. Wishing to become something else denies the nature of that being.
3. The law of identity is the same as the law of causality.
4. Essence of existence.
5. Actions of a being are determined by its nature.

25.

Which of the following can be best captured as 'Identity and 'Identification'?


1. College as identity; perception of cultural events as identification.
2. Twitter as identity; perception of Twitter as identification.
3. Government as identity; perception of taxation of citizens as identification.
4. Marriage as identity; perception of children as identification.
5. MBA as identity; perception of campus placement as identification.

26.

The author would interpret Francis Bacon's "Nature, to be commanded, must be obeyed" as:
1. Reality should not to be modified or escaped but faced.
2. Man's existence depends on nature's whims.
3. Essentially and objectively nature is superior to humans.
4. Obstacles are better circumvented than confronted.
5. Before channelling nature one, must first comply with it.

Bulls Eye

XAT 2016

www.hitbullseye.com

SECTION B
DECISION MAKING

Number of questions: 23

DIRECTIONS for questions 27 28: Analyse the following caselet and answer the questions that follow:
The City of Yashmund is served by licensed taxis operating on officially sanctioned metered rates and driven
by licensed drivers who do not own the taxis but pay a monthly rent to the taxi-owners. Shaliesh Nair, the
mayor of Yashmund, perceived that most of these taxis do not offer sufficient comfort and safety to
passengers.
27.

The Mayor wants the owners and drivers to care about comfort.
Which of the following decisions, IF TAKEN, is MOST LIKELY to increase the comfort levels of
passengers?
1. The mayor issues a guideline that taxis will be randomly inspected by the police for the comfort
level.
2. The mayor ensures banks grant drivers loans to own cars. Owner driven cars generally offer greater
comfort.
3. The mayor introduces licensing of air-conditioned taxis which can charge increased rates to the rich
customers.
4. The mayor introduces a feedback system that records passenger satisfaction with comfort levels;
this will affect renewal of annual taxi license.
5. The mayor permits doubling metered rates which will ensure enhanced income for owners to invest
in greater comfort.

28.

The mayor wants to involve the car owners in finding a solution to the problem of comfort and safety.
He is concerned that the customers may not be willing to pay more for safety.
Which of the options below is MOST LIKELY to convince the owners?
1. The taxi owners who clear comfort-inspection can charge higher rentals from the drivers and
drivers with impeccable safety record can charge the same from customers.
2. The taxis that clear comfort-inspection can charge 25% above the metered rates; studies have
shown that customers are willing to pay around 18% extra for comfort.
3. If a taxi owner has a consistent record of comfort and safety the government will subsidize a second
loan.
4. Taxis can charge 25% more if they clear comfort-inspection. However, owners of the taxis found
compromising on safety will be jailed.
5. Taxis that pass comfort-inspection test can charge 25% more. Should they violate any traffic rule
this privilege would be withdrawn.

DIRECTIONS for questions 29 30: Analyse the following caselet and answer the questions that follow:
Chatterjee, the MLA of Trikathapur, owes his election success to his close friend and businessman Ghosh.
The victory had appeared unlikely for Chatterjee after the arrival of Bhowmick, a budding politician with
hordes of money. However, his clean image along with Ghosh's money ensured Chatterjee's resounding
victory.
29.

After the elections, Ghosh requested Chatterjee to sanction the land adjoining his factory, for
expansion. However, the requested government land was a green belt reducing harmful pollution from
the factory.

10

XAT 2016

Bulls Eye
www.hitbullseye.com

Which of the following is the BEST option for Chatterjee in these circumstances?
1. Chatterjee should approve the sale only after Ghosh plants a large number of trees around the
factory and the city.
2. Chatterjee should oblige Ghosh provided he recruits 20 locals as his employees on condition that
they plant and maintain a tree each in their locality.
3. As Ghosh is paying market rates Chatterjee should approve the sale with no riders.
4. Chatterjee should approve the sale and ensure that the green belt is shifted to a different tract of
land outside the city, purchased from the proceeds of the sale.
5. Chatterjee should unconditionally approve the transfer of the land to Ghosh as a token of gratitude.
30.

Inspired by Bhowmick's manifesto, Chatterjee is contemplating a green policy which can adversely
affect Ghosh's business interests.
Which of the following actions from Ghosh is likely to convince Chatterjee NOT to pursue this
policy?
1. Request Chatterjee to defer implementation of the green policy by 3 years, the time needed to make
his factory green.
2. Remind Chatterjee that it is for his clean image that people voted him and not for Bhowmick's
green policy.
3. Warn Chatterjee that all industrialists will turn against him and despite his clean image he may be
hated by the industry.
4. Appeal to Chatterjee's sympathy citing the potential loss his business will suffer if the policy were
to be implemented.
5. Threaten Chatterjee that he should not take his loyalty for granted as Bhowmick has invited him to
join his party.

DIRECTIONS for questions 31 33: Analyse the following caselet and answer the questions that follow:
Indian Institute of Research is a Government-established body ,to promote research. In addition to helping in
policy making, it also provides free online access to all the articles to the public. It has a mission of publishing
high quality research articles. Till 2010, the publication of articles was very slow because there was no
incentive for researchers to publish. Researchers stuck to the mandatory one article a year. Most of the
researchers engaged in offering consultancy and earned extra income. Since its inception, the institute was
considered the best place for cutting edge research. The new director of the institute was not happy with the
work done by researchers in silo and came out with a new research policy in 2013 to increase research output
and improve collaboration among researchers. It was decided that extra benefits would be offered to
researchers with new publications. As a result, the number of research articles increased fourfold in 2014. At
the 2015 annual audit, an objection was raised against the new benefits scheme. Auditors1 were not happy
with increased expenses towards remuneration for researchers. Further, the Government opined that the
publication was itself a reward and hence researchers need be paid nothing extra. The director tried to defend
his policy but the response from the government was not encouraging.
1. Note: Auditors role is to verify accounts.
31.

The following facts were observed by an analytics team hired by the government to study the extant
situation.
1. Therewasafour-foldincreaseinthenumberof researchersleavingthe organization in 2014.
2. A researcher died while on duty.
3. The quality jof articles published declined substantially.
4. The average number of people accessing an article decreased by 2%.

Bulls Eye

XAT 2016

www.hitbullseye.com

11

Which of the following options would justify the government's intention to DISCONTINUE the
scheme?
1. 1 and 2
32.

2. 2 and 3

3. 3 only

4. 4 only

5. 3 and 4

The director still wanted to persuade the government to review its stand. He had framed the following
arguments:
1. Most famous researchers in the world are also the highest paid.
2. American institute of research gives extra benefits to its scientists,
3. This year's highest paid researcher had won the Nobel Prize last year.
ConsideringtheGovernmenttobereasonablewhichof thefollowingoptionsis UNLIKELY to convince the
Government?
1. 1 and 2

33.

2. 2 only

3. 2 and 3

4. 1 and 3

5. 1,2 and 3

The director wanted to promote good decision making at Indian Institute of Research. A few trusted
colleagues offered the following suggestions:
1. Auditors need not be allowed to object to extra benefits schemes.
2. Auditors need not pin-point sudden increase in expenditure.
3. Auditors need not be consulted before taking any policy level decision.
Which of the following combination of options should the director agree THE MOST with?
1. 1 and 2

2. 2 only

3. 2 and 3

4. 1 and 3

5. 1,2 and 3

DIRECTIONS for questions 34 35: Analyse the following caselet and answer the questions that follow:
Kamal Chinnappa, Vimal Rao, Ganesh Krishnan and Dinesh Kumar own a saloon each on the Barbil street.
They are the only hairdressers on that street. Each of them offered three services viz. haircut, shaving and
hair-dye. One evening, all four of them met in a nearby tea-stall and agreed to charge ?100 for any of the three
services (haircut, shave and hair-dye) on weekdays. They also agreed to increase this rate to ?115 on
weekends and holidays. All verbally decided to implement the agreement.
34.

The following day Kamal, being the most competent hairdresser on the street, was contemplating
charging higher than agreed upon price.
Which of the following would enable him to charge more with minimal violation of the agreement?
1. He should introduce a new and specialized service at Rs. 130.
2. He should open another shop on the same street and charge Rs. 150.
3. He should charge Rs. 130 for those wanting to jump the queue.
4. He should charge Rs. 115 for a service to a particular customer and give the next service free.
5. He should open his shop two hours before others and close it two hours after.

35.

Vimal relies heavily on a bunch of loyal customers. He is concerned about retaining them.
Which of the following options should he choose if he does not want to violate the agreement?
1. He should charge differential rates for loyal customers.
2. He should charge the loyal customers lower.
3. He should make every third visit free for his loyal customers.
4. He should charge all the agreed upon price.
5. He should allow his loyal customers to jump the queue.

12

Bulls Eye

XAT 2016

www.hitbullseye.com

DIRECTIONS for questions 36 38: Analyse the following caselet and answer the questions that follow:
Six people working at the Bengaluru office of Simsys are planning to buy flats at a real estate project at
Whitefield. Their preferences are listed below:
Person
Bhatia

Designation
Vice President

First Preference
Ground floor flat

Second Preference
Price < Rs. 50lacs.

Third Preference
Shopping
mall
within 5km.

Client
Relationship Distance to office < Recreation Club
Manager
10 km.
Project Manager
Recreation Club
Place for morning Car parking
Khan
walk
Senior
Software Shopping mall with Price < Rs. 30lacks Place for morning
Singh
Engineer
15km
walk
Software Price < Rs. 50lacs
Distance to office <
Yadav Assistant
Engineer
10km
Software Recreation club
Lingdo Assistant
Engineer
Patel

They have identified 7 real estate projects with following facilities available (marked with):
Real
Project
Price

Estate

Distance to office
Place for morning
walk
Recreation Club
Distance
to
shopping mall
Car
parking
facility
Availability
of
ground floor flat
36.

Rs. 6080 lacs


< 5km

Rs. 4550lacs
< 10km

Rs. 2025lacs
> 20km

Rs. 6580lacs
> 15km

Rs. 3545lacs
<2km

Rs. 2540lacs
<10km

Rs. 2030lacs
<5km

Inside

> 25km

< 2km

Inside

< 5km

> 10km

>20km

2. N only
3. P only
5. In all projects at least one person will be 'satisfied'.

Identify the project(s), where AT LEAST 3 of the 6 persons will be 'satisfied'.


1. M only
4. M, Q and S only

38.

Identify the project(s) where NONE of the 6 persons will be 'satisfied'.


1. M only
4. N and P only

37.

2. S only
5. M, Q and R only

3. Q and R only

The marketing managers of all the six projects have agreed to add a recreation club and a car parking
facility to the projects. In this changed scenario identify projects where AT MOST 2 of the 6 persons
will NOT be 'satisfied'.
1. N, Q and R only
4. N and P only

2. P only
5. M, N and P only

3. M and P only

Bulls Eye

XAT 2016

www.hitbullseye.com

13

DIRECTIONS for questions 39 41: Analyse the following caselet and answer the questions that follow:
Purushottam Bhatnagar owns and operates a sweetshop Puru and Sons. He is about 60 years old and is eager
to hand over the business to his sons Ratan and Pramod. He however, fears that his sons, fresh from college
may not understand the tricks of the trade.
39.

Purushottam sends a batch of sweets to the Police station across the street every day. Ratan construed
it as a bribe and wanted to stop this practice.
Which of the following arguments, IF TRUE, would BEST convince Ratan NOT to give up this
practice?
1. In the last three years, three attempts to burgle Puru and Sons were effectively foiled by the Police.
2. Each policeman receives only two pieces of sweet, too small to be considered a bribe.
3. The police in return send two policemen in mufti to mingle with the customers during rush hours to
prevent pickpockets.
4. Every day, Purushottam also sends a batch of sweets to the school next to the station, an orphanage
nearby and the temple at the end of the street.
5. Purushottam's competitor Uttampurush who runs a sweetshop in the same street and his neighbour
Mahapurush who runs a samosa stall, both do similar things every day.

40.

Purushottam's eldest son discovered that the shop repackaged sweets that were close to expiry and
sold them at a discount under different names. These sweets usually get sold very fast. But his son
was concerned about the possible consequences of this practice.
Purushottam was thinking of the following arguments to convince his son.
1. These sweets are consumed the same day and therefore there is no cause for worry.
2. Reduced prices give enough indication about the sweets to the customers.
3. These products are preferred by those who cannot afford full price and in a way, this is a service
done to them.
4. In the past 30 years not a single person has reported ill because of consumption of these sweets.
5. Repackaging and selling sweets is a common practice.
Which combination of arguments below is MOST LIKELY to convince Ratan?
1. 1 and 3

41.

2. 1 and 4

3. 2 and 3

4. 2 and 5

5. 4 and 5

Purushottam's younger son Pramod discovered that10% of their customers whom Purushottam called
privileged customers purchased sweets at prices fixed 10 years ago (which is significantly lower than
the current prices). Purushottam told him, "This 10% are my core and loyal customers with whom I
have personal connect and therefore they deserve this privilege". Pramod refuted his father's argument
citing the following information.
1. These customers form the top 20% of the income bracket of the city.
2. These customers frequently purchase from other sweetshops at market prices.
3. None of them recognises and greets Purushottam at the shop or at anywhere else.
4. None of them was present at Pramod's marriage.
5. These customers actually buy sweets at Puru and Sons for others not part of the core and loyal
customer group.
Which of the following combination of the above will MOST LIKELY convince Purushottam to
charge market price to all?
1. 1 and 2

2. 2 and 4

3. 2 and 5

4. 3 and 4

5. 4 and 5

14

Bulls Eye

XAT 2016

www.hitbullseye.com

DIRECTIONS for questions 42 44: Analyse the following caselet and answer the questions that follow:
Recently a private food testing agency reported the presence of a harmful chemical in Crunchy Chips, a
product of a fast moving consumer goods giant. The report sparked a nationwide outcry.
42.

Rajan Shekhawat, the CEO of the company, feared this incident might affect the company's image
among consumers. Rajan had the following options:
1. Apologizing publicly for this inconvenience and immediately withdrawing the products from all
stores.
2. Communicate 'the correct findings' to the public.
3. Hire a reputed independent testing agency to verify the claims of the report.
4. Establish internal mechanisms to prevent repetition of such incidences in future.
5. Give higher incentives to distributors and retailers for selling the company brands.
Which of the following would be the MOST APPROPRIATE ORDER of options for Rajan, starting
from the immediate?
1. 3,1,5

43.

2. 3,2,4

3. 1,3,5

4. 1,2,5

5. 5,3,2

Mukesh Routray, a shopkeeper in a remote village was surprised to read in the newspaper, his only
source of information, about harmful chemicals in Crunchy Chips. He had stocked a large quantity of
Crunchy Chips for the forthcoming festive season. He also realized that people in his village are
completely unaware of this controversy. He had the following options:
1. Sell the entire stock at a discount before the news spreads.
2. Destroy the entire stock and advise customers not to buy this product from other shops as well.
3. Donate the entire stock of Crunchy Chips to a local orphanage.
4. Inform customers about the controversy but understate its seriousness.
5. Ignore the news and sell the stock at the forthcoming festive season as planned.
6. Explore the veracity of the report and then take decision.
If arranged from ethical to unethical which of the following is DEFINITELY the WRONG order?
1. 6,5,1

44.

2. 6,1,4

3. 4,5,1

4. 2,4,3

5. 2,4,1

An independent and trustworthy confidante of Rajan Shekhawat, the CEO of the company, informed
him that one of their main competitors had bribed the food testing agency to manipulate the report.
Which of the following actions will BEST help Crunchy Chips to bounce back?
1. Proclaim over the media that their product is completely safe.
2. Secretly hire a food testing agency to ascertain the quality of the competitor's product.
3. Hire another food testing agency to test and communicate the outcome to the consumers.
4. File a defamation case against the competitor for their alleged involvement in the conspiracy.
5. File a defamation case against the food testing agency.

DIRECTIONS for questions 45 47: Analyse the following caselet and answer the questions that follow:
Nicky, Manoj and Benita are graduates from a top ranked B-school. They joined ABC corporation a year ago.
ABC is known for its performance oriented culture. This is the first time the organization recruited from a top
ranked B-school. They are part of a five member team with twoothers from lower ranked B-schools. Nicky,
Manoj and Benita draw 40 per cent higher salaries than other team members. This team reports to Amelia
Ganeshmurthi, a senior executive.

Bulls Eye

XAT 2016

www.hitbullseye.com

45.

15

Amelia is disappointed with the performance of Nicky, Manoj and Benita. She came to know that
ABC was not their first choice and they had spent the first ten months applying to other organizations.
However, they have now started liking ABC and promised to do their best henceforth. Amelia has to
rate their annual performance and decide about their future. She has the following choices:
1. Fire them from ABC for insincerity and save the organization's time and money.
2. Give them average ratings with a year to prove their worth and fire them from ABC if they fail to
show significant progress.
3. Impose a pay-cut of 15% since they have not delivered on the promise, but give them relatively
high ratings.
4. Give them relatively poor ratings with one year time to improve and fire them from ABC if they
fail to show significant progress.
5. Give them high ratings and give them a second chance to prove their worth.
Which of the following options rank the above choices in the order of MOST APPROPRIATE to
LEAST APPROPRIATE?
1. 1,2,4

46.

2. 2, 1,4

3. 4,2,5

4. 4, 3, 1

5. 5,2,3

Recruiting Nicky, Manoj and Benita was part of a larger initiative to make the organization attractive
to prospective employees. Recently Amelia's boss informally told her that the trio's perception of the
organization might influence future recruitment from top B-schools. However, the trio had already
expressed their unhappiness about the organization to Amelia. She suspected that her promotion due
next year might depend on the trio!
Which of the following is the BEST way for Amelia to deal with this situation?
1. Henceforth, she should be lenient with the trio.
2. She should promise the trio an early promotion if they can help her recruit good talent from top Bschools.
3. Henceforth, she should occasionally invite the trio for dinner and informal outings.
4. She should tell her boss that it is unfair to link her promotion to the trio's behaviour.
5. She should convey the trio's unhappiness to her boss.

47.

Nicky's performance on the job is disappointing though she is considered a very helpful person
outside the workplace helping her teammates and others in the organization with their personal needs
e.g. finding a place to rent, a good place to get homely food etc. On the other hand, Manoj and Benita
are performing well in their respective jobs and are perceived by their teammates as important to the
team. But they are not interested in helping outside the workplace. Amelia has to decide the future of
the trio. She has the following options:
1. Inform the higher authorities about Nicky's poor performance and ask them to take a call.
2. Send Nicky for a one month training earmarked for top performing employees.
3. Serve Nicky an ultimatum to improve within the next six months or get fired.
4. Even though they performed well, give Manoj and Benita average ratings because of their
disinterest in helping outside workplace.
5. Give Manoj and Benita high ratings based on their performance.
Which of the following combination of above options will be the MOST APPROPRIATE?
1. 1 and 5

2. 2 and 4

3. 2 and 5

4. 3 and 4

5. 3 and 5

16

Bulls Eye

XAT 2016

www.hitbullseye.com

DIRECTIONS for questions 48 49: Analyse the following caselet and answer the questions that follow:
Geetha Gawde can cultivate up to 6 crops a year. Crop A and B are ready for harvest in 2 months; crop C and
D in 3 months, and crop E and F in 4 months. Crop A can be cultivated from January to June; crop B can be
cultivated from April to September; crop C can be cultivated from May to December; crops D as well as E can
be cultivated from August to December, and crop F from November to May. If Geetha plans a change of crop
the soil should be left fallow for one month; however, if the same crop is sown no fallow time is needed.
Sowing takes place only at the beginning of a month. Geetha can only harvest a maximum of 1000 units of
any crop at any point in time. The production cost per unit (incurred at the time of sowing) and price per unit
of crop are as follows:
Crop Production cost per unit crop (in USD) Price per unit crop (in USD)
A
20
60
B
5
55
C
25
70
D
15
75
E
5
65
F
35
75
For Geetha soil preparation does not incur any cost. If a crop is abandoned before the scheduled harvesting,
she gets no money. Geetha is preparing a cropping schedule to maximize her annual profits (i.e. price - cost).
She plans to replicate the schedule in the coming years.
48.

Which of the following would DEFINITELY be a part of the ideal schedule?


1. Cultivate crop B in August or September,
2. Cultivate crop B from April to September.
3. Do not cultivate any crop in August but cultivate crop D in September.
4. Cultivate crop D or crop E in August or September.
5. Do not cultivate any crop in August; but cultivate crop D or crop E in September.

49.

Which of the following schedules would maximize her annual profit while minimising the costs, if
Geetha decides NOT to repeat a crop in a calendar year?
1. Crops A, B and E
3. Crops B, D, E and F
5. Crops A, B, D or E

2. Crops B,D and F


4. Crops C, D and F

Bulls Eye

XAT 2016

www.hitbullseye.com

17

SECTION C
QUANTITATIVE ABILITY & DI
50.

Akhtar plans to cover a rectangular floor of dimensions 9.5 meters and U.5 meters using tiles. Two
types of square shaped tiles are available in the market. A tile with side 1 meter costs Rs. 100 and a
tile with side 0.5 meters costs Rs. 30. The tiles can be cut if required. What will be the minimum cost
of covering the entire floor with tiles?
1. 10930

51.

Number of questions: 29

2. 10900

3. 11000

4. 10950

5. 10430

Anita, Biplove, Cheryl, Danish, Emily and Feroze compared their marks among themselves. Anita
scored the highest marks, Biplove scored more than Danish. Cheryl scored more than at least two
others and Emily had not scored the lowest.
Statement I: Exactly two members scored less than Cheryl
Statement II: Emily and Feroze scored the same marks.
Which of the following statements would be sufficient to identify the one with the lowest marks?
1. Statement I only.
2. Statement II only.
3. Both Statement I and Statement II are required together.
4. Neither Statement I nor Statement II is sufficient.
5. Either Statement I or Statement II is sufficient.

52.

Rani bought more apples than oranges. She sells apples at Rs. 23 apiece and makes 15% profit. She
sells oranges at Rs. 10 apiece and makes 25% profit. If she gets Rs. 653 after selling all the apples and
oranges, find her profit percentage.
1. 16.8%

53.

4. 18.5%

5. 19.1%

2. 2

3. 3

4. 50

5. 99

In the figure below, AB = AC = CD. If ADB = 20, what is the value of BAD?

1. 40
55.

3. 17.9%

Consider the set of numbers {1, 3, 32, 33, .,3100}- The ratio of the last number and the sum of the
remaining numbers is closest to :
1. 1

54

2. 17.4%

2. 60

3. 70

4. 120

5. 140

In an amusement park along with the entry pass a visitor gets two of the three available rides (A, B
and C) free. On a particular day 77 opted for ride A, 55 opted for B and 50 opted for C; 25 visitors
opted for both A and C, 22 opted for both A and B, while no visitor opted for both B and C. 40
visitors did not opt for ride A or B, or both. How many visited with the entry pass on that day?
1. 102

2. 115

3. 130

4. 135

5. 150

18

56.

3. 7293sq. cm.

2. 800 sq. cm.


5. Insufficient data to answer

3. 8002 sq. cm.

The difference between the area of the circumscribed circle and the area of the inscribed circle of an
equilateral triangle is 2156 sq. cm. What is the area of the equilateral triangle?
1. 6863
4. 6503

59.

2. 4863 sq. cm.


5. None of the above

A square piece of paper is folded tinge times along its diagonal to get an isosceles triangle whose
equal sides are 10 cm. What is the area of the unfolded original piece of paper?
1. 400 sq. cm.
4. 1600 sq. cm.

58.

www.hitbullseye.com

ABC and XYZ are equilateral triangles of 54 cm sides. All smaller triangles like ANM, OCP,
QPX etc. are also equilateral triangles. Find the area of the shape MNOPQRM.

1. 2433 sq. cm.


4. 43743 sq. cm.
57.

Bulls Eye

XAT 2016

2. 1000
5. None of the above

3. 9612

A person standing on the ground at point A saw an object at point B on the ground at a distance of
600 meters. The object started flying towards him at an angle of 30 with the ground. The person saw
the object for the second time at point C flying at 30 angle with him. At point C, the object changed
direction and continued flying upwards. The person saw the object for the third time when the object
was directly above him. The object was flying at a constant speed of 10 kmph.

Find the angle at which the object was flying after the person saw it for the second time. You may use
additional statement(s) if required.
Statement I: After changing direction the object took 3 more minutes than it had taken before.
Statement II: After changing direction the object travelled an additional 200V3 meters.

Bulls Eye

XAT 2016

www.hitbullseye.com

19

Which of the following is the correct option?


1. Statement I alone is sufficient to find the angle but statement II is not.
2. Statement II alone is sufficient to find the angle but statement I is not.
3. Statement I and Statement II are consistent with each other.
4. Statement I and Statement II are inconsistent with each other.
5. Neither Statement I nor Statement II is sufficient to find the angle.
60.

For two positive integers a and b, if (a+b)(a+b) is divisible by 500, then the least possible value of a b
is:
1. 8
4. 12

61.

2.

7
3

4.

16
7

2. 929
5. None of the above

2. 120

5.

16
9

3. 951

3. 130

4. 140

5. 150

A water tank has M inlet pipes and N outlet pipes. An inlet pipe can fill the tank in 8 hours while an
outlet pipe can empty the full tank in 12 hours. If all pipes are left open simultaneously, it takes 6
hours to fill the empty tank. What is the relationship between M and N?
1. M:N=1:1
4. M:N = 3:2

65.

9
7

Two numbers in the base system B are 2061B and 601B. The-sum of these two numbers in decimal
system is 432. Find the value of 1010B in decimal system.
1. 110

64.

3.

f is a function for which f(l) = 1 and f(x) = 2x + f(x-l) for each natural number x 2. Find
f(31).
1. 869
4. 991

63.

3. 10

Pradeep could either walk or drive to office. The time taken to walk to the. office is 8 times the
driving time. One day, his wife took the car making him walk to office. After walking 1 km, he
reached a temple when his wife called to say that he can now take the car. Pradeep figured that
continuing to walk to the office will take as long as walking back home and then driving to the office.
Calculate the distance between the temple and the office.
1. 1

62.

2. 9
5. None of the above

2. M:N = 2:1
5. None of the above

3. M:N = 2:3

Company ABC starts an educational program in collaboration with Institute XYZ. As per the
agreement, ABC and XYZ will share profit in 60:40 ratio. The initial investment of Rs. 100,000 on
infrastructure is borne entirely by ABC whereas the running cost of Rs. 400 per student is borne by
YZ. If each student pays Rs. 2000 for the program find the minimum number of students required to
make the program profitable, assuming ABC wants to recover its investment in the very first year and
the program has no seat limits.
1. 63

2. 84

3. 105

4. 157

5. 167

20

66.

Bulls Eye

XAT 2016

www.hitbullseye.com

Study the figure below and answer the question:

Four persons walk from Point A to Point D following different routes. The one following ABCD takes
70 minutes. Another person takes 45 minutes following ABD. The third person takes 30 minutes
following route ACD. The last person takes 65 minutes following route ACBD. If all were to walk at
the same speed, how long will it take to go from point B to point C?
1. 10 min.
4. 40min.
67.

2. 20min.
3. 30min.
5. Cannot be answered as the angles are unknown.

Each day on Planet M is 10 hours, each hour 60 minutes and each minute 40 seconds. The inhabitants
of Planet M use 10 hour analog clock with an hour hand, a minute hand and a second hand. If one
such clock shows 3 hours 42 minutes and 20 seconds in a mirror what will be the time in Planet M
exactly after 5 minutes?
1. 6 hours 18 minutes 20 seconds
3. 6 hours 23 minutes 20 seconds
5. 7 hours 23 minutes 20 seconds

68.

a, b, c are integers. |a| |b| |c| and -10 a, b, c 10. What will be the maximum possible value of
[abc - (a+b+c)]?
1. 524
4. 970

69.

2. 693
5. None of the above

2. 12

l3. 10

4. 9

5. 8

If a, b and c are 3 consecutive integers between - 10 to + 10 (both inclusive), how many integer values
are possible for the expression
1. 0

71.

3. 731

ABCD is a quadrilateral such that AD = 9 cm, BC = 13 cm and DAB = LBCD = 90. P and Q are two
points on AB and CD respectively, such that DQ: BP = 1:2 and DQ is an integer. How many values
can DQ take, for which the maximum possible area of the quadrilateral PBQD is 150 sq. cm?
1. 14

70.

2. 6 hours 22 minutes 20 seconds


4. 7 hours 17 minutes 20 seconds

2. 1

a 3 b 3 c 3 3abc

a b c 2
3. 2

?
4. 3

5. 4

In the figure below, two circular curves create 60 and 90 angles with their respective centres. If the
length of the bottom durve Y is \0n, find the length of the other curve.

1.

15
2

2.

20
2
3

3.

60
2

4.

15
3

5. 15

Bulls Eye

XAT 2016

www.hitbullseye.com

21

DIRECTIONS for questions 72 75: Study the graph below and answer the questions that follow:
This graph depicts the last eight years' annual salaries (in ?lacs.) offered to students during campus placement.
Every year 100 students go through placement process. However, at least one of them fails to get placed. The
salaries of all unplaced students are marked zero and represented in the graph.

The bold line in the graph presents Mean salaries at various years.
72.

In which year were a maximum number of students offered salaries between Rs. 20 to Rs. 30 lacs
(both inclusive)?
1. 2008
4. 2012

73.

2. 2009
5. Cannot be determined

Identify the years in which the annual median salary is higher by at least 60% than the average salary
of the preceding year?
1. 2009, 2010
4. 2009, 2012, 2014

74.

2. 2012, 2014
5. 2009, 2010, 2012, 2014

3. 2009, 2010, 2012

Identify the number of years in which the difference between the average salaries of the top 25% and
the bottom 25% is more than Rs. 20 lacs:
1. 0

75.

3. 2010

2. 1

3. 2

4. 3

5. 4

If the average salary is computed excluding students with no offers, in how many years will the new
average salary be greater than the existing median salary? Refer the table below for number of
students without offers.
Year
Number without job offers
1. 3
4. 6

2008
9

2009
5

2010
20

2011
2

2012
2

2013
4

2014
15

2015
2

2. 4
3. 5
5. Cannot be solved without additional information.

22

Bulls Eye

XAT 2016

www.hitbullseye.com

DIRECTIONS for questions 76 78: Study the data given in the table below and answer the questions that
follow:
Region North
Shop Type
Grocers
34.7
Pan Bidi
7.1
Food Shops
11.8
General stores
12.4
Electrical Hardware
8.3
Chemists
6
Cosmetic Stores
3.8
Others
15.8
Total
100

East

West

South

32
21.2
7.9
9.1
5.6
5.8
3.6
14.8
100

32.2
13.1
14.8

30.2
19.1
12
6.6
5.7
5.7
3.9
16.8
100

12
7.7
5
3.3
12
100

All India
32.4
14.6
11.6
10.1
6.7
5.7
3.7
15.2
100

Based on a survey of 'shop types' Kamath categorized Indian states into four geographical regions as shown in
the table above. His boss felt that the categorization was inadequate since important labels were missing.
Kamath argued that no further labels are required to interpret the data.
76.

A consultant observing the data made the following two inferences:


Inference I: The number of Grocers per-thousand-population is the highest in North India.
Inference II: The number of Cosmetic per-thousand-population is the highest in South India.
Which of following options is DEFINITELY correct?
1. Inference I alone is correct.
3. Either of the inferences is correct.
5. Inference I will be correct only if inference II is correct.

77.

2. Inference II alone is correct.


4. Neither of the inferences is correct.

The average size of Food Shops in East India was twice that of Food Shops in West India. Which of
the following CANNOT be inferred from the above data?
1. As far as 'Food Shops' are concerned, customers in East India prefer spatial surroundings compared
to customers in the West India.
2. As far as 'Food Shops' are concerned, Rentals are very high in West India compared to East India.
3. The ratio of customers buying from 'Food Shops' in East India to customers buying from 'Food
Shops' in West India is 15.8: 11.8.
4. There are 740 'Food Shops' in West India.
5. There are 240 'Food Shops' in South India.

Bulls Eye

XAT 2016

www.hitbullseye.com

78.

23

Bala collected the same data five years after Kamath, using the same categorization. His data is
presented below:
Region North
Shop Type
Grocers
Pan Bidi
Food Shops
General stores
Electrical Hardware
Chemists
Cosmetic Stores
Others
Total

30
7.1
4
12.4
15
7
3.9
20.6
100

East

West

South

All India

32
25
7.9
9.1
5.8
5.8
3.6
10.8
100

32.2
13.J
14.9
12
7.6
5
3.2
12
100

40
19.1
12
7
5.7
5.7
3.9
6.6
100

32.4
14.6
11.6
10.1
6.7
5.7
3.7
15,2
100

Which of the following statements can DEFINITELY be concluded?


1. In the last four years the number of Electrical hardware shop types has increased in North India.
2. In the last four years the number of Grocers shop types has increased in South India.
3. For the last four years in All India the number of Chemists shop types has remained constant.
4. In the four years in East India the number of 'others' shop type has decreased.
5. As per the new survey conducted Pan Bidi shops in East India are next only to Grocers.

24

Bulls Eye

XAT 2016

www.hitbullseye.com

SECTION D
GENERAL KNOWLEDGE
1.

In percentage terms, which Indian state has the highest forest cover?
1. Madhya Pradesh
4. Arunachal Pradesh

2.

2. Harit Pradesh
5. Jammu and Kashmir

3. Nagaland

3. Painter

5. Physicist

Meghnad Saha was a famous:


1. Historian

3.

Number of questions: 30

2. Economist

4. Novelist

Read the following statements:


1 .Russia supports President Assad of Syria.
2. Saudi Arabia opposes President Assad of Syria.
3. Iran supports President Assad./
4. The United States support President Assad
Which of the above statements are true:
1. 1 and 2 only
4. 2 and 3 only

4.

2. ITC

2. Brazil

5. Painter

3. TCS

4. Adani

5. Murugappa

3. South Africa

4. Australia

5. New Zealand

2. Business Consulting Company


4. Business Magazine

The Islands that are under dispute of ownership between Japan and China are:
1. Osaka Islands
3. South China Islands
5. None of the above

9.

4. Historian

The "New Horizon" was in news in 2015. It refers to:


1. Award winning book
3. Mission to Pluto
5. Award winning Oscar film

8.

3. Writer

Which of the following is the closest country to Antarctica?


1. Argentina

7.

2. Physicist

Which of the following is not a conglomerate?


1. Reliance

6.

3. 1,2, 3 and 4

Angus Deaton is a famous:


1. Economist

5.

2. 1, 2 and 3 only
5. 2, 3 and 4 only

2. Muntheetu Islands
4. Senkaku Islands

The Inuit Paradox:


1. is a place in the article circle near Greenland.
2. is that the Inuits eat a lot of fat and hardly any fruits and are still very healthy.
3. is that the Inuits eat anything that moves but spare the reindeer.

Bulls Eye

XAT 2016

www.hitbullseye.com

25

4. is that the inuits have very little exposure to the Sun but suffer no deficiency from Vitamin D.
5. is a Glacier that looks like a circle from a distance but is actually a rectangle.
10.

Takata Corporation was in the news recently because it:


1. was involved in airbag recalls.
2. was the chief aide to Shinzo Abe who promised to revive the economy.
3. was the Commander in Chief of the Japanese army accused by critics of war mongering.
4. was the second largest seller of phones in China.
5. was the name of a band that fuses Japanese and American music.

11.

Which Sea lies to the west of Yemen and Saudi Arabia?


1. The Gulf of Aden
4. Black Sea

12.

2. Gulf of Oman
5. Persian Gull

3. Red Sea

In the budget Estimates of 2015-16:


1. Non-planned expenditure was more than revenue receipts
2. Revenue receipts were higher than non-planned expenditure
3. Planned expenditure was higher than Non-planned Expenditure
4. Fiscal Deficit was lower than Revenue deficit
5. Total expenditure was 20% higher than Total Receipts

13.

Consider the following statements:


1. Sinai is a part of Egypt.
2. Sinai is in Asia.
3. Sinai is in Africa.
4. Russian plane was downed in Sinai in November 2015.
Which of the above statements are true?
1. 1 and 3 only
4. 2 and 4 only

14.

3. Singapore

4. Hong Kong

5. India

3. Jounralist

4. Actor

5. Director

2. Malaria

3. AIDS

4. Syphilis

5. Bird Flu

2. aircraft carrier

3. ship

4. submarine

5. aircraft

2. Painter

Plasmodium Vivax causes:


1. Typhoid

17.

2. New Zealand

Perumal Murugan i s a(n):


1. Writer

16.

3. 1, 3 and 4 only

In 2015, which country was ranked number one as "as best place to do business in"?
1. U.S.A

15.

2. 1,2 and 4 only


5. 3 and 4 only

Pilatus PC-7 is a(n):


1. frigate

26

18.

Bulls Eye

XAT 2016

www.hitbullseye.com

Which of the following commodity is not rightly matched with its country, if 'right match' is
considered as one of the top five producers of that commodity?
1. Oat: Russia
4. Carrot: India

19.

2. Wheat: United States


5. Rice: Indonesia

Which of the following countries has the lowest External Debt?


1. United Kingdom
4. India

20.

2. Lucknow

2. GM

2. Matrix

5. Itanagar

3. Volkswagen

4. Tesla

5. Apple

2. former name of iMac


4. credit card swiping machine

3. Memento

4. Inception

5. Prestige

Timbuktu is:
1. A city in China
3. A form of expression

25.

4. Pithampur

Which of the following films is not associated with Christopher Nolan:


1. Interstellar

24.

3. Jamshedpur

Raspberry PI is a:
1. computer sized credit card
3. credit card sized dessert
5. credit card sized computer

23.

3. USA

Elon Musk is closely associated with:


1. Google

22.

2. France
4. China

Which of the following city is not related to the automobile industry?


1. Dharwad

21.

3. Apple: China

2. A fictional city
4. A city in Mongolia

RuPay is related to:


1. Rural Payment of Wages office of India
2. National association for facilitation of Rural Payments
3. National Payments Corporation of India
4. Russia Pay, a Russian competitor of Paypal
5. Rustic Poetry, an Annual Festival held in Jaipur, India

Essay Topic
Technology and nature are natural enemies

5. A city in Mali

Bulls Eye

XAT 2016

www.hitbullseye.com

27

Answer key & Explanations


Set
B
1.
2.
3.
4.

Key

Explanations

E
C
B
C

Only the final product shows that the proof is in the eating.
I sought a soul tells us that the verses are talking about exploration of inner space.
Vivisect is to inspect deeply: explore is to inquire deeply.000000000000
Deviant refers to actions that violate social norms.
We cannot infer from the conversation that Natural Laws are creations of philosophers as the
facts do not refer to it. It is completely beyond the facts stated in the passage.
Statement II states that positions should be open to everyone, so it assumes that everyone is
exposed to all opportunities
The fact that previous season also had similar situation reiterates the authors view that the
country will survive whatever the texture of politics
We trust a person whose action benefits us and there is a high probability that we can do
something in cooperation with him.
P, Q , R are a mandatory trio and S should follow 1
SP are a mandatory pair, and R has to follow 1 because 1 states that symmetry has to be
defined while R says how can we define it.
Nowhere has it been mentioned that the more successful coaching hinges on the popularity of the
sport.
Check the pronunciation. According to the U. K. accent, it is the best way to pronounce
Refer to the last line of 1st para (Today, most academic. Do not lead to economic efficiency)
All the statements except E can clearly be inferred from the passage.
Refer to the opening line of the passage- Advances in..illuminated the limits of markets. Para
2 illustrates the fact by citing the example of the Asian countries.
Interchangeable literally means apparently identical; very similar. Hence, the option that fits best is
-E
Refer to the 3rd line in penultimate para of the passage- Communication between us begins when
there is some overlap.
The fact that the writer considers Dumb Charade as the best metaphor for communication clearly
shows that the limitation of Ping Pong as compared to it is that the number of players is limited.
The processes mentioned exemplify action, interaction and transaction.
This is not an approximation to the complete truth. Refer the 1st para We know that we do not
know. As such option B is an approximation but Option D is not.
All others find a base in the given information
Theoretical physicists imagine, deduce and guess, do not experiment.
refer last line of the 1st para
refer last line of para 2 The law of causality statesby its very nature, i.e., by what it is.
refer last line of the passage
One needs to learn to comply with the rules of nature before one can command it.
The mayors action will force the owners and the drivers to care about comfort as otherwise they
stand to lose the license.
The only close choices are C and D. We need to address both the issues safety and comfort.
People wont pay more for safety, so the owners can be allowed to charge more for comfort , that
would be an incentive and safety issues can be handled by threat of punishment.
The green belt is needed near the factory to reduce its pollution and Ghosh has requested for the
land. Option A fulfils both the conditions.
Option A and Option b are close. As the statement clearly says that Chatterjee was inspired by
Bhowmicks Manifesto, Option B is the correct option.
The main reason here is quality degradation. Thus, option C is the answer
1, 2, and 3 are weak arguments in favor of giving extra benefits to the scientists.
The role of the auditors is to verify accounts; as such the director cannot ask them not to do this. 1
and 3 do not fall within the role of the auditors, so the director can agree with these options.
All the four hair dressers have agreed to charge a particular sum for three particular services, as
such kamal cannot charge more for them but can charge more for a new specialized service.
All the four hair dressers have agreed to charge a particular sum for three particular services, as

5.

6.

7.

8.

9.

10.

11.

12.
13.
14.

A
A
E

15.

16.

17.

18.

19.

20.

21.
22.
23.
24.
25.
26.

E
B
D
E
B
E

27.

28.

29.

30.

31.
32.

C
E

33.

34.

35.

Bulls Eye

XAT 2016

28

36.

37.

38.

39.

40.

41.

42.

43.

44.

45.
46.

C
E

47.

4849.

48.

49

50.

51.

52.

www.hitbullseye.com

such vimal cannot offer any discounts or charge a different price, so he can only allow his loyal
customers to jump the queue.
For project P, none of the persons will be satisfied. Persons Bhatia, Singh and Yadav are rejected
due to price. Patel, Khan and Lingdo are rejected due to Recreation Club.
M, Q, S are the projects for which at least 3 people will be satisfied.
Project P is not satisfied by 3 persons namely Bhatia, Singh and Yadav even after adding two
facilities to the projects. So definitely person P is out. So only A option satisfies such scenario.
Option D clearly tells that what Ratan understood as a bribe, was not so.
1 and 4 support Purshotams argument that the sweets can be repackaged and can be sold at a
discount.
Purshotam gives discount to the customers for their loyalty 2 and 5 show that the customers are
not loyal and some of those benefitting from the discount are not his loyal customers.
As the CEO, Rajan has to first verify the claims of the report, make the correct findings public. And
make sure that this does not happen in the future.
The order has to be unethical to ethical. As 6, and 2 are ethical actions, so C is the correct option.
The action is needed to bounce back and the statement talks about the competitors manipulation
of the report. Option E addresses this issue.
5 being the least appropriate action the correct option is C.
Option A, B and C are unethical and D is not a prudent action. therefore the best option is B.
Taking note of manoj and Benitas performance 5 is a must; looking at nikkis poor performance at
work, it is best to serve her an ultimatum.
CROP
A
B
C
D
E
F
(in months) 2
2
3
3
4
4
Cultivation January April
May Aug Aug
November
June
September Dec. Dec .Dec. May
The ideal schedule would be
B (April, May) B(June, July) 1 month break (August) D (September, October, November) 1
month (Dec) A (Jan, Feb)
Going by the options.
(A) Profit = A(40) + B(50) + E(60) = 150
(B ) Profit = B(50) + D(60) + F(40) = 150
(C) for crops B, D, E & F Arrangement is not possible.
(D) Profit = C (45) + D(60) + F(40) = 145
(E) For Crops A, B, D/E, Profit = A(40) + B(50) + D/E (60) = 150
Thus, maximum profit is for options A, B and E. But we also have to minimize the costs. The cost
is minimum for Option A.
Area of room = 9.5 11.5 = 109.25 m2
Now, Cost of covering 109 m2 (with 1 1 tile) = 109 m2 Rs. 100 = Rs. 10900
And cost of covering 0.25 m2 (with 0.5 m square tile) = Rs. 30 (for a single tile)
Thus, total cost = Rs. 10,930.
1st statement is not sufficient as nothing is mentioned about D and F. But 2nd statement alone is
sufficient as if E and F scored the same marks, so F cant score lowest. Hence, D scored lowest
marks.
Lets say Rani bought x apples and y oranges. So 23x + 10y = 653. Since x>y, so only option left
for x and y is 21 and 17.
So total CP =
profit %

23
10
21
17 420 136 556.
1.15
1.25

653 556
100 17.4%
556

1, 3, 32, 33, ______ 3100. This is a GP with 1st term 1 and common ratio 3. Last number is 3100.
53.

Sum of remaining numbers =

1 3100 1 3100 1

3 1
2

Bulls Eye

XAT 2016

www.hitbullseye.com

Required ratio

29

3100 2
3100 2

2
3100 1
3100
A

20

100
54.

D
40

140

40

20

Hence BAD = 1000 + 200 = 1200


Drawing the Venn diagram, we get
A

B
30

33

22
0

55.

25

25

15

C
So total number of people = 150.
Point R & M trisect the length AB and MNOPQR is a regular hexagon.
Area of regular hexagon = 6 Area of Equilateral
56.

= 6

57.

3
(18)2 = 486 3 sq.cm.
4

When a square sheet is folded in half, its area is also halved. Now, an isosceles right triangle is
formed after folded. For 2nd & 3rd folds, again areas halved each time and isosceles right triangle
is formed.
Area of last such triangle = 1/2 10 10 = 50
Thus, area of original square = 50
Let, radius of in circle = r
Thus, radius of circum circle = 2r
ATQ, [(2r)2 - (r)2] = 2156
3r2 - 2156

58.

r=

2156 7

3 22

686
3

Now, height of equilateral = 3r

2 2 2
= 400 sq.cm.
1 1 1

Bulls Eye

XAT 2016

30

www.hitbullseye.com

686
3

a
3
2

a, side of =

686 2

Thus, area of triangle

3
3
686 4 = 686 3
a2
4
4

200 3

59.

300

300
600

From the diagram, observe that if object does not change direction at point C then it will be at D
above the person. But since it is given that object changed direction and continued flying upwards,
thus object would reach point E.
Time taken to reach point C (from B) =

200 3
6 3
D 200 3m

min
min .
5
5 10km / h 10 50 / 3

Now, according to statement I, time can increase only when the angle with ground level will
increase.
But, according to statement II, if CD is 200 3m (= BC), then at constant speed, it will not take any
additional time and thus there should not be any increase in angle.

60.

But in first statement direction has been changed whereas in second statement direction has not
been changed. So both the statements are inconsistent with each other.
a+b
If (a + b)
is divisible by 580, then least value of (a + b) is 10. So least value of (a b) would be
9 1 = 9 as a + b = 10 (taking a = 9 and b = 1 or vice versa).
1 km
House

61.

x
Temple

Let walking speed = a. So driving speed = 8a. Let the distance between temple and office = x. So
as per question,

62.

office

x 1 x 1

a a
8a

Solving this equation, we get the value of x as 9/7


f(1) = 1, f(x) = 2x + f(x 1)
Put x = 2, f(2) = 5. Put x = 3, f(3) = 11
2
Putting x = 4, f(4) = 19. We can identify the logic as f(n) = n + (n 1). Putting n = 31, we get the
2
answer as 31 + (31 1) = 961 + 30 = 991.

Bulls Eye

XAT 2016

www.hitbullseye.com

63.

Making the equation as 2B3 + 6B + 1 + 6B2 + 1 = 432


Solving this equation, we get B = 5.
So 10105 would be 130 in decimal system.
Note: This question is technically wrong as we cannot have digit 6 in Base 5.
As per the question, the equation can be formed as

64.

65.

31

1
1
1
M N (M N)
8
12
6

Solving this equation. We get the ratio of M:N as 2: 1


We can get the answer as 2 : 1 if minimum number of pipes is required to be calculated. This case
is satisfied for 2 inlet pipes and 1 outlet pipe. But if we change this to 4 inlet pipes and 2 outlet
pipes, then tank gets filled in 3 hours which violates the condition given in the question. To satisfy
this condition, if 4 inlet pipes are taken, then we need to take 4 outlet pipes. Hence the ratio in this
case becomes 1 : 1. Since multiple ratios are possible, hence answer should be none of these.
For program to be profitable both companies must recover costs before they can start making
profits.
Since, ABC wants to recoup investment in the first year and there is no limit of number of
students, profits can only be shared after both companies can reach a situation of minimum profits
(zero profit), which would be:
400x + 100000 = 2000x.
Left hand side represents total costs and right hand side represents revenues.
Let x be minimum number of students required to reach a situation of minimum profits (in this case
0).
For the entire program the MINIMUM students required for minimum profit is =
100000/(2000 400) = 62.5 = 63.
A

x
66.

67.

68.

As per the question, x + m + w = 70 min


x + y = 45 min, z + w = 30 min, z + m + y = 65 min
Solving these equations, we get the value of m as 30 min.
Required time would be
10 (3hrs 42 min 20 sec) = 6 hrs 17 min 20 sec. So time after 5 min = 6 hrs 22 min 20 sec.
|a| |b| |c| and 10 a, b, c 10.
Max. value of [abc (a + b + c)] will be obtained, when we take a, b, c as 10, - 9, 8. So required
value = 720 (- 10 9 + 8) = 720 (- 11) = 731
10

69.

Q x

13
2x
B

Bulls Eye

XAT 2016

32

www.hitbullseye.com

Lets assume DQ = x and BP = 2x Thus, ATQ


Area of ( BPD + BQD) Area of quadrilateral PBQD

1
1
2 x 9 x 13 150 31x 300 x 300/13 9.68
2
2

Thus, for x to be an integer and positive, 9 different values (1 to 9) are possible.


Since a, b, c are consecutive integers.
Thus, take a = b 1 and c = b + 1
Simplifying
70.

a 3 b 3 c 3 3abc ( b 1) 3 b 3 ( b 1) 3 3( b 1)b( b 1)

(a b c ) 2
( b 1 b b 1) 2

b 3 3b b 3 b 3 3b 3b 3 3b
9b 2
6b 3 3b 2b 2 1

3b
9b 2

Putting different values of b from 9 to 9, we can verify that only 1 and 1 satisfies to get integer
values for the expression.
x
y
A
=

B
P

600
Q

71.

Let P and Q be the centres of the circles with arcs x and y respectively.
ATQ, length of arc y = 10

2r 10
360

2r 10
Where
360
1
Where = 600 2 r 10 r 30
6
Thus,

Thus, AB = 30 (Since AQB is an equilateral ) Further in APB (Right ISOSCELES ), AP =

30
2

(Radius)

Thus, arc length =

72.

73.

74.

15
90
30

2
360
2
2

At least 25% of the students definitely got salaries between 20 - 30 lacs in the years 2008, 2010
and 2012. However, the graphs do not tell the actual salaries. Hence, no comment can be made
on the number of students within a particular salary range. Option E is the correct answer.
It is clear from the above table that in 2012 and 2014, the median of existing year is higher than
1.6 times mean of previous years. Hence the correct answer is: 2012 and 2014. Option B is the
correct answer.
Question wrong as options did not have the right answer.

Bulls Eye
www.hitbullseye.com

75.

76.

77.

78.

33

The answer can be found based on the table below:


New Average Salary = (Old Average Salary) 100/(100 - students without jobs) Based on the
additional information, the new average would be higher than the graph average of all the years.
During two years old averages were already higher than median (2008 and 2013). With 20
students taken out, the new average salary for 2010 would jump to 23.4, which is clearly higher
than median salary of 22. Similarly, the value of 2013 would jump further by at least one hence
making new average definitely higher than existing median. In 2008, even the existing mean is
visibly higher than median. Hence, the three 3 years are: (2008, 2010, and 2013).
Option A is the correct answer.
Since no data is given on population as well as distribution of states into four zones,
it is not possible to calculate per thousand numbers.
Option A and B can be one of the possible conclusions (there are many others) as average size of
food shops in East India is higher.
Option D also is a possible conclusion as percentage given is 14.8, which can be converted into
740 (14.8 50) or a similar multiple of 14.8. Similarly
Option E can also be a possible conclusion since 240 is a multiple of 12
(12 20 = 240).
To conclude option C, we require additional data about number of customers and hence cannot
be concluded on the basis of data above.
Option A cannot be concluded as there is no mention of gross number in the two time periods.
Using a similar rationale, Option B, C and D cannot be concluded.
Option E can be concluded as population for Pan Bidi shops and Grocers is the same. These are
percentages of the same base (total number of shops in East India.

General Awareness Section


Set
B
1.
2.
3.
4.
5.
6.
7.
8.
9.
10.
11.
12.
13.
14.
15.
16.
17.
18.
19.
20.
21.
22.
23.
24.
25.

XAT 2016

Key
D
E
B
A
C
A
C
D
B
A
C
A
B
C
A
B
E
D
D
E
D
E
B
E
C

Indias Fastest Growing Test Prep Portal

Potrebbero piacerti anche